Anda di halaman 1dari 169

Table of Contents

P.A.R.T •
MACHINE DESIGN SHORT PROBLEMS

I would like to dedicate this work to: 1. Strength of Materials 1

2. Mechanics 20

3. Machine Shop 40

My wife gemma Rosa!5lIcorcon

4. Machine Elements 53

5. Stresses 58

To our children,

. 6. Shaft 88

7. Keys 119

Jolin Cristophet and 5?l[e~andra

8. Coupling 130

9. Pressure Vessel 136

For their support . 10. Bolts and Power Screw 149

11. Flywheel 162

12. Spring 172

13. Gears 187

14. Bearing 211

15. Belts 219

16. Brake 227

17. Clutch 231

18. Machineries 239

P.A R."'T'" ••

SITllATIONAL PROBLEMS 274

Strength 0/ MClteriaLs ~ Beams

STRENGTH C)F
1 IIVIA.TE R:J:A.LS

PARTl

"" CANTILEVER BEAMS


S;;HO!RT

... ~ .. .•... .. .. ' . . . .. .. ...• '0' .. , ", ~


1. Couple

~ Maximum Shear =a
~
PRO:BLEM1S
~
~
Maximum Moment = Mo
8 = maximum slope
e = M L

L
~
M

EI

With Fo;rmulas
~ Y = maximum deflection
M L2

y= 2Et

2. Concentrated load at free end

~
~

~ e= 2EI
t P

P L3
~ v > 3E I

3. Concentrated load at the mid-span

~ Shear = P
~ Moment =. -~.
PL
2
~ *-p-

L
2 51 reTlCjth or Materials - Beams St r enqt l i or Materials - Hewns 3
P L3
~e=PL2 ~ v> 48E I
8El
2. Uniform load
5P L3
~ y= 48E:1
~ Shear « wL
2
4. Uniform distributed load
~ Moment= w~
8
3
wL
~
~ e:;; 24Et
4
5wL
~ ~Y:;;384El

~
3. Varying load

5. Uniformly varying load

wL
~
2

~ Moment
L

~. RESTRAINED BEAMS
~

1. Concentrated load at mid-span.

t :
2:F

~ Shear",
c SIMPLY SUPPORTED BEAMS
~ Moment",
PL
~
8
1. Concentrated load at mid-span
3
P PL

- --Lk
~ Y"'192EI
~

.J!i.~.:J A 2. Uniform distributed load


~
wL
~ Shear", ­
2
~
4 St r en.qtl: of MClteriClls Beams
St icnqt t: (if l\,l(l(crials - Beams 5

2 ,", For Rectangular Beams:


~ Moment:;:;··WL
12
1. SI = flexural stress
SI = ,6M
bh 2

Where: M = maximum moment

3. Uniform varying load


2. Ss = shearing stress
3V
~ Moment Ss = --.
. 2bh

Where: V = maximum shear

2
wL
~ Momem at B = --­ L
20 PROBLEM 1 (Apr. 1999)
~ Y = deflection at mid-span A horizontal cantilever beam, 16 ft long is subjected to a load of 500 Ib located to its
wl4
center. The dimension of the beam is 2 x 4 inches respectively. W = 100 Iblft, find
y == 768
its flexural stress.
A. 3150 psi C 4510 pSI
B. 2912 psi D. 5663 psi

SOLUTION
, PROPPED BEAMS

IJ
For a cantilever beam: 500lbs
1. Concentrated load at mid-span with fixed at left end and
roller at right end. = ~ w =100 'b/..
~ ~p
F total load at the center
Hl--.r--.--r--~-1-i
~ y = mid-span deflection •. F = 500 + 100(16) = 2100lbs
L '"""" L = 16 ft
M = maximum moment

2. Uniform distributed load with fixed at left end and pin at M F x (U2)
right end
M 2100(16/2) = 16,800in-lb
M_c_= 16,800(4/2~ = 3150 psi
S =
I (2)(4)3
12
~

PROBLEM 2
A 12 m simply supported beam with 30 KN load at the center has a maximum
2
~ deflection of: (EI = 6000 KN.m )
A. 150 mm C. 170 mm
B. 160 mm D. 180 mm

~
Under simply supported beam table.
6 SI rcn qtli o] Mat erials - Beams St r cn at ! of Materials -Beams 7

y = maximum deflection R 1 (10) = (4 x 2)(8) + 10(2) 4- I'T1 10 KN

PL 3 R 1 = 8.4 KN :2 kN/rT1l ~2rn


y = 48E I
...- *30KN 2:M 1 = 0 10 rn

where:
P = 30 KN ti. 12 m ::6 R2 (10) = (4 x 2)(2) + 10(8)
t R1 R2
L = 12 m R2 = 9.6 KN
2
EI = 6000 KN.m
By cutting at 10 KN section, and consider the right side of the section:
3
(30)(12) =0.180m=180mm MA = R2 (2) = 9.6 (2) = 19.2 KN.m
y = 48(6000)
PROBLEM 5
PROBLEM 3 A 10m simply supported beam with uniform load of 3 KN/m from right end to left end
A 10m simply supported beam with 25 KN load 3 m from the left end has a 2
has a maximum deflection of: (EI = 12,000 KN.m )
2
maximum slope at the left end: (IE = 8000 KN.m ) A. 43.23 mm C. 54.23 mm
A. 00056 rad C. 0.0186 rad B. 32.55 mm D. 36.45 mm
B. 0.0765 rad D. 0.1823 rad
SOLUTION

14-" li it-U'
125 KN y = maximum deflection 'W = 3 KN/nl
2 2 3m... 7rn
Pb(L _b )
5 w L4

fk- ---'--------'­
6 (E I) L
A 10m ~ Y= 384 EI

where:
where: w = 3 KN/m
P = 25 KN L = 10 m
b =7 m EI = 12,000 KN.m
2

L = 10 m
2
EI = 8000 KN.m 4
5 (3) (10) = 0.03255 m = 3255 mm

y = 384 (12,000)

25 (7) (10 2 - 72 )
fk - -':---'--'-,------'- 0.0186 radians

6 (8000) (1 0)

PROBLEM 6

An 8 m simply supported beam has a uniform load of 2 KN/m from left end to right

PROBLEM 4 end and concentrated load of 10 KN at the center has a maximum deflection of: (EI =

A 10m simply supported beam has a uniform load of 2 KN/m extended from left end 2
5,000 KN.m )

to 4 m and has a concentrated load of 10 KN, 2 m from the right end. Find the A. 12.34 mm C. 21.33 mm
2
maximum moment at the 10 KN concentrated load. (EI = 10,000 KN.m ) B. 42.66 mm D. 34.34 mm
A. 19.20 KN.m C. 23.45 KN.m
B. 26.34 KN.m D. 12.34 KN.m
~1-]'·"Im'·
..1-]lIO[-U' Considering the 10 KN concentrated load at the center.

Solving for the reactions at both ends: 3


PL
yl = 48E I
LM2 =0
8 St r enqt ii o] Materials ~ Beams St r c-iunl: of MUlerials -Beam s 9
P L2

iP
3 ()= -­
(10)(8)_ '" 0.021333 m 8EI
yl '" 48(5000)
where: ~ =25 KN
P = 25 KN
y1 '" 21.333 mm L", 14 m
2 L= 14 m
EI '" 9,000 KN.m
Considering the effect of uniform distributed load of 2 KN/m.
25 (14)2
5wL4 5(2 )(8)4 8 0.068 radians
Y2 '" 384 EI
----- '" 0.021333 m = 21.333 mm 8(9000)
384(5,000)
PROBLEM 9
y = y1 + y2 A 10m cantilever beam has a uniform load of 2.5 KN/m from left to right end. Find
2
the maximum deflection of the beam. (EI = 11,000 KN.m ) •
y = 21.333 + 21.333 42.66 mm A. 423. 45 mm C. 323.45 mm
B. 198.23 mm D. 284.10 mm
PROBLEM 7
A 10m cantilever beam has a concentrated load of 10 KN at the free end. Find the
2
maximum deflection of the beam.(EI = 7,000 KN.m ) EI!!IIl'!13I
A. 476.20 mm C. 544.34 mm
B. 342.34 mm D. 764.44 mm W L4
v SET>

~+.J'."[.J". where:

y = maximum deflection

y",
P L3
3EI
~ t P.10 KN
w '" 2.5 KN/m
L = 10 m
EI '" 11,000 KN.m
2

10 m 4
- ~~L = 0.2841 m = 284.10 mm
where: y- 8 (11,000)
P = 10 KN
L = 10 m PROBLEM 10
2
EI '" 7000 KN.m A 10m cantilever beam has a uniform load of 2 KN/m from left to right end and with
concentrated load of 8 KN at the center. Find the maximum slope of the beam. (EI =
2
10(10)3 13,000 KN.m )
Y= - - - - = 0.47619 m = 476.20 mm A. 0.011 rad C. 0.033 rad
3(7000)
B. 0.022 rad D. 0.044 rad

PROBLEM 8
A 14 m cantilever beam has a concentrated load of 25 KN at the mid-span. Find the EI!!IIl'!13I
maximum slope of the beam. (EI = 9,000 KN.m )
2
Considering the effect of the uniform load of 2 KN/m using beam table,
A. 0.00234 rad C. 1.23 rad
B. 0.068 rad D. 0.123 rad
81 "'~-~
EI!!IIl'!13I 6EI
2 (10)3
Using beam table, for a cantilever beam with concentrated load at mid-span, fl l '" '" 0.0256 rad
6(13.000)
10 St renqili of MateriClls . Becuns
Strength of MCllcliu[s lkwns 11

Considering the effect of concentrated load at the mid-span, l


O2 = W L
6EI
2
_8 (10)2
fh =PL 8 (13,000) = 0.007692 rad
8EI
1.5(12)3
tl 2 0.054 radians
6 (8,000)
8 = 81 + 82
8 = 0.002564 + 0.007692 = 0.0333 radians
8 = 81 + 82
PROBLEM 11 8 = 0.072 + 0.054 = 0.126 radians
A 14 m cantilever beam has a load of 16 KN 6 m from the fixed end. Find the
2
maximum slope of the beam. (EI = 6,000 KN.m ) PROBLEM 13
A. 0.183 rad C. 0863 rad A 10m simply supported beam has a triangular load from zero at left end to a
B. 0.048 rad D. 0.064 rad maximum of 10 KN/m at the right end. Find the maximum deflection of the beam. (EI
2
= 10,000 KN.m )
A. 5424 mm C. 84.67 mm
IHN"imD B. 65.10 mm D. 76.56 mm
For a cantilever beam with a concentrated load at a distance from fixed end:

where:
Pa 2
2EI I 6m
IP
T
=16KN
.
L = 14 m
Em!!miD
For a triangular load with simply supported beam,

2.5 W L
4

P = 16 KN Y= 384EI
a=6m
2
EI = 6,000 KN.m
where: w = 10 KN/m
L = 10 m
16 (6)2 EI = 10,000 KN.m
2
8 0.048 radians
2(6,000)

PROBLEM 12 4
y 2.5(10.li1.QL = 0.06510 m
65.10mm
A 12 m cantilever beam has a uniform load of 1.5 KN/m extended from fixed end up -384(10,000)

to 4 m and a concentrated load of 8 KN at the free end. Find the maximum slope of
the beam. (EI = 8,000)
A. 0.126 rad C 0.654 rad PROBLEM 14­
B. 0.234 rad D. 0.345 rad A 8 m cantilever beam has a triangular load from left end to a maximum of 12 KN/m
2
at the fixed end. What is the maximum deflection of the beam? (EI = 12,000 KN.m )
A. 136.53 mm C. 165.34 mm
Em!!miD B. 145.34 mm D. 15434 mm

Considering the effect of concentrated load at free end:


.1e ] , I " [ e 1S'
PL 2 P = 8 KN/m
81 = ­
2EI For a triangular load of cantilever beam,
W =1.5 KN/m
8 (12)2
81 ----

2(8,000)
0.072 radians L =12 m L W
4

y = 30EI

Considering the effect of uniform load of 1.5 KN/m. where:


w = 12 KN/m

j
12 Sf rCllgt h of Materials - Beams
SlrcIII/II, ojl\lul('riuls Beams 13

L=8m
2 I'I<OULJ:-:M 17
EI = 12,000 KN.m
A vertical load of 400 N acts at the end of a horizontal rectangular cantilever beam 2
m long and 25 mm wide. If the allowable bending stress is 130 Mpa, find the depth of
y= _22(8)~ __ the beam.
30 (12,000) - 0 13653 m = 136.53 mm
A. 23.45 mm C. 56.34 mm
B. 38.43 mm D. 45.34 mm
PROBLEM 15
A 12 m restrained beams with two end fixed has a concentrated load of 15 KN at the
2
center. Find the maximum deflection of the beam. (EI = 7,000 KN.m )
E::1!!iirmI
A. 8.45 mm C. 19.28 mm For a cantilever beam with load act at the free end:
B. 34.23 mm D. 24.67 mm P = 400 N = 0.40 KN
M=PL
E::1!!iirmI M = (0.40)(2)
For restrained beam with concentrated load at the center,
M = 0.80 KN.m
PL3
P = 15 KN
Using the formula of flexural stress,

~ ~
Y = 192EI
6M
s= --.
2
bh
~ t. p ::: 400 N
where: P = 15 KN
L = 12 m
EI = 7,000 KN.m
2
L:12m 130000 = 6 (0.80)
2m

, (0.025) h2
15(12)3 h = 0.03843 m = 38.43 mm
0.01928 m = 19.28 mm h
Y = 192-(7,000)

PROBLEM 18
PROBLEM 16
A simply supported beam is 50 mm by 200 mm in 25 mm
A 15 m restrained beam with two end fixed has a uniform load of 3 KN/m, find the
2 cross section and 4 m long. If the flexural stress is not to exceed 8.3 Mpa, find the
maximum deflection of the beam. (EI = 9000KN-m )
maximum mid-span concentrated load.
A. 56.34 mm C. 34.56 mm

A. 2.77 KN C. 3.45 KN
B 23.56 mm D. 43.94 mm

B. 6.34 KN D. 4.34 KN

E::1!!iirmI SOLUTION

For restrained beam with uniform distributed load,


For a simply supported beam with load at the center,
M = maximum moment
wL4 M = PU4
~p

~
Y = 384EI
M=~~
where: w = 3 KN/m
L = 15 m
M=P
4
A 4m -:.a
EI = 9000 KNm
2 L = 15 m Using the shearing stress formula for beam
200 mm
3 (15)4
Y = - - - - - - = 0.043945 m = 43.945 mm s= §~2
384(9000) bh
Where:
]4 SI r ctiqt.t: of Materials - Beams
Sln'T/fl/l, III j\!1u/eriols Beams ]5
b = 0.050 m V = 2(7.5L = 7.5 KN

h = 0.20 m 2

6P 6M 6(14)
._-- - 2 2.77 KN e. S = -- = ~--------- = 26,880 Kpa = 26.880 Mpa
8,300 = (0.05) (0.20)
b h2 (0.05)(0.25)2

PROBLEM 19
A simple wooden beam, 50 mm wide by 250 mm deep and 7.5 m long has a f. S = 2.'-". = _3J7.5l~ = 900 K a
2bh 2(0.05)(0.25) P
maximum deflection of 102 mm under a uniform load "w". E = 12,411 Mpa
a. What is the uniform load, w.
b. What is the slope? PROBLEM 20
c. What is the maximum moment? A simply supported beam having a span of 6 m carries two concentrated loads of 80
d. What is the maximum shear? KN at its middle thirds. Compute the deflection at the mid-span due to this load. Let
9
e. What is the flexural stress? EI = 30,670 x 10 N .rnrn".
f. What is the shearing stress? A. 18 mm C. 24 mm
B. 20 mm D. 30 mm
Em!!'.iIrSmI 4-i-]'lin-gl
a. For a uniform loading beam under simply supported:
5wL4 y = 23PL
3
2mP~ 2m ~P2m
y = 384EI

bh 3
1=­
12
648EI

Y= 23(80,000)(6,000)3 = 20 mm
648 (30,760 x 10 9
)
a 6m ~

(0.050)(0.250 )3 = 0.0000651 m 4
1= 12 PROBLEM 21
A cantilever beam having a span of 6 m, carries a triangular load of 20 KN/m at its
250 mm fixed end to zero at the free end of the beam. Determine the deflection at the free
4 12 2
0.102= 5w(7.5) end of the beam if EI = 50 x 10 N.mm .
384(12411 x 103 )(0.0000651) A. 17.28 mm C. 15.62 mm
B. 16.48 mm D. 12.63 mm
50 mm
w = 2 KN/m
Em!!'.iIrSmI
3
wL 2 (7.5)3
b. e 0.0435 rad
24EI 24 (12411 x 10 3 )(0.0000651) For triangular load of cantilever beam:
w = 20 KN/m = 20 N/mm
c. M = maximum moment wL4
y = 30 EI

M=wL 2/8= 2(7.5)2 = 14KN.m

Y= 20 (6,000)4 = 17.28 mm
8
12
30 (50 x 10 )
d. V = maximum shear = w U2

1
Strength of MUlerials - Beams 17
If) St rcnot u or Materials - Beams

PROBLEM 22 "1-]lliit-a
P = 50 KN
A simply supported rectangular beam 50 mm wide by 100 mm deep carries a uniform

~ t
load of 1200 N/m over its entire length. What is the maximum length of the beam If
PL3
the flexural stress is limited to 20 Mpa.
A 3.33 m C. 5.2 m
Y = 3E1
B. 4.25 m D. 6.0
m
29.63 = 50,000 (4,000)3
38-'­
1i'-]lI"t-U'
EI = 35.9996 x 10 9 N.m
2
= 35,999.6 KN.mm 2 = 36,000 KN.m 2
wL2
Moo --­
8 PROBLEM 25
A restrained beam carries a triangular load which increases from zero to 30 KN/m
s , 6M 100 mm
from A to B having a span of 4 m. Determine the moment at A
bh 2
A. 12 KN.m C. 18 KN.m
6M B. 24 KN.m D. 16 KN.m
20 = ------ 50 mm
(50)(100)2
M = 1,666 x 10 3 N-mm = 1,666 N-m Emut-U' w:;; 30 KN/m
2
1 666 = 1200 L 2
, 8 wL
MA
L = 3.33 m 30
\p
B'
I I I I I ~/
I
A
M = EO) (4)2 L=4 m
16 KN.m
PROBLEM 23
A simply supported beam having a span of 20 m carries a concentrated load of 10 KN
A
36'
at a distance of 5 m from the left support and a counter clockwise couple of 6 KN.m
acting 3 m from the right support. The maximum moment due to this load is: PROBLEM 26
A cantilever beam 3 m long carries a concentrated load of 35 KN at its free end The
A. 39 KN.m C. 36 KN.m
material is structural steel and the maximum bending stress is not to exceed 125
B. 43 KN.m D. 48 KN.m
Mpa. Determine the required diameter of the bar if it is circular

..i-]'II1t-a A. 204.5 mm
B. 188.6 mm
C. 164.6 mm
D. 179.2 mm

LMs = 0

20 R1 = 10 (15) + 6
R1 = 7.8 KN
A6
5

tR'
m+
110 KN

20 m
f 6 KN-m

3m

(\B
SOLUTION

Moo 35 (3) ~
P =35 KN

t
M = 7.8 (5) = 39 KN.m 6
M = 105 KN.m 105 X 10 N.mm

PROBLEM 24 s= ~c

A cantilever beam having a span of 4 m caries a concentrated load of 50 KN at the I

free end. What is the flexural rigidity if the beam deflects 29.63 mm at its free end.
2 2
A. 36,000 KN.m C. 24,800 KN.m
2 2
B. 42,000 KN.m D. 54,200 KN.m
IS SIII'II9Ih of M(lterials - Beams Strength oiMa u-ii at-, Ii/,w,," I' ,

M fJ8 KN.m
\ 125
105xlO h (di2)
1'1\( lllLEM 29
nd
1 64
A restrained beam having a span of 6 m carnes a uniform IOdd 01 ~O Nzrn throughout
6
Its span. If E = 200,000 Mpa, I = 6080 x 10 rnrn" determine the deflection at the mid­
d = 20450 mm span.
A 0.14mm C. 036 mm
PROBLEM 27 B. 0.25 mm D. 0.48 mm
A steel beam 2 m In length is simply supported at each end and carnes a
concentrated load of 100 KN acting 05 m from one of the supports. Determine the Em!!ImD

~
maximum bending stress set up in the beam if the cross-section is rectangular, 100
mm wide by 150 mm deep.
4
A. 100 Mpa C. 97 Mpa wL
y
\ B. 120 Mpa D. 150 Mpa 384 EI
L = 15 m
\ Em!!ImD y-----
50,000 (6) (6,000)3
0.14 mm
\ 6
LMB = 0 384 (200,000)(6,080 x 10 )

2 R1 = 100 (1.5)
0.5 m ~ p = 100 KN
R, = 75 KN

M = 75 (0.50)
6
.a-- 2m- - B
iI'50mm
M = 37.5 KN.m = 37.5 x 10 N.mm

S
6M
= ---.-
bh2
= 6(37.5X 10 6 )
-------~---
100(150)2

tR1
2
S = 100 N/mm = 100 Mpa

100 mm
PROBLEM 28
A beam made of titanium has a yield point of 850 Mpa. The beam has 25 mm x 50
mm rectangular cross-section and bends about an axis parallel to the 25 mm face. If
the maximum bending stress is 650 Mpa, find the corresponding bending moment.
A. 6.8 KN.m C. 4.5 KN.m
B. 8.4 KN,m D. 8.4 KN.m

m:mmmI

S = ---.
6M
bh 2
6M
650= - - - ­
f150mm
(25)(50)2
25mm
M = 6.8 X 10" N.mm
20 Mechunics
Mccllullics 21
Note.
~ If a body is at rest, then initial velocity IS zero.
~ If a body is moving at constant speed, then acceleration IS zero.
~ If a body is completely stop after the trip, then velocity is zero

2. Free Falling Body


Vf=O
a. VI = va ± gt **
" "
,
1''''''­
,,
b. Vl
2
= Vo
2
± 2g8 ,
I
I
••

c. S= Vbl ±%gf
I
I
I


I
I
·
I
I
I
I

IEImm •• I

5 I
where:
·• I
I

r,
• I
g = acceleration due to gravity

Fr =f N ~w g is + if a body is moving down


a is - if a body is moving up vJ
F = frictional force
f = coefficient of static friction
N = normal force
_BL:r Vo = initial velocity
v, = final velocity
S = distance traveled
t = time

tN
-
Note:
~ If a body is dropped freely, then initial velocity is zero
~ If a body is thrown upward, then it reached to a maximum height where the
velocity is zero.
1. Rectilinear motion
- motion is along a straight path. 3. Projectile Motion

-----.
Va Vf
-----. 9=0
~
(. If speed is uniform:
S = vt • - ---~-----~
• H
14 ~I
" If speed varies: s
a. Vr = Vo ± at 2
b. Vf Vo
2
± 2aS c.S= Vat ±% a t;l. R
~
x
where
a = acceleration Components of initial velocity, va

a is + if accelerating

a is - if decelerating
vox =v; cose Voy = va sine

V o = initial velocity

VI = final velocity
For Horizontal Displacement:

S = distance traveled

t = time
"The horizontal component of initial velocity will not change throughout the
=
flight. vox = Vlx contant

l x = horizontal displacement = vox t (v, coss) t


22 Mcch anic:« Mechanics 23

2 . (. For uniform speed:


Vo sln28
" R = maximum range
9
" a
For Vertical Displacement:

, y = verticaldisplacemeI1t vay t 1t':igrS v"Sjn8.t - 112 9 t2 2nRN nDN


" v
" Vfy = final velocity along vertical = Vay • g t.= vasih!:l· gt Where:
w = angular velocity, rad/s, rev/sec
2
a = angular acceleration in rad/s", rev/s
t = time
" 8 = rad or rev
a = acceleration, m/s"
f., Vt == final resultant velocity == R = radius of rotation, m
N = speed, rev/s
v = velocity, m/s
Vo
c;.. H = maximum Note:
29 Use + if accelerating
Where: Use - if decelerating
Vox = initial velocity along horizontal
v oy = initial velocity along vertical
Vlx = final velocity along horizontal -
Vfy = final velocity along vertical
VI = final velocity " Polar Moment of Inertia
Vo = initial velocity
t = time of flight Using English units:
Note:

~ The horizontal component velocity is constant at any time of flight.

~ At maximum point the value of 8 = O.

~ If Y is below the reference point, then the sign of y is negative.

Where:
J m = polar moments of inertia of masses, It-lb-sec"
2
p = Ib!ft
4. Circular Motion
L = length, It
2
g = 32.2 ft/sec
Radian - is a unit of angular measure equal to 57.30°

J = polar moment of inertia of area with constant cross-section, ft4


Angular speed - is the angle through which it turns per unit time.

&zgular acceleration - is the rate of change of its angular speed with respect to time.

Using 51 units:
- e-­
FORMULAS: Wo ••·•
.. ' ..........
Jm = P L J

""\~\
~/
Where:
For accelerating object: 3
Jm = kg_m2 p = kg/m L = length, m J = m"
c;. WI :: Wa ± at
2
~r (, Radius of Gyration
'W?=Wa ± 2a6 :/ WI
Using English Units
-,

" 6 == wo t ± 112 a r ".


"~""
-:

......... __ .::
Mechu7Iics 25
24 Mechu7Iics
K" = radius of gyration (f = anqular acceleration, rad per sec?

P~g
2

Ko == J m := Wk o USing 81 Units:

9
2
T o:= J m a == mko a
Where: Where:
J m = polar moments of inertia of masses, It-lb-sec" To := torque, N-m
2
Ko = ft J,'l := moment of inertia, kg_m
2
g = 32.2 tt/sec" a := angular acceleration, rad per sec

W := weight, Ib

Using 51 Units Kinetic Energy

(. Kinetic Energv of Translating body


2

2
1 2 KE == Wv
KE:= -mv
Where: 2 29
J m:= kg_m2 p:= kg/m 3 Ko:= m m := mass, kg
Where: KE = kinetic energy, Joules
" Radius of Oscillation m:= mass, kg
v = velocity, mls
For isosceles triangle: 314 of height W:= weight, N
For circle: 5/8 of diameter g = acceleration due to gravity, rn/s"
For parabola: 5/7 of the height
(, Kinetic Energy of Rotating body
" Center of Percussion
KE == 1fz Jln {t)2
Q := distance from axis of rotation to center of percussion Where:
J m:= moment of inertia, tt-lb-sec'' ()) = angular velocity, radlsec

(. Total Kinetic Energy

KE := Y2mv2 + Yz. J m {t)2


Where:
r := the distance form axis of rotation to center of gravity of body
(,. Force of a Blow:
(., Formulas relating Torque and Angular Acceleration ws
Average force of blow :=
d
Using English units: Where:
8 := total height, It
W := weight of driver in Ibs
To == J m a o := distance in feet which pile is driven
«:.. Linear Impulse and Momentum
Where:
To := torque in pounds-feet ' W
L Inear rnornentum» m v = - v
2
J", := moment of inertia, ft-lb-sec 9
Ko:= radius of gyration, It
r,
\1­ 26 Mechanics Mcrhultics 27

I Linear impulse

F :=: forcet = time

~ Angular impulse and Momentum


» Fxt

m = mass v = velocity W :=: weight


\, Unbanked curve

tan (8 + x) :=: ==
a
9
v2
-~

gR
H
AOQularmornentUfTl :=: Jm W

Angular impulse :=: To xt Where:


f :=: coefficient of friction
Angular impulse :=: Change in angular momentum R :=: radius of curvature
TdX f=,J m{4>f -wo) 8 = super elevation angle or banking angle

.
~,
. .
x :=: side thrust friction angle
H = super elevation

Fe
\,. Acceleration

Normal acceleration :=: V'/r .~


V

Tangential acceleration :=:dvldt W FR reverse effective force


•I
I

= .m!l = w
, Centrifugal Force
I
j I FR ~a
9
\ Fe :=: rnas, :=: rI1 (l/r) I R
I I

Ig', :=:
wv 2
gr
I
I
I
Direction of Reverse Effective Force
I
... 1. If the object is accelerating, the direction of reverse effective force is
opposite to the direction of motion.
Where:
v = velocity, m/s
r :=: radius of curvature, m
f = coefficient of friction
. FR
----,.~Motion
a = acceleration, m/s"
m = mass, kg
W = weight, N ~' ~n• • _ • • • 9

g :=: 9.81 m/s" = 32.2 ft/s 2


2. If the object is decelerating, the direction of reverse effective force is the
same with the direction of motion.

.
" Banking Curves:
FR

c.
\, Banked curve
H ---.~ Motion
a v2
tan f -;;::._
.....
~._ ..

9 gR i

Where:

m = mass, kg W:=: weight, N a = acceleration, rn/s"

~
:2H !Hech ci nie s Mecltwlics 29

PROllLEM I (OCt. 2000)


A car travels with an initial velocity of 10 m/s or 36 km/hr. If it is decelerating at the
~
rate of 3 rn/s", how far, in meters does it travel before stopping? From the ground to a maximum height:

..,.
A. 17 C. 19 V" = 114km/s==114,000m/s 6 m/s
B. 21 D 15
.....
/ /. £"\
,,'-
VI == 0 at maximum height
PROBLEM
Vt==Vo-gt
Vt
2==V
0
2
-2aS .: ~,

(0)2 == (10)2 - 2 (3) S


o == 114,000 - 9.81 t i
:_------------- --­
~ l
t = 11620.795 sec == 3.228 hrs
S = 16.67 m
Time of object in going up == time of object in going down
PROBLEM 2 (Oct. 2000)
A block weighing 56 Ibs rest on horizontal surface. The force needed to move along So that total time to go up and back.
the surface is 20 Ibs. Determine the coefficient of friction. == 3.228 + 3.228 == 645 hrs
A. 0.0 C 0.36

B 0.112 D.028
PROBU':M 5 (ME Bd. Oct. 97)
An occupant moves toward the center of a merry go around at 6 rn/s If the merry go
Em:!!imD around rotates at 6 rpm. Compute the acceleration component of the occupant
normal to the radius.
F==fN==fW A. 6.79 rn/s" C. 7.54 rn/s"
B. 8.29 m/s" D. 3.77 rn/s"
20 == f (56)

f == 0.357
~
v == 2rcRN

PROBLEM 3 (Oct. 2000)


6 == 2 rt R (6/60)

A baseball is thrown straight upward with a velocity of 20 m/s. Compute for the time

R == 955 m

elapse for the baseball to return. Assume for a zero drag.

A. 1.84 C. 250
a == acceleration
B. 2.21 D. 2.04
v2
a
Em:!!imD R
62 2
a 3.77 m/s
VI = Va - g t 9.55

o == 20 - 9.81 t
PROBLEM 6 (ME Bd. Apr. 98)
t == 2.038 sec A wheel accelerates from rest with a = 5 rad/sec.sec. Compute how many revolutions
are made in 4 seconds.
PROBLEM 4 (Apr. 1999) A. 5.71 rev C.700rev
A 114 km/s speed is projected vertically how long will it take to go back to its initial B. 6.36 rev D. 2000 rev
position.
A. 645 hrs C. 7.23 hrs
B. 1.23 hrs D. 845 hrs ...-r.JX'ur.1~.

,\'

1 rev == 2rc rad

,~
,I';,
..
Mechanics 31
30 Me(' han i ('.s

H c= w.t + 1/2 (f e mv~


mg Fe
w, c= 0 (from rest)
R
o c= 0 + 1/2 (5/2rr)(4)2 i 9 R t V

//--1'_.'l~-.
c= I
e = 6.36 rev / = 9.81(1.2)
v c= 343 m/s

PROBLEM 7 (ME Bd. Apr. 98)

What minimum distance can a truck slide on a horizontal asphalt road if it is traveling Solving for N:
at 25 m/s. The coefficient of sliding friction between asphalt and rubber is at 0.6 The v==2rrRN ;'
, A.2m),

weight of the truck is 8500 kg.


A. 44.9 C. 532
···
'
'
.,
,,

343 = 2 rr (1.2) N · , ,
,
B. 58.5 D. 63.8 "~"""" ...... "
N = 0.455 rev/s X 2rr = 2.86 rad/s
SOLUTION

v = 25 m/s v=o PROBLEM 10 (ME Bd. Apr. 98)


--. --. Compute the speed a satellite to orbit the earth at an elevation of 100 km. Earth's
radius is at 6400 km. Assume no change of gravity with the elevation
I I.. ;1 I
A. 6320 m/s
B. 7120 rn/s
C. 8740 m/s
D. 7920 rn/s

Considering the truck:


Em!!DtmI
FR
-----. To maintain the satellite to orbit the earth,
IFH = 0 Fe = r,
Fr c=

f W = ---a

FR
W
~ L,<.-X,,l, I mv_
2
== mg

g R
2
a c= fg = (0.6)(9.81) c= 5.887 m/s i gR

After the slide it will stop so that V2 = o. v


2
981 (6400,000 + 100,000)
2 2
V2 = V1 + 2aS
o = (25)2 + 2(5.886)S v == 7985 m/s

S = 532 m

PROBLEM 11 (ME Bd. Apr. 98)


PROBLEM 9 (ME Bd. Apr. 98) An elevator weighing 2000 bs is moving vertically upward with an acceleration of 3
A liquid full is to be rotated in the vertical plane. What minimum angular velocity in tt/s". A man standing in said elevator weighs 180 lbs. Compute the tension in the
radians/sec
A.
B.
is needed to keep the liquid not spilling if the rotating arm is 12 meters?
2.26
25.6
C. 3.16
D. 2.86
supporting cable under this condition.
A. 2180lbs
B. 2191 Ibs
C 1820lbs
D. 23831bs
T t
SOLUTION
....{.)'i'iMDI
To keep the liquid not spilling out,
I Fv c- 0 FRt
00
W == total load
f Fe = Fg W = 2000 + 180 = 2180 Ibs

MechcUlics 3:)
32 Mechanics
PROBLEM 14 (ME Bd. Apr. 96)
T W + FH A drop hammer of 1 ton dead weight capacity is propelled downward by a 12 in
W diameter cylinder. At 100 psi air pressure, what IS the Impact velocity if the stroke IS
T W+ - a
g 28 inches?
2180 A. 63.2 fps C. 15.8 fps
T 2180 +~--
32.2
(3) = 23831bs B. 316 fps o 474 fps

PROBLEM 12 (ME Bd. Apr. 98) Em!!ImD


The capsule orbits the earth 180 km above the surface. In what velocity in m/s
necessary for a circular orbit consider the earth radius at 6400 km and at 92 rn/s"? W = 1 ton = 2000 Ibs
A. 8864 C. 7780
B. 7016 O. 8058 Solving for the force acting on piston:

Force = Pressure x Area


Em!!ImD
Force = P (2C 0 2 )
4
Fe = F g
rnv"
~
Force = 100 [ (rr/4)(12)2]

--=mg 6400 km Force = 11,310 Ibs

R
mv 2 F = m a

--=g F = (W/g) a

R w
11,310 = (2000/322) a

R = distance from center of 2


Earth a = 182.1 ft/s

earth to the capsule


v/ = V,2+ 2aS

R = (6400 + 180) 1000 = 6,580.000 m


v, = 0 (from rest)

Substitute: a = 1821 + g
2

a = 1821 + 322 = 2143 ft/s


v2
------=9.2

6,580,000
v} = 0 + 2(214.3)(28/12)

v = 7780 m/s V2 = 31.6 fps

PROBLEM 13 (ME Bd. Apr. 96) PROBLEM 15 (ME Bd. Apr. 96)
A flywheel rotates at 120 rpm or 12.57 rad/sec slowed down to 102 rpm or 10.68
A truck skids to a stop 60 m after the application of the brakes while traveling at 90
rad/sec during the punching operation that requires 3/4 second of the punching
Km/hr. What is its acceleration in rn/sec"?
A. -5.21 rn/sec" C. 6.36
portion of the cycle. Compute the angular acceleration of the flywheel in rad/sec".
B -7.06 O. 5.76
A. -2.52 rae/sec" C. 3.15 rae/sec"
B. -2 75 rae/sec" 0 2.22 rad/sec''

V1
SOLUTION

90(1000)
= --._.---
3600
v/ = V,2 + 2aS
= 25 rn/s
v
~

-
= 90 kph

......
'4

5 = 60 m
~

-
v=o

....,
.... .:r:.~.ji[.l~.

a W2 ="!1

10.68 -12.57
V2 = 0 (stopped) ex -_.---~----

o = (25)2 + 2a(60) 0.75


a = -521 rn/sec"
u -252 rae/sec'

' ,.
\ '~r
34 Mcciuinics Mechanics 35
A. 350 m C. 400 m
B 500 m D. 600 m

PROBLEM 16

A 2-ton weight is lowered at a constant acceleration of 2 rt/s". What is the cable

"'.-]",,[-1\­
stress?

Solving first the acceleration:


A 4,344.34 Ib C. 3,751.55 Ib
Vr = Vo + at
v =20 m/s v = 50 m/s
B. 5,344.56 Ib D. 6,356.24Ib ----+ ----+

SOLUTION
50=20+a(10)

Solving for the distance traveled:



I.
•~I
S =0 Vo t + Y2 a t
2 t = 10 seconds.
Cable Stress
S = 20(10) + 1/2 (3)(10)2

S = 350 m

When lowering the load:

PROBLEM 19
FR = direction of FR is upward since the
direction of load is downward.

FR= - a
W
r F
R
A box sliding on a floor has a frictional resistance of 100 N. If coefficient of friction is
0.3, find the weight of the box.
A. 222.22 N
B. 111.11 N
C. 444.44 N
D. 333.33 N
g

- Ex200~) (2) = 248.45 Ib W =2 tons Em!!ImD


FR - 32.2

Fr =0 frictional resistance

l:F v =0 0 r. = fN
Cable stress = W - FR

Cable stress =0 (2 x 2000) - 248.45

100 = 0.3 N
Cable stress =0 3,751.55 Ibs
N = 333.33 N
PROBLEM 17 W=N =0 333.33 N
The speedometer of a vehicle changes from 20 km/hr to 80 km/hr In 10 seconds.

Find the acceleration for this period in m/s".


PROBLEM 20
A. 1.67 C. 2.34 An object dropped on a gravity travels 300 m after how many seconds?
B. 6.34 0 8.45 A. 5.34 sec C. 7.82 sec
B. 6.34 sec D. 10.23 sec
SOLUTION

V1 = 20 (1000/3600) v =20 kph v =80 kph ~


V1 = 5.55 m/s

----+ ----+
V2 = 80 (1000/3600)

V2 =0 22.22 m/s

• • Va =0
0 (dropped)
~ t = 10 seconds. ~
S t + '12 g t2
V2 = V1 + at
=0 Vo
5 =300 m
22.22 = 5.55 + a(10)

a =0 1.67 rn/s" 2

300 =0 0 + '12 (9.81) t


I
\ PROBLEM 18 t =0 7.82 sec

Ii A car changes accelerates its speed from 20 m/s to 50 rn/s in 10 sec. Find the

,
distance traveled during the period of acceleration.
3G Mechanics Mechanics 37
PROBLEM 21
An object IS projected upward at the rate of 40 m/s. How high will the object rise? (45)2 Sin 2(30)
R
A. 5434 m C 65.34 m 981
B. 7645 m O. 8155 m R 17877 m

SOLUTION PROBLEM 24
A shot is projected 40° from the horizontal at the rate of 30 m/s will reached the
maximum height of:

VI
2
- v, 2 - 2 g S A 20.34 m C. 32.34 m

s B. 18.95 m O. 4534 m
VI = 0 (at maximum height) V =40 m/s
--­
0
2
= (40)2 - 2 (9.81) S t E:mImD
S = 8155 m H = maximum height
2 . 2
H = 'u sin 0.
PROBLEM 22 ~---_.- .~._._,---

A stone is projected upward from the ground travels a maximum height of 50 m. Find 2g
the velocity of stone when it returns to the ground.
(~0)2 sin ~O
2
H =
A. 31.32 m/s C. 4323 rn/s
B. 5434 m/s O. 2345 m/s 2(9.81)

IlE!!i.mD --:-::--0" -,
, ,\
H=1895m

From maximum height to the ground: ' , PROBLEM 25


'
'
I
I
A capsule orbit the earth at the rate of 6000 rn/s. If the radius of earth is 3000 km,
' I

• I
find the height of capsule from the surface of the earth ..
Va = 0 ' I A. 435.66 m C. 543.23 m
' I

2
' I
B. 669.72 m O. 78545 m
Vt = v/ + 2 g S
' I

~
' I

V? = 0

Vf = 31.32 m/s
2
+ 2 (9.81) (50) S'SOml tv V .i·"··"·lh"
The capsule will orbit the earth if:

6400 krn
Fe = Fg
PROBLEM 23 Wv 2
An object is projected upward at the rate of 45 m/s at 30° from the horizontal. Find - - ~W w
gR
the maximum range of projectile.
A. 178.77 m C 18934 m R = distance from center of earth to the Earth
B. 165.34 m o 10934m 2
capsule above the earth surface.
v = g R
,
SOLUTION " /

,
~~
/
(6,000)2 = 9.81 R
~<;: ,

R maximum range "r;;. 30'


" ,

~
R = 3669724.771 m

2 sin 28 R = 3669.724 km

R
g
/'7/7/%7777/// ; \ R = r" + H

I.. .1 3669724 = 3000 + H

H = 669.72 m
38 Mcclwnic!i Mechanic!i 39
PROBLEM 26
(40)2
A car that travels 100 mph reduces its speed 50 mph in 30 seconds. Find the Tan H - ---

distance traveled by the car until it stops. 9.81 (1 000) v = 40 m/s BI""'1II
~LiII- ...
A 4402.10 m C. 5434.56 m Tan H 0.1631 ;' .....
B. 3452.34 m D. 6456.35 m /
/ "' \
,
0
8 = 9.26 I \
\
v =100 mph V = 50 mph V=O I
\
0
R=1000m,
~I
~ ~ ~
SOLUTION
e..
I. Sl
..
~u S2 ~I
\
\ /
I

"' ..... ..... /

Va = 100 (5280/3600) = 14667 ftls


30 sec
...... _-- .... -"

VI = 50 (5280/3600) = 73.33 ft/s

VI = va - at

14667 = 73.33 - a (30)

a = 2.44 m/s"

SI = Vo t - Y2 a t2

S, = 146.67 (30) - Y2 (2.44) (30)2

S, = 3300.10 m

Solving for S2:


2 2
VI = Va - 2a S2

0 2 = (73.33)2 - 2(2.44)S2
S2 = 1102 m

S = total distance traveled


S = 3300.10 + 1102 = 4402.10m

PROBLEM 27
A car traveling at 40 m/s in a radius of 1000 m will have a banking angle of:
0
A. 7.34° C. 9.26
B. 183.4° D. 12.3°

SOLUTION

v2
Tan 8
gR
40 Mact n n«: Shop Practice Machine Shop Pta ct icc 41

m:mmB
IVII~C=""':J: 11'1 E
3 !:» ..... C» F»
Co Time = Length to be,welcl
Welding rate

(; Volume Consumption of gas. V

V = (Va + Va) L

~
Where:
V = total gas consumption of oxygen and acetylene
Va = volume of oxygen consumed per unit length of seam
Va = volume of acetylene needed per unit length of seam
Im!!Jm] L = length of seam

(. Cutting Speed, v: v = 11: 0 N


Where: ~
v = cutting speed
D = diameter of workpiece " Time of Cutting = Length to be cut
N = speed Cutting rate

t.. Time of Milling = Length to be cut PH.OBLEM 1 (Oct. 2000)


Cutting rate Determine the time in seconds, to saw a rectangular magnesium bar 5 in wide and 2
in thick if the length of cut is 5 in. The power hacksaw does 120 strokes/min and the
feed/stroke is 0.127 mm.
(; Time» .~L~e_n..-':g,,--th,-tCC'0_bCC'e_c_ut A 189 C. 99
No. ot teeth x Speed, N x Feed rate B. 500 D. 20

~1·]'·"[·B·

~ Length to be cut
time --"- -.-------

Cutting rate
" Time = Length to be cut
Cutting rate
For a hacksaw. there is only one cut stroke per revolution.
Feed per stroke = 0.127 mm = 0.005 in
'" For a hacksaw, there is only one cut stroke per revolution.
5
c. Time: .__ Lengthtobe cut time = strokes
--_._--_..- - - - - - - - = 8.33 min
in
500 sec
Speed, N x ~trok~ x . Fee.~
120-.
min
(0.005---)
stroke
rev stroke
PROBLEM 2 (Oct. 2000)
In an Oxy-Acetylene manual welding method. to weld a 3 1/2 ft long seam in a 0.375"
thick steel plate at a consumption rate of 9 ft3/ft for oxygen and 7 ft31ft for acetylene.
Compute for the total combined gas consumption in ft3
A 48 C. 56
B 24.5 D. 31.50
42 Machine Shop Practice Machine Shop Prcut ic«: 43
SOLUTION Cutting time- 10.42 min

v = total gas consumption PROBLEM 6 (Oct. 1998)


Compute the cutting speed in fpm of a workpiece with 2 inches diameter and running
v = (Vo + Va) L at 100 rpm.
A. 52 C. 25
V = (9 + 7) (35) = 56 ft3 B. 62 D. 26

PROBLEM 3 (Oct. 1998)


Compute for the drill penetration in in/min when a drill turns at 1200 rpm and the feed
ED:!ImI:I
of 0.005 inlrev. Material is steel. V=ITDN
A. 1.8 C. 12
B. 3.6 D. 6 V = IT (2/12) (100) 52.35 ftlmin

SOLUTION
PROBLEM 7 (ME Bd. Apr. 98)
Drill Penetration = (Feed rate) N Compute the cutting speed in fpm of a workpiece with 2 inches diameter and turning
Drill Penetration = (0.005 --'-~ )(1200 rev)
at 100 rpm?
6 in/min A. 72 C. 62
rev min
B. 102 D. 52
PROBLEM 4 (Oct. 1998)
What is the value in degrees of 1 radian? ED:!ImI:I
A. 5749° C. 57.29°
B. 57.94° D. 57.92° v=rrDN

SOLUTION v = IT (2/12)(100) = 52 ftlmin

1 rad x] 80° 57.29° PROBLEM 8 (Oct. 2000)


IT rad
Determine the average time to cut by automatic oxygen-acetylene (machine)
crosswise a 4 ft x 8 ft x 4 in thick steel plate.
PROBLEM 5 (Oct. 1998) A. 6.85 min C. 10 min
Compute the manual cutting time in minutes, of a steel plate 4 ft by 8 ft by 2.5 cm B. 318 sec D. 360 sec
thick with a hand cutting speed of 3.8 to 4 mm/sec, cutting lengthwise.
A. 10 C. 0.38 -=-~.~
B. 1.81 D. 8.16
Using the average cutting rate of 9 in/min for oxygen-acetylene cutter.
SOLUTION

. Length to be cut
Considering the average value: TIme =
cutting rate

Cutting speed ~8+4 =3.9 mm/sec 4(12) .


2 Time --- -r- 5333 min = 5333(60) 320 sec
9
Length = 8 ft (12) (25.4) = 2438.40 mm
, PROBLEM 9 (ME Bd. Apr. 96)
Length Calculate the rpm for machining a cast iron workpiece 6 inches in diameter. The
2438.40 = 625.23 sec
Cutting time = ·Cuttingspeed
3.9 lowest cutting speed for cast Iron is 50 fpm.
44 Machine Shop Practice
l\!Jachine Shop Pra ct icc 45
A. 35.3 C. 33.3
C. 43.3 O. 53.32 I'I~O()LEM 12 (ME Bd. Oct. 95)
How long Will it take to saw a rectangular piece of aluminum plate 8 in wide and 1 1/2
SOLUTION In. thick if the length of the cut is 8 in, the power hacksaw makes 120 rev/min and
average feed per stroke is 0.0060 in?
A.13.11 C.11.11
v=nON
B 14.01 O. 12.03
50 = rr(6/12)N
E1':'m!immI
N = 31.83 rpm

The answer is: C (nearest) Length of cut


time ------------

Cutting Rate
PROBLEM 10 (ME Bd. Oct. 95)
With the arc welding rate of 18 in/min, how long will It take to weld 1/2 in thick plate by For a hacksaw. there is only 1 cut stroke per revolution.
3 feet long seam?
A. 3 min C. 2 min . &n .
B. 1.5 min O. 4 min 11 A1 J1 time = - - - -.--- -------..- - - . - - = 11.11 min
.r.
120 ev x~troke xO.0060 In
1/2" min rev stroke
SOLUTION

Length to be weld PROBLEM 13 (ME BD. Oct. 95)


time - ' " - - - - - -

Welding rate Using oxyacetylene welding method to weld a 3 ft long seam in a 3/8 in thick plate at
L = J' a consumption rate of 9 ft31ft of weld for oxygen and 7 felft for acetylene. What is the
total combined gas consumption in ft3?
time = (3 x 12)/18 = 2 min A. 51 B. 48
C. 45 O. 55
V V
PROBLEM 1 1 (ME Bd. Apr. 95) E1':'m!immI
How long will it take to mill a 3/4" by 2" long keyway in a 3" diameter shafting with a
24 tooth cutter turning at 100 rpm and 0.005" feed/tooth? v total gas consumption of oxygen and acetylene
A. 0.136 min C. 0.196 min
B. 0.166 min O. 0.106 min ft3
v (9 + 7 ) - - - (3) ft length = 48 ft3
ft length
SOLUTION

PROBLEM 14
Length to be cut A grinding wheel rotates 1750 rpm and has a surface speed of 2290 ft/min. Find the
time -----_. - . -
diameter of the wheel in inches.
Cutting rate A. 3 in C. 4 in
B. 5 in O. 6 in
time 2in
24 teet~ x1 o~e_v x~.OO~-~--- "-N !Ii it-D
rev min tooth
v=rrON
time 0.167 min 2290 = rr (0) (1750)
0=5 in
46 Mcicttinc Shop Practice MUc!liTI<' Shop Practice 47
PROBLEM 15 (ME Bd. Apr. 96) o = 0.955 ft (12) == 11.461n
To facilitate the milling (roughing) work of a cast steel material using a 1 1/2 inch
diameter cutter choose between the two available stock. Find the speed of the
cutter in rpm. -high speed steel cutter with a cutting speed of 50 fpm - carbide tipped PROBLEM 18
cutter with a cutting speed of 200 fpm A workpiece turning at 20 rad/s has a diameter of 4 in. Find the peripheral speed of
A. 389 C. 572 workpiece
B. 509 O. 412 A. 1.23 fps C. 5.23 fps
B. 3.33 fps O. 7.34 fps
SOLUTION
EmmmD
v=nON
N 20 ( 1rev)
High speed steel cutter (slower) 2n
N 3.183 rev/s
50(12) = n(1.5)N
v == nON = IT (4/12) (3.183) = 3.33 fps
N = 127 rpm

Carbide tipped cutter (faster)


PROBLEM 19
Two plates 4 ft x 8 It plate % in thick is to be joined lengthwise. A welding machine to
be used has a welding rate of 20 in/min. How long will it take to join the plates?
200(12) = n(1.5)N
. A. 7.34 min C. 4.80 min
B. 1.23 min O. 6.45 min
N = 509 rpm
PROBLEM 16 EmmmD
A piston reciprocates 100 times per minute (single stroke). The length of the stroke is
6 in. What is the average wristpin velocity in ft/min?
A. 50 ft/min C. 75 ftimin \
B.100ft/min 0.125ft/min 4"
Length to be weld
SOLUTION Time --------._--
Welding rate

v == LN , ~

( I
Time 8 x 12' = 4.8 min 8" 8''''
ft stroke . 20
v == (6/12) _ . - (100) ---.- = 50 ft/rnin
stroke rrun

PROBLEM 17
A workpiece turninq at 300 rpm has a tangential speed of 15 ft/s. Find the diameter. PROBLEM 20
of the workpiece. I A 10 It long seam can be weld in 10 min. The welding rate is:

A. 11.46 in C. 13.23 in A. 8 in/min C 10 in min


B. 15.23 in O. 17.34 in B. 12 in/min O. 14 in/min

EmmmD EmmmD
V=nON . Length to be weld
Time = . - - - ----.---
Welding rate
15 == IT (0) (300/60)

~fi." _
4~)
i'vlachirlc ShoJi )'/rl( I ice
48 Machine Shop Prartire
10(12)
1l4·I l!mm
10 -- -- --- -----
Welding Rate Length to be cut
time = - ,-----'
Cutting rate
Welding rate 12 in/min

5
PROBLEM 21 time
0.2
A seam can be weld in 3 min. If welding rate is 15 in/min, find the length of seam.
A. 20 in C. 25 in
time 25 sec
B. 30 in D. 45 in

SOLUTION
PROBLEM 24
It takes 1 min to mill a 3;'/ x 4" keyway of a hub by means of a cutter having 30 teeth
at 120 rpm. Find the equivalent feed per tooth, In/tooth
Length to be weld A 000222 C 0.00333
Time -------~--

Welding rate B 000444 D. 0.00111


L

Length to be weld
3 ---------~

15 EI!!ImD
Length to be weld = 45 in Length to be cut
time ---.---------_._-
Cutting rate
PROBLEM 22 4
- - ---- ,---~ ------.
How long will it take to mill %" by 1.5" long keyway by means of 25 teeth cutter turning rev teeth
at 120 rpm and 0.008" feed/tooth. 120 . x 30 -- - x Feed per tooth
min rev
A. 1.23 sec C. 4.23 sec
B 634 sec D. 375 sec
Feed per tooth = 0.00111 in/tooth

SOLUTION PROBLEM 25
Length to be cut How long will it take to cut a rectangular plate 4 it wide by means of a saw that has a
time -- -~--
speed of 160 rev/min and an average feed per stroke of 0.009 in?
Cutting rate
A. 22.22 min C. 55.55 min
B 3333 min D. 44.44 min 4"
1.5
time ------------"
120 ~ev x 25 teeltl x O.OOB in EI!!ImD
min rev tooth
Length to be cut
time 0.0625 min (60) = 3.75 sec time -.--- ---

Cutting rate
-

For a hacksaw, there is only one cut stroke per revolution.


PH.OBLEM 23
A 5 in long keyway is 10 be mill at the rate of 0.2 in/min. How long will it take to mill
4 (12) = 33.33 minute
the keyway? lime = .' ----.
A. 20 sec C 25 sec 160 rE3~ x l~trokE3. x 0.009 in
B. 15 sec 0 30 sec min rev stroke
50 ,~!uchine Shop Piact icc r\i!uchinc Shop Prac t icc 51

PROBLEM 26
A 4ft wide x 8 ft long and 1,12 In thick steel plate is be cut into 4 equal parts crosswise ~
If cutting rate is 0.02 In/stroke and saw speed is 110 rpm, how long will it take to do
the job?
A. 53.45 min C. 65.45 min L total length of cut 4"
B. 3423 min D. 67.45 min
4" L 4 cuts (4 ftl cut) = 16 ft
SOLUTION
Length to be cut 8"
time ---, ..- - - - - - - - -
L total length of plate to be cut Cutting rate
L 3 cuts (4 ft / cut) = 12 ft
16 (12)
Length__to be cut
.._--
time 19.20 min
time --
10
_
.
~
~
-
_
.
_
-
Cutting rate
PROBLEM 29
For a hacksaw, there is only one cut stroke per revolution. USing oxyacetylene welding method to weld 10ft long seam In a 1,12 In thick plate at a
12(12) . 3 3
time = ---.-------~------__c_-- = 65.45 min consumption rate of 10ft /ft oxygen and 8 ft /ft for acetylene. What is the total gas
rev stroke In consumption?
110- x 1 - --- x 0.02
min rev stroke A 150 ft3 C,160fe
B. 170 ft'! o 180 fe
PHOBLEM 27 1/2"

EW'M~':sazc~
It takes 5 min to cut a plate 20 ft long by means of a saw having a speed of 250 rpm. ~
What is the average feed per stroke 01 the saw, in/stroke?
A 0.534 C. 0.745
B. 0.192 D 0.342 v Total gas consumption

~ v Volume ()f~9~ (Length to be weld)


length to be weld
V (10+8)(10)
Length to be cut
-_.__
time .~_-----

V 180 ft'!
Cutting rate 11
20
PROBLEM 30
For a hacksaw, there is only one cut stroke per revolution. A 24 ft2 lateral surface for a cylindrical tank is to be made by joining 2 ft by 2 ft steel
3/lt
plate to form 6 It x 4 ft. The consumption rate for both oxygen and nitrogen is 20 ft
20 (12) using oxyacetylene welding method. Find total gas consumption needed, fe.
5 = - -.---- .----.--.----.---
rev stroke A, 250 fe C. 440 fe
250 .-- x 1 -- - x Feed per stroke 3
min rev B. 470 ft3 D. 280lt

Feed per stroke o 192 in/stroke ~ 2"


2"
PROBLEM 28 4"
An oxyacetylene cutter IS use to cut a 4 ft x 8 ft steel plate crosswise and divided the L = total length to be weld
whole plate into five equal parts. If the cutting rate is 10 in per minute, how long Will It
take to cut the plates. L = 6+4+4 = 14ft
A 19.20 min C. 21.23 min 6"
~ ~
B. 13.45 min D 1734 min
52 Mac n in« Shop Practice Mach inc Elcrncli t s 53
Total gas consumption
Total gas consumption

PROBLEM 31
(20) (14)
280 ft3

A 10ft X 20 ft plate is to be divided into 8 pieces 5 ft x 5 ft steel plate by means of an .


·
4. ...
M1ACHINE
ELEIVIEN-r!:»
oxyacetylene cutter that has a cutting rate of 10 in per minute. How long will it takes
to do the Job?
A. 50 min C. 60 min
B 70 min O. 80 min

SOLUTION

-
5"
5"
L = total length to be cut 10"

L = 20 + 10 + 10 + 10 1. Cylinders Rolling in opposite direction:

L = 50 ft 20"
. Length to be cut 50 (12) ~ Tangential speed V2
time = - - - . - - - = 60 min
Cutting rate 10 V1 = V2 = TI 01 N1 = TI 02 N2

(. Relation of diameter and speed


D1 N1 = D2 N2

Speed of Driver
to Speed Ratio == Speedof the Driven c

". Center Distance R + R


1 2
= D1 +~2
2

2. Cylinders Rolling in the same direction

4L. Tangential speed


V1 = V2 = TI D1 N1 = TI D2 N2

\. Relation of diameter and speed


01 N1 = D2 N2

Speed of Driver
(. Speed Ratio = Speed of the Driven
I
I
101( ..
l:)?=_l:)l . C I

(. Center Distance Rz - R1
2
54 Mach.irie Elements Mci ci': inc ElcITICf/( S 55
PROBLEM 1 (ME Bd. Apr. 97) PROBLEM 3
Two parallel shafts connected by pure rolling turn in the same direction having a A 5 ft diameter pulley is turning at 200 rpm. Find the speed of the pulley mid-way of
speed ratio of 2.75. What is the distance of the shafts if the smaller cylinder is 22 the center and outer surface of the pulley.
cm in diameter? A. 20.45 ft/s C. 24.34 ft/s
A. 16.67 cm C. 30.25 cm B. 30.45 fils O. 26.18 fils
B. 25.25 cm O. 19.25 cm

SOLUTION
E'iI!!ImD
v == 2 IT Im N
Using the relation of 0 and N:
0 1 N1 == O2 N 2 r == 5/2 == 2.5 ft
22(2.75) == O2(1)
O2 == 60.5 cm rm == 2.5/2 == 1.25 ft

v == 2 t: (1.25) (200/60)
For cylinders in the same direction:
C == O2 _=-P-l v == 26.18 fils
2
60.5 - 22 PROBLEM 4
C _ ..~-----

A wheel is turning at 360 rpm. If tangential speed of the wheel is 15080 ft/s, what is
2
C 19.25 cm the radius of the flywheel?
A. 8ft C. 4 ft
B. 2ft O. 10ft
PROBLEM 2 (ME Bd. Apr. 97)
A lineshaft runs at 360 rpm. An 18" pulley on the shaft is belt connected to a 12"
pulley on the countershaft. From a 15" pulley on the countershaft, motion is E'iI!!ImD
transmitted to the machine. Compute/check the required diameter of the pulley on the
machine to give a spindle speed of 660 rpm. v==ITON
A. 16" B. 121/4"
C. 101/2" O. 81/2" 150.80 == IT 0 (360/60)

SOLUTION 12" o 8ft

R 8/2 4ft
OL N L == Dc Nc

18(360) == 12N c
If center distance is
Nc == 540 rpm (speed of countershaft)

Dc Nc == OMNM Lineshaft Countershaft Machine

15(540) == OM(660)
SOLUTION
OM == 1227

Say 12 1/4" Speed ratio == ~- 3


N - -1-
2

0 1 N 1 == D 2N 2

O2 == (N,/N 2)DI '.


I C = 40 in.1"
Machine Elcmcnt s 57
5G Machine Elements
EmmiImI
D2 = DI (3/1)
DA N A = DB NB
D2 = 3 D, DA (6) = DB (4)
DA = 2/3 DB
For cylinders rolling in opposite direction:
C =~_-+:~ DB NB = De Ne
2 DB (4) = De (2)
3D + D De = 2 DB
1
1 _._-
40 -,,-
D Dc
2 CAe = - A + DB + --
D, 20 in (smaller pulley) 2 2 ~c =30in

PROBLEM 6 CAe = 2/3


----DB 2D B
+ DB + --
Three cylinders A:8:C rolling in external contact has a speed ratio of 4:2:1. If the 2 2
diameter of cylinder A is 10 ern, find the center distance between cylinder A and C. 30 = 2.33 DB
A.40cm C.45cm DB = 12857 in
8. 35 cm D. 50 ern DA = 2/3 (12.857) = 8.571 in
Dc = 2 (12.857) = 25.714 in
K-N':JmmI De Ne = Do No
25.714 (2) = Do (1)
DA NA DB NB
Do = 51.428 in
10(4) DB (2) D 2Do
CAO = - A + DB + Dc + - -
2 2
DB = 20 cm 8571 51.428
CAO = - - - + 12857 + 25714 + - - - 68.57 in
2 2
DB NB o, Nc
20 (2) o, (1)
Dc = 40 cm

DA
CAe - + DB+--
Dc
2 2
10 40
CAe -- + 20 + ---
2 2
CAe 45 cm

PROBLEM 7
Four cylinders A:8:C:D has a speed ratio of 6:4:2:1. If center distance between A and
C is 30 inches, find the center distance A and D.
A. 68.57 m C. 5634 m
8. 83.45 m D. 7634 m
58 51 resses Stresses 59
I 12. Strain (unit strain) - is the total deformation measured in the direction of the line

5
of stress.

STRESSES 13. Modulus of Elasticity - is the ratio of stress to the strain.

14. Residual Stress - internal inherent, trapped locked-up body stress that exists
within a material as a result of things other than external loading such as cold
working, heating or cooling, etching, repeated stressing and electroplating.

15. Compressive Strength - is the ability of a material to resist being crushed.

16. Bending Strength - is the quality of a material which resists forces from causing

~
a member to bend or deflect in the direction in which the load is applied.

Force . 2 2
1. Stress (S) = -----, lb/in , kg/cm , KN/m
2 ~
Area
Stress - a total resistance that a material offers to an applied load. 1. Tensile Stress (S,)

2. Ultimate stress (Su) - is the stress that would cause failure S, = Ft


A
D
3. Yield stress(Sy) - maximum stress without causing deformation For solid circular cross-section: A = 2: 0"
4
4. Allowable stress(Sail) = Ultimate stress/Factor of Safety
For hollow circular cross-section: A = ~(Oo 2 _Oj2)
5. Design stress(Sd) - stress used in determining the size of a member. 4 F

Based on ultimate stress: Based on yield stress: ~"i=--c± For rectangular cross-section: A "'base x height

where: FS = factor of safety


6. Working stressfSc) = stress actually occurring under operating condition.
2. Compressive Stress(Sc)
7. Endurance limit or fatigue limit = the maximum stress that will not cause failure F
Fe
when the force is reverse indefinitely. Sc = A
8. Elastic limit - is the maximum stress to which a standardized test specimen may For solid circular cross-section: A = ~ 02
be subjected without a permanent deformation. 4 ~D

9. Proportional Limit - is the stress at which the stress-strain curve deviates from a For hollow circular cross-section: A = ~ (0 0 2 _Oj2)
straight line. 4

For rectangular cross-section: A = base x height


10. Strength - is the ability of metal to withstand loads without breaking down.
Note: For both compressive and shearing stress, the area is perpendicular to the line
11. Shear strength - is the ability of metal to withstand forces thus following a of force.
number of twist.
Srress es 5 I r esses 61
3. Shearin g Stress( Ss}

A For single bolt of rivet needed to join to plates together.


D 5. Factor of safety( FS}
Ss F
a. Based on yield strengt h
F
where:
For single rivet: A .iil4p~ (fO~ ~ircular cross section)
For double riveted joint: A =2(rr74 p2)
F '" applied load F b. Based on ultimat e strengt h
Note: The shearin g area is always parallel
to the line of force.
II
B. Shearin g due to punchin g of hole. where:
Sail ::: allowab le stress
Su = ultimate stress
5 y '" yeild stress

(for punchin g a hole)


(for square hole)
6. Torsion al Shear Stress( Ss}
T
t~

~
16T


Where: S '" length of side of square o
t '" plate thickne ss nOs
where: T '" torque
FROM MACH INERI ES HAND BOOK p. 1924: J '" polar momen t of inertia

Pressur e needed to punch a hole. F: (focsolipShaft)


F= (fXlx8 0,tons ~£.

c '" distanc e from neutral axis to the farthest fiber


Where: d '" hole diameter, in c '" r (for circular cross section)
t '" thicknes s, in d = diamete r

7. Bendin g Stress( Sr}


FROM MACH INERI ES HAND BOOK p. 1149: F

W ::: working strength to screwe d up tightly to packed joint


+ ""J
g....--L--:a
2 Sf:= Me
W:=iSI (O.55d ;>0;250) I
where:
Where: Sj '" working stress, psi M '" momen t
d '" bolt diamete r, in c = distanc e of farthest fiber from neutral axis
F I '" momen t of inertia about the neutral axis
.. P,ojec ted "'ea I = bh
3/12
(for rectang ular cross section)
I M
4. Bearing Stress( Sb) Z := section modulu s := - : = -

~
e Sf

L 8. Strain and Elonga tion

y
D c Strain
where: A = DL 1< )j
D
62 Stresses
Stresses 63
F
(. Stress ==
A Sa=' Smax=.§min
2
Smax == maximum stress
Smm == minimum stress
L
"
(., Poisson's Ratio(u) == is the ratio of lateral unit deformation to axial unit
(. deformation.
E

where: Y == elongation due to applied load


~ where:
U= -
2G
G == shear modulus of elasticity
L == original length E == modulus of elasticity
F == force F
E == stram ==-.-
.
....
F
A == area
S == stress
AE

== By.
9. Thermal Elongation; Stresses Lateral Strain Ez
U == -----,,___ ~ ==.~

Longitudinal Strain

,I
Ex Ex

~ 1AtrJ4UJ.ij
v
Ex == L z -L1
e (t2 - tl) L
h2 q I""" L
L1
E == WZ - W1
y
w1
E == t2 - t 1
z
11

e.
-J V/2 . k-V/2
E
PROBLEM I (Apr. 1999)
where: A steel rod 30 mm & 800 mm long has an allowance elongation not to exceed 1.5
Y == elongation due to temperature difference mm, find the allowable load in kN.
k == coefficient of thermal expansion. m/m-oC A. 278 C. 316
B. 248 D. 236
t 1 == initial temperature, "C
b == final temperature, °C
4-t-l i 'i'(,]:'1
(., Relation between shearing and tensile stress based on theory of failure:
For steel:
~y Ssrnax
6
E == 30 x 10 psi 206,785,714 kpa

(., Variable Stress 1 Y _ FL


FS AE
where: F(0.800)
FS == factor of safety 0.0015 == - - -

Sy == yield point ~ (0.03)2(206,785,714)


4
Sn == endurance limit
Sm == mean stress F == 274 KN

Smax +Smin 1'[{OBLf<:M 2 (Oct. 2000)


2 Compute the induced/compressive stress, In kpa. of a steel solid shafting of 50 mm
Sa == variable component stress diameter and 800 mm in length that is subjected to an increase of temperature by
80"C.
Stresses 65
64 Stresses
A. 181,816 C. 218.182 SOLUTION
B. 242,816 D. 196,560
Z == Polar section modulus
SOLUTION
Z == TIS
Where:
For steel:
T == torque
6 S == stress
E == 30 X 10 psi == 206,785,714 kpa
6
G == 12 x 10 psi == 82,714,286 kpa
6 16 T
a == thermal elongation == 11.7 x 10- m/mDC S
IT d3
Using thermal elongation formula: Simplify:
S == K E (t2 - t1) 3
IT d _ IT (3)3 == 5.30 in3
TIS
S == 11.7x10
6(206,785,714)(80)
193,551.43 Kpa
16 - 16

The answer is: D (nearest) PROBLEM 5 (Apr. 1999)


Compute the allowable load in kN on a 20 mm x 120 cm long steel rod with a
PROBLEM 3 (Oct. 2000) maximum elongation must not exceed 1 mrn.
Compute for the load in KN on a 3 cm diameter, 100 cm long steel rod if its maximum A. 35 C. 66
elongation exceed 0.12 cm. B. 55 D 76
A. 178 C. 148
B. 196 D. 287 ~
Em!!ImD For steel:

For steel: E == 30x10 6psi == 206,785,714kpa


6
E == 30 x 10 psi 206,785,714 kpa y == ~

FL AE
y == - -
AE F (1
0001 ==
0.0012 == -~
F (1.0)
.._ - - - - - - -J (0020)2(206,785,714)
IT (0.03)2 (206,785,714)
4 F == 54.1364 KN

F == 175.40 KN PROBLEM 6 (Oct. 2000)


Determine the load in kw on a 25 mm diameter x 1200 mm long steel shaft if its
The answer is: A (nearest)
maximum elongation exceeds 1 mm.
A. 83 C. 103
B. 125 D 234
PROBLEM 4 (Oct. 2000)
Compute for the polar section modulus of a SAE 1060 shafting having a diameter of 3
inches. Use a factor of safety of 2 and design stress at 800 psi. Ei-l"imD
A. 4.7 C. 4.2
B. 6.1 D. 5.3 For steel:

E == 30 X 10 6 psi zz: 206785,714 kpa


St resse's 67
bb Stress es

Y = _.-
FL ast·m1D(.]tI

AE

Sn 28,000

F(120) . S
0.001 = . - - . - . - - . FS 3

IT (0.025)2(206,785,714)

4
S == 9,333.3 3 PSI
F = 84.59 KN

PROB LEM 10 (ME Bd. Oct. 97)


to fix a lever to a shaft,
PROB LEM 7 (ME Bd. Oct. 97) Determ ine the minimu m mean diamete r of a taper pin for use
in-lbs. The shaft diamete r is 2 inches and
The total weight of steel plates 3/4" x 3' x 20' is: if it is transmi t a maximu m torque of 700
factor of safety of 2.
A 1838 Ibs C. 3676 Ibs the materia l allowab le stresse s is 15,000 psi. Use
B. 1698 Ibs D. 1848 Ibs A. 7.0 mm C. 5.2 mm
B. 7.2 mm D. 6.2 mm

Em!!mm 3/4"
3'
3
DI!!iIir+D
Using density of steel == 0.284 Ib/in
T
F
Weight = Volume x density r
Weight = [(3/4)(3 x 12)(20 x 12)J(0.284) 700
Weight = 1840.32 Ibs F 700lbs
1

PROB LEM 8 (ME Bd. Oct. 97) s, == ~


r thru a 1/4 in steel
What pressur e is required for punchin g a hole 2 inches in diamete A
plate? 700
A.10to ns C. 20 tons 15,000 = .. -~
(IT/4)d 2
B 30 tons D. 40 tons

d = 0.2437 in = 6.19 mm

SOLU TION
PROB LEM 1 1 (ME Bd. Apr. 97)
a steel plate made of SAE
How many 5/16 inch holes can be punch in one motion in
55 tons. The ultimate strength for shear is
1010 steel, 7/16 Inch thick using a force of
50 ksi and use 2 factor of safety.
From Machin eries Handbo ok, 1924: C. 5
A. 58
B 37 D 62
Pressur e == D x t x 80, tons

Where: D = hole diamete r ~

t == plate thickne ss

Solving for the equival ent diamete r for a force of 55 tons.

1/4"
Pressur e = 2 x 1/4 x 80 == 40 tons
F
~ Ss
2" A
PROB LEM 9 (ME Bd. Oct. 97) F
enduran ce strength is Ss
A link has a load factor of 0.8, the surface is 092 and the rtdt

ing stress of the link if it is subject ed to a reversin g


28,000 psi. Compu te the alternat
load. Assume a factor of safety of 3. 10 3 = (55 x 2000)


50 X
A. 8150 C. 9333 rrd(7/1 6)
B 10.920 D. 7260 I¢'-H
d == 16 In
68 Stresses
Solving for the equivalent no. of holes Stresses 69

No. of holes = _~Ji~ = 5.12


PROBLEM 14 (ME Bd. Apr. 96)

(5/16)
A steel tie rod on bridge must be made withstand a pull of 5000 Ibs. Find the diameter

of the rod assuming a factor of safety of 5 and ultimate stress of 64.000 psi.

Therefore no. of holes is 5 because the punching machine cannot punch entirely the
A 0.75 C. 0.71
remaining 0.12 in.
B. 0.84 D. 0.79

SOLUTION
PROBLEM 12 (ME Bd. Apr. 97)

The shaft whose torque vanes from 2000 to 6000 in-lbs has 1 1/2 inches in diameter

SI = F/A

and 60,000 psi yield strength. Compute for the shaft mean average stress.

A. 6036 psi C 5162 psi Su

If factor of safety is given: St


B. 6810 psi D. 5550 psi FS
Where: FS = factor of safety D
SOLUTION
_§'l = ~
FS A

Using shaft stress formula for pure torsion: 64,000 5000

Smax ~6!max
3
=~~()OO) = 9054 psi 5 J1:d2 F = 5000 Ibs
nd n(1.5)3 4

16Tmin 16(2000) d = 0.71 in

SOlin 3018 psi


nd3 = - n(1sT
PROBLEM 15 (ME Bd. Apr. 96)

If the weight of 6" diameter by 48" long SAE 1030 shafting is 174.5 kg, then what will

Using variable stress formula equation: be the weight of chromium SAE 51416 of same size?

S mean -- Smax + Smin __ 9054 + 3018 A. 305.5 Ibs C 4264 Ibs


--~---- --,~-~_._
6036 psi
2 2 B. 384.61bs D. 465.1 Ibs

PROBLEM 13 (ME Bd. Apr. 96) EE!l!ii tel II


A journal bearing with diameter of 76.2 mm is subjected to a load of 4900 N while The major component of different steels is iron, therefore their densities do not differ
rotating at 200 rpm. If its coefficient of friction is 0.02 and UD = 2.5, find its projected much.
. 2
area In mm . Ib
Weight 174.5 kg (2000 ---)

A. 12,090 B. 14,165 kg

C. 13,050 D. 14,516 Weight 38481bs

4900 N
SOLUTION
I
., Projected A rea
PROBLEM 16 (ME Bd. Apr. 96)

If the ultimate shear strength of a steel plates is 42,000 psi, what

force is necessary to punch a 0.75 inch diameter hole in a 0.625

~ = 2.5

~~L
inch thick plate?

o A 63,0081bs C. 68,080 Ibs


Substitute the diameter:
B. 61,800 Ibs D. 66,8001bs
L

- - - = 2.5
76.2

L = 190.5 mm

EmmmmI
76.2 mm
A = projected area is rectangular
SOLUTION: 0.625"
A = 0 x L
8
A = 76.2(190.5) = 14,516 mm 2
Force -r­ Stress x Area ~O.75"
r.;~,
:!~;
Where: Area = Circumference x thickness
~~:
..~
70 SI resse» 71
Stress es
Area == n D t

Force == 42,000 [n: x 0.75 x 0.625J

";ON""'.
Solving for diamete r of bushing hole after heating:

y == kL(t2 - t.) == 0.00000 68(1.99 9)(24.5 ) == 0.00033 3"

Force == 61,850 Ibs


D == 1.999 + 0.00033 3 == 199933 3"

Solving for diamete r of shaft after cooling:


PROB LEM 17 (ME Bd. Oct. 95)
y == -0.0007 3(2) == -0.0014 6"

What is the modulu s of elasticity if the stress is 4,000


psi and unit a strain of d ==
2 - 0.00146 == 1.99854 0"
0.00105 ?
6
Clearan ce == 1.99933 3 - 1.99854 0 == 0.00079 3"
A. 41 .905 x 10 6
B. 42.300 X 10
C. 41.202 X 10 6 D. 43.101 X 10 6 PROB LEM 20 (ME Bd. Oct. 95)
What modulu s of elasticity in tension is required to obtain
SOLUT ION
a unit deforma tion of
0.00105 m/m from a load produci ng a unit tensile stress of 44,000
6
psi?
A. 42.300 x 106 psi C 41.202 x 10 psi

E == Stress/S train 6 6

B. 43.101 x 10 psi D. 41.905 x 10 psi


E == 44,000/ 0.00105 == 41.905 x10 6psi

EmmmD
PROB LEM 18 (ME Bd. Oct. 95) E == Stress/S train == 44,000/ 0.00105 == 41.905 x 10 6 psi
A 2.5 in diamete r by 2 in long journal bearing is to carry a
5500-lb load at 3600 rpm
using SAE 40 lube oil at 200°F through a single hole at
25 psi. Comput e the PROB LEM 21 (ME Bd. Oct. 95)
bearing pressure. What force P is required to punch a 1/2 in. hole on a 3/8 in.
A. 1100ps i
thick plate it the ultimate
C. 900 pSI shear strength of the plate is 42,000 psi?
B. 1000 psi D. 950 psi A. 24.9401 bs C. 24,6201bs
5500lb s
SOLU TION I
..
Project ed A
rea
B. 24.9601 bs D. 24,7401bs

EmmmD
Bearing pressur e is the same as bearing stress.
~/
b . •2"
P == Stress x Area

F 2.5"

s, == LD P == Ssu X (x d t) == 42,000(n: )(1/2)(3/8) == 24,7401 bs


5500 I< >[
Sb == == 1100 psi
2(2.5) .5 in i
PROB LEM 22 (ME Bd. Apr. 95)
What pressur e is required to punch a hole 2" diamete r through
a 1/4" steel plate?
A 10 tons C. 20 tons
PROB LEM 19 (ME Bd. Oct. 95) B. 30 tons D. 40 tons
It is a problem of expans ion and shrinka ge of steel materia l
so that the slightly bigger
shafting of 2" diamete r can be inserted/fitted to the slightly
smaller hole of a steel SOLU TION
bushing of 1.999" diamete r with the following process /materia l/data to apply:
Coefficient of expans ion of carbon steel == 0.00000 68"I""F tempera
ture raised by gas From Machin ery's Handbo ok p. 1924:
heating == 24.5°F. Cooling media to use dry ice with boiling point
of -109.3° F (-78SC )
Shrinka ge rate below boiling point is 0.00073 in/in Determ
ine the final clearan ce P == d x t x 80 tons == 2 x 1/4 x 80 == 40 tons
betwee n the expand ed steel bushing hole against the shrinka ge of the steel
shaft.
A. 000079 3"
1'!{OH LEM 23 (ME Bd. Apr. 95)
C. 0.00075 0" Comput e the working strength of 1" bolt which is screwed
B. 0.00069 3" D. 0.00080 0" up tightly in packed joint
when the allowab le working stress is 13.000 pSI
72
Stress es Stress es 73
A. 3600 psi C. 3950lb s PROB LEM 26 (ME Bd. Apr. 94)
B. 3900lb s D. 3800lb s Determine the estimated weight of an A-36 steel plates size 3/16
1 in x 6' x 20'
A.9191 bs C.1012 1bs
SOLU TION
F B. 8291bs D. 7351bs

From Machinery's Handbo ok p 1149: F


SOLU TION

W = working strength to screwed up tightly to packed joint

W = St (0.55d 2 - 0.25d) From Faires p. 574,

Where: St = working stress, psi

d = bolt diameter, in Density of steel = 0.284 lb/in"

W = 13,000 [0.55(1 l- 0.25(1)] = 3900lb s Weight = Volume x Density 6'

Weight = (3/16)(6 x 12)(20 x 12)(0.284) = 9201bs.

PROB LEM 24 (ME Bd. Apr. 95)

What is the working strength of a 2" bolt which is screwed PROB LEM 27

up tightly in a packed A 19 mm stud bolts is used to fastened on a 250 mrn diamete


jointly when the allowable working stress 12,000 psi? r cylinder head of diesel
engine. If there are 10 stud bolts, determine the pressure inside
A. 20,1201bs C. 20,1001bs the cylinder if bolt
B. 20,4001bs D. 20,2001bs stress is limited to 50 Mpa.
A. 288.8 Kpa C. 3426 Kpa '
B. 2888 Kpa D. 4828 Kpa

SOLU TION

20mm

From Machin ery's Handbo ok p 1149: F E:1!!ImD


W = St (0.55d 2
- 0.25d) F Fb = force applied per bolt

W = 12,000 [0.55(2 l- 0.25(2)] = 20,400 s, = Fb / A

Ibs A = cross-sectional area of bolt

50,000 =. Fb

PROB LEM 25 (ME Bd. Apr. 94) (rcl4)(0 .019)2

Determine the estimated weight of an A-36 steel plate size W' Fb = 14.176
x 4' x 8'.
A. 280 kgs C. 332 kgs
B. 301 kgs D. 297 kgs F = total force applied on cylinde r head
250mm
F = 10(14.176) = 141. 76 KN

SOLU TION Pressure = F/A

A = area of cylinde r

141.76
Pressure = 2 = 288.8 Kpa

From Faires p. 574, (n/4)(0 .25)

density of steel = 0.284 Ib/in3


Weight = volume x density PROB LEM 28
1/2(4 x 12)(8 x 12)(0.284) A column supports a compressive load of 250 KN. Determine
1~"1 the outside diamete r of
Weight = 654.336 Ibs = 297 kg V column if inside diamete r is 185 mm and compressive stress of
50 Mpa.
4' A. 200.62 mm C. 216.42 mm
B. 201.47 mm D. 208.41 mm

L
s: resses 5tn'ss(·..., 75

SOLUTION Im!!Im3
F = 250 KN
20mm
S 0= F/A S = F
A
50000 0= 250 F F
, A S = -------
2 F
A 0= 5 X
3
10- m2 (7t/4)0
2) F
A (rr/4 0 0 - (rr/4 0;2)
0=
2 2 80,000 = ----
A 0= rr/4(00 - 0, ) (7t/4)(0.02)2

Substitute: F = 25.132 KN
5 x 10- 0= rr/4 (0 02 - (0. 185)2J
3
PROBLEM 31
6.366 X 10- 0= 0 02 - 0.034225
3
What force is necessary to punch a 30 mm hole in 12.5 mm thick plate if ultimate
Do 0= 0.201472 m 0= 201.47 mm
shear stress is 410 Mpa?
A. 564 KN C. 483 KN
PROBLEM 29 O. 983 KN
B. 342 KN
A steel hal/ow tube is use to carry a tensile load of 500 KN at a stress of 140 Mpa. If
outside diameter is 10 times the tube thickness, find the thickness of the tube. Im!!Im3
A. 11.24 mm c. 20.64 mm Using shearing stress formula:
B. 107 mm O. 22.61 mm
F
S
SOLUTION A
F
S
S 0= £: ndt
A F
500 410,000 = 7t(0.03)(0.0125)
140,000
A t
~
A 0= 3.57143 X 10- 3 m 2 F 0= 483.0198 KN

Do 0= 10t PROBLEM 32
Do 0= OJ + 2t Two 30 mm thick plate is fastened by two bolts, 25 mm in diameter. If the plates is
10t 0= D, + 2t subjected to 50 KN tension, find the bearing stress in bolts.
0; 0= 8t A. 33,333.33 Kpa C. 5555.55 Kpa
B. 4444.44 Kpa O. 555,555 Kpa
A 0= rr/4 (0 02 _ 0,2)
3
3.57143 x 10- 0= 7t/4((10t)2 - (8t)2J 25mm
4.5473 x 10-3 0= 36e SOLUTION
t 0= 0.112389 m 0= 11.24 mm
50 KN
+- 50 KN
PROBLEM 30 Using bearing stress formula: ---l
A 20 mm diameter rivet is used to fastened two 25 mm thick plate. If the shearing S = £.
stress is 80 Mpa. What tensile force applied each plate to shear the bolt? A
A. 26.35 KN C. 30.41 KN A = area = 2 d t ( for two bolts)
B. 28.42 KN O. 25.13 KN
Stress es

S ==£ Stress es 77
dt
PROB LEM 35
S == 50 What is the modulu s of elasticit y if stress is 300 Mpa and strain
of 0.00138 ?
2(0.025 )(0.03) == 33,333. 33 Kpa A. 217.39 x 103 Mpa C. 220.61 Mpa
3
B. 200.61 x 10 Mpa D. 215.21 Mpa

PROB LEM 33 SOLU TION


A 2.5 inches shaft is subjecte d to 3 KN-m torque. Find the stress
develop ed.
A. 48.62 Mpa C. 59.68 Mpa . Stress
B. 52.75 Mpa D. 38.64 Mpa Modulu s of Elasticit y, (E) = --.--
Strain
300
E = == 217.39 x 10 3 Mpa
SOLUT ION 0.00138

d == 2.5 in x 1/12x1 /3.281 == 0.0634 9m PROB LEM 36


In a 2.0 m cantilev ered I-beam, a 2 Mton weight is applied at
free end. If the allowab le
Using shaft stress formula : stress in beam is 110 Mpa, determi ne the section modulu s.
A. 18.54 in 3 3
16 T C. 26.83 in
S
TC d3
T =3 KN.m B. 21.77 in 3 D. 24.28 in
3

S == 16 (3) SOLU TION


TC (0.0634 9)3 p= 2 Mtan

S == 59,680. 68 Kpa == 59.68 Mpa For a cantilev er beam with load at free end:
M == F xL
M == (2 x 1000 x 0.00981 )(2) = 39.24 KN-m
~ t
PROB LEM 34
A shaft when subject ed to pure torsion develop ed a stress of 50 Z == Section modulu s
Mpa. If polar momen t Z == IIc == MIS == 39.24/1 10,000 == 3.5673 4 3
7 4
of inertia is 6.1359 x 10. m determi ne the maximu m torque the shaft could handle.
x 10. m x (3937)3
, 3
A. 1.23 KN-m Z == 21.77 in
C. 1.84 KN-m
B. 1.68 KN-m D. 2.48 KN-m

IEm!ImD PROB LEM 37


A 5 KN force acting at the end of a 3 m cantilever beam. If section
modulu s of the
beam is 10 ;n 3, what is the stress induced ?
J == polar momen t of inertia A. 86,285 Kpa C. 29,684 Kpa
4
TC d B. 78,318 Kpa D. 91,535 Kpa
J == - -
32

6.1359 xlO. 3 == TCd


4
~ P=5 KN

~
32
d == 0.05 m For a cantilev er beam with load at free end:
M == maximu m momen t = F x L
M == 5(3) == 15 KN-m
t
S= 16T
3 Z == Section modulu s == lie
TC d Z == 10 in 3x 1/(39.37 )3 = 1.6387 x10·4m 3
16 T M 15
50,000 == S = flexural stress == - = 4
91,535. 62 Kpa
TC (0.05)3 Z 1.6387x1 0-
T == 1.227 KN-m
78 Stresses Stresses 79
PROBLEM 38
A 6 mm steel wire is 5m long and stretches 8 mm under a given load. If modulus of
Em!!IiImI
elasticity is 200 Gpa, find the load applied.
A.7KN C.9KN Y = ~
B. 8 KN D. 10 KN AE
2
A == 0.5 ern" x 1/(100)2 == 5 X 10'5 m 0.5 cm
2

SOLUTION
0.001 == (225xO.00981)(4) 4m
(5 x 10-5 )E
Y == FL 6mm E == 176580000 Kpa == 176.58 Gpa
AE
5m
0.008 == F(5)
[(nI4)(0.006)2](200x10 6 )
i mm

F == 9.047 KN PROBLEM 41
A 20 m rod is stretches to a strain of 0.001. Determine the deflection of the rod.
PROBLEM 39 A. 20 mm C. 30 mm
B. 25 mm D. 35 mm
A steel wire 10m long, hanging vertically supports a tensile 'f' F
load of 2 KN. Neglecting the weight of wire, determine the required diameter if the
SOLUTION
stress is not to exceed 140 Mpa and the total elongation is not to exceed 5 mm.
Assume E == 200 Gpa.
A. 2 mm C. 4 mm Strain == Y/L
B.3mm D.5mm
0.001 == Y/20
Im!!ImD Y == 0.020 m == 20 mm

. FL D
PROBLEM 42
Y == elongation == - A rail having a coefficient of linear expansion of 11.6 x 10'6 m/m-oC increases its
AE length when heated from 70°F to 113°F. Determine the strain.
0.005 == ~ 2(10) A. 2.04 x 10'4 C. 4.21 X 10'4
A(200x10 6 ) B. 6.05 X 10'4 D. 2.77 X 10'4
A == 2 X 10'5 m2
2
A == n/4 d
2 x'lO'5 ==n/4 d2 Em!!IiImI
d == 0.00504 m == 5.04 mm Converting to DC difference:
L1Clt.F == 519
t.C/(113-70) == 519
PROBLEM 40
L1C == 23.889°C
An iron rod 4 m long and 0.5 ern" in cross section stretches 1 mm when a mass of Strain == S/E == K (t2 - t.) == 11.6 X 10'6 (23.889) == 2.77 x 10'4
225 kg is hang on it. Compute the modulus of elasticity of the iron.
A. 176.58 Gpa C. 160.41 Gpa
PROBLEM 43
B. 169.81 Gpa D. 180.26 Gpa What temperature will the rails just touch if steel railroad is 10m long are laid with
clearance of 3 mm at initial temperature of 15°C? Use k == 11.7 X 10'6 m/m-oC.
A. 35.64°C C. 45.64°c
B. 40.64°C D. 50. 64°C
80 Stresses Stresses 81

E5t!!ImD Sf = 15,000 psi

PROBLEM 46

A1A'Wb
Y=kL(b-t1) 2 2
For a given material, the modulus of elasticity is 100 GN/m in tension and 40 GN/m
Clearance = Y/2 + Y/2 in shear. Find the Poisson's ratio.
Clearance = Y = 0.003 A. 0.10 C. 0.25

6
J V/2 k-V/2 B. 0.50 D. 0.35
0.003 = 11.7 x 10. (10)(b - 15)
t2 = 40.64°C U·)!'mmI
PROBLEM 44 G=_E_
2(1 + u)
For a given material, the modulus of elasticity is 15,000,000 PSI In tension and
E
6,000,000 psi in shear. Determine the Poisson's ratio for this material. u = --1
A. Q4 C. 0.3 2G
B. 0.25 D. 0.5 100
u = ---1
2(40)
SOLUTION
u = 0.25

Using Poisson's ratio formula: PROBLEM 47


E A 25 mm shaft is keyed to a 300 mm diameter pulley and transmits 3 kw of power.
G = ---
2(1 + u) The keyed assembly rotates at 1725 rpm. What is the tangential force at the key?
A. 1.33 KN C. 2.33 KN
6 000 000 = 15,000,000 B. 5.23 KN D. 8.34 KN
" 2(1 + u)
2(1 + u) = 2.5
u = 0.25 SOLUTION

PROBLEM 45 P=2rcTN
A 1000 lb force acts at the end of a 10ft cantilever beam. The section modulus for 3 = 2 rt T (1725/60)
3
the steel beam is 8.0 in . How much stress is induced? T = 0.01661 KN-m
A. 10,000 psi C.12,000psi
300mm
B.15,000psi D.14,200psi But: T = F x r
0.01661 = F (0.025/2)
SOLUTION

For a cantilever beam:


~
10ft
P'7 01
'
F = 1.33 KN

PROBLEM 48
What force is necessary to punch a 1 in hole in a 118 in steel plate if the ultimate
shearing stress is 60,000 psi and the ultimate compressive stress is 80,000 psi?
M = F x L = 1000(10) = 10,000 ft-Ib
M = 120,000 in-Ib
A. 34,520.34 Ib C. 12,344.56 Ib
B. 8,345.56 Ib D. 23,561.90 Ib
z = section modulus
z= -
M E:r!!iIm
Sf
Solving for the shearing area during punching:
3 = 120,000
Sf
82 Stresses Stresses 83
2
A = re d t = re (1) (1/8) = 0.3926 in PROBLEM 51
S= ~ A hollow rivet has an outside diameter of 5 mm and an inside diameter of 3 mm. If
6 2
A the allowable shearing stress is 400 x 10 N/m , what maximum shearinq force can
the rivet sustain if subjected to double shear?
60,000= _F_
0.3926 A. 10 KN C. 15 KN
B. 20 KN O. 25 KN

e
F = 23,562 Ibs

EmDImII

2
A=2 [%(0 -0;2)]
k 5 mm )1

PROBLEM 49

~D= 1 in A= 2
4
0

!: [(0.005)2 -
5
(0.003)2]
2
F F

A = 2.51327 X 10- m
A U-bolt supports a load of 6000 lb. The cross-section of the bolt has a diameter of
1/2 inch. How much stress is induced in the sides of the bolt? 6 2 3 2
F = 400 X 10 N/m = 400 x 10 KN/m
A. 12,734.45 psi C. 13,345.56 psi
B. 14,567.23 psi O. 15,278.12 psi
S = F/A F
SOLUTION 3- -5
400 x 10 - 2.51327x10

1/2" F = 10.05 KN
A = 2[ ~ (1 I 2)2] = 0.3927 in
2
PROBLEM 52
A vertical load of 400 Newtons acts at the end of a horizontal rectangular cantilever
2
beam 2m long and 25 mm wide. If the allowable bending stress is 130 MN/m , find
the depth of the beam in mm.
S= ~ = 6,000/0.3927 = 15,278.84 psi
6000lb
A. 23.44 mm C. 30.23 rnrn
A B. 38.43 mm O. 43.23 mm

PROBLEM 50
A 1 in diameter shaft has a 2 in diameter collar resting on a support. The axial load
SOLUTION ~ tP=400 N
on the shaft is 10,000 Ib and the thickness of the collar is 1/2 in. How much shearing 2m
stress is induced?
A. 6,366.20 psi C. 5,244.56 psi Moment = 400(2) = 800 N.m = 0.80 KN.m
B. 4,234.56 psi O. 8,456.34 psi F =
10,000 Ibs
z = MIS = [0.80/130 x 103] (1000)3 h
SOLUTION Z = 6153.85 rnrn" 25mm
;~ SHAFT· 1" dia. bh 2
z=-
F 6
COLLAR· 2" dia.
Ss = reds t
6153.85 = (25)~
1/2" 6
Ss = 10,000 h = 38.43 mm
re(1)(0.5)
.1/2"
S, = 6,366.20 psi
1"
84 Stresses
PROBLEM 53 Stresses 85
A simply supported timber beam is 50 mm by 200 mm in cross-section and 4 m long. F
If the fiber stress is not to exceed 8.3 Mpa and the beam weight is neglected, find the 210000 = - -
, 0.00144
maximum mid-span concentrated load that the beam can support if the 200 mm
F = 302.4 KN = 302,400 N
dimension is vertically oriented.
A. 2,766.67 N C. 2,345.56 N
B. 3,233.45 N D. 1,234.45 N
PROBLEM 55
A steel hanger of 2 m length is to carry an axial load of 18 KN. If the tensile stress is
SOLUTION restricted to 104 Mpa and the elongation caused by this load is restricted to 1 mm,
what is the minimum cross-sectional area the member can have? E = 200,000 Mpa
~p A.1Mm~ C.1~m~2
B. 173 rnrn" D. 190 mm
For rectangular beam:
Z = section modulus of rectangular beam SOLUTION
2
bh 200mm
Z= - -
6 Based on tensile stress: 2m

Z
(50)(200)2
= -'---'--'----'-- SOmm
S = ~
A
6
4 3 104 = 18,000
z = 333,333.33 rnrn" = 3.33 x 10. m
A
2
M=Sz A '" 173 mm
6)(3.333 4
Moo (8.3 x 10 10. ) = 2,766.67 N.m
F = 18 KN
For simply supported beam: Based on the given deflection:
M= ~L-. FL
4 y
276667 = P(4)
AE
,. 4
18,000 (2,000)
P = 2,766.67 N
1 = A (200,000)
A = 180 rnrn"
PROBLEM 54 Therefore use the higher area: A = 180 rnrn"
A lap joint consists of steel plate 250 mm by 18 mm in thickness is connected by 4-20
mm diameter rivets. Compute the bearing capacity of the rivet connection if the
allowable bearing stress is 210 Mpa. PROBLEM 56
A. 302,400 N C. 420,340 N A short hollow steel cylinder with a wall thickness of 38 mm is to carry a compression
B. 360,200 N D. 460,240 N load, applied uniformly on the end, of 7,800 KN. If the allowable working stress in
steel in compression is 138 Mpa, then the minimum outside diameter of the cylinder
SOLUTION required to safely support the load is:
A. 512 mm C. 396 mm
F B. 482 mm D. 460 mm
Sb = -
A
A = bearing area
A = 4(dxt) = 4 (0.020 x 0.018)
F Em!!imD
2
A = 0.00144 m
s = F
A
86 Stresses Stresses 87
7,800,000
138 SOLUTION
A
A = 56,521.74 rnrn" S = KE (t2 - t1)
A=Ao-A,
6
A = ~ D 2 _ ~ D1 2 = ~ [D 2 ~ D 2J S = (6.5 x 10 6 inl'F) (30 x 10 psi)( 130) = 25,350 psi
4
4° 4 01

Dj = Do- 2t PROBLEM 59
D j = DO - 2(38) = Do - 76 A steel rod with a cross-sectional area of 160 rnrn" is to be attached between two
TC 2 2 fixed points 1.25 m apart. If the rod is too short by 0.25 mm, find the stress applied to
56,521.74 = -[Do -(D o-76) J put the rod back to fitness. E = 200,000 Mpa
4
2 2 38mm~~38mm A. 40 Mpa C. 50 Mpa
79,965.71 = D0 - D0 + 152D o - 5,776
Do = 511.5 mm B. 30 Mpa D. 60 Mpa
~ )1
k Do
PROBLEM 57 SOLUTION

A 25 mm diameter steel bar is loaded in double shear until failure, the ultimate load is

.( J
found to be 446 KN. If the allowable stress is to be based on a factor of safety 3 what L = 1.25 - 0.00025 = 1.24975 m
must be the diameter of a pin designed for an allowable load of 26 KN in single shear
be? A. 14.79 mm C. 13.24 mm y = S~

E
B. 15.64 mm D. 12.63 mm
0.00025 = S ( 1.24975 )
SOLUTION 200,000,000 1.25 m
S = 40,008 Kpa = 40.008 Mpa
Fu
Su = -
A
For double shear:
446,000
Su = 454.52 Mpa
2[~(25)2]
.4

Sail Su _ 454.52 = 151.51 Mpa


FS - 3
F
Sail
A
151.51 = 26,000
~d2
4
d=14.79mm

PROBLEM 58
A steel bar, initially free of stress, is held between rigid supports. Determine the
stress in the bar if temperature drops 130°F. K = 6.5 x 10'6 inl'F, E = 30 x 106 psi
A. 25,350 psi C. 24896 psi
B. 26,234 psi D. 23,654 psi
88 Shafting 89
Shafting
e Shaft power

6 Sha~t A. Power Formula in 81 unit:

where:
TN T

P == power, KW
T = torque, KN-m
N == speed, rps

B. Power in English

mmmmm! where:
P == power, Hp
1. Shaft - is a rotating member that is used to transmit power.
T == torque, ft-lb
N == speed, rpm
2. Axle - a stationary member carrying rotating wheels, pulleys. F
(; Torque, T R
3. Machine shaft - shaft which is an integral part of the machine.

4. Line shaft - transmission shaft driven by prime mover. where:


F == applied force
R == radius = 0/2 Shaft
5. Counter shaft - transmission shaft intermediate between the line shaft and
the driven machine.
" Stresses in shaft when sUbjected to pure torsion(Ss)
T
6. Spindles - is a short axles and shafts.

7. Transmission shaft - is a shaft used to transmit power between the source and
.&EiiU&i:Q:a:UiUU,W
the machines absorbing the power, and include countershafts, line shafts, head
shafts, and all factory shafting.
A. For solid shaft:
~ OTO R Machine Shaft

W • ,,~~1% %4~

I'
I,
Main Shaft
0

~1,
Counter Shaft
T]

B. For hollow shaft: •


-JI.
.. ~
~,~
...

where:
S5

tOlE t
Driven Machine 1 ~ Do == outside diameter
0, == inside diameter
t
Driven MaChine 2
90 Shafting Shafting 91
J = polar moment of inertia b. For hollow shaft:
c = is the distance of the farthest fiber from neutral axis
T = torque
d = diameter of shaft
" Torsional deflection(8), rad

where:
• Strength of shaft with assumed allowable stresses
L = length
of shaft
J = polar moment of inertia (PSME Code p. 18)

a. For Main Shafts:

b. For Line Shafts:

G = modulus of rigidity in shear = 11.5 x 10 6


psi for steel
c. For short shafts:
FROM MACHINERIES HANDBOOK:
where:
For solid shaft:
P = power, Hp
N = speed, rpm
o = diameter, inch
For hollow shaft:


iii Stress in shaft when subjected to Torsion and Bending
(From Machineries Handbook)
loads:
• Shaft diameter for 0.08 degrees per foot of length of shaft
F
a. For solid shaft: deflection.
~
'a.iIJJ2i1taa!a~il'
T

Where:
where:
T = torque o = diameter, in
M = moment T = torque, in-Ib
5 s = maximum shear stress P = horsepower
51 = maximum tensile or compressive stress N = speed, rpm
92 Sh (~(t ino
Shu!tin.t} !cJ:1
For Sl units:
2. Rectangular
D = 2.26 tIT 'or D = 125.70 ~.
I~d
bd(b 2 +d2 )
J = 12
Where:
o :::: diameter, mm 2
bd I. b tol
P = power, watts
N = speed, rpm 3 +1.6 <l.
T = torque, N-mm d
Where:
• Shaft deflection of 1 degree for a length of 20 times its b -= bose
d = height (shortest side)
diameter.

~ 3. Circle

}
D=0.10 Wf·. orD=4.0
4 3
Where" ltD Zp = ltD
J= -
32 16
P = power, hp Where: 0 -::= diameter
D = diameter, in
N =speed, rpm 4. Hollow tube
T~ torque, in-Ib

• Linear deflection of shafting J = -"-(D 4_d4 )


32
.
3. Shafting subjected to no bending action of pulleys except its own weight
L =6.95 frJ2
b. Shafting subjected to bending action of pulleys, etc D =
Z =
p

outside diameter
-"-r
16
D4 4
-d
D
\. J r
L=5.2 ~ d = Inner diameter

Where: L = shaft length, ft


5. Hexagonal
o
= shaft diameter, In

J = 1.0825 s' = 0.12F'


5 F 5
POLAR MOMENT OF INERTIA & POLAR SECTION MODULUS Zp = 0.20F'
J :::: polar moment of inertia S = length 01 side
Zp - polar section modulus

1. Square
6. Triangle

~Io
4
J = 0.1667a
z, = 0.206 a' J = 0.036 s'
Where:
a = length of side
Z, = 0.05 s'

S length of side

s
n4 ShuJll nq 95
ShaJting
5 5
PROBLEM I (Oct. 2000) A 0.75x10 C. 1.20 X 10
5
Compute for the ~wisling moment in m-fb developed when the shaft delivers 20 hp at B.1.0X10 D. 2.10 X 10"
1200 rpm.
A. 1166 C. 1050 &!!I1t_gl
B. 915 O. 945
P ~ 2rrTN
E!!Dl':D
20 ~ 2 IT (T) (200/60)
P ~ 2rrTN
~ ~
6
T 0.9549 KN-m (1000)(1000) 0.9549 x 10 Nmm
33,000
20 ~ 2rrT(1200) The answer is: B (nearest)
33,000
T ~ 87.53 ft-Ib (12) ~ 1050.42In-lb PROBLEM 5 (Ocl. 2000)
, A 102 mm diameter shaft is driven at 3000 rpm by a 300 hp pnme mover. The shalt
PROBLEM 2 (Oct. 2000) , drives a 121.9 cm diameter chain sprocket having 85% output efficiency. Compute
Find the torsional deflection, in degrees, of a solid steel shaft, 110 mm 00 x 1.4 m the torque In in-Ib developed in the shalt.
fJ
long subject to 3.1 x 10 N-mm torque. The torsional modulus of elasncrty is 80,000 A. 5.600 C 8,150
N/mm-mm. B. 7.100 D. 6.300
A. 0.221 C. 0.28
B. 0.195 D 0.21
mm:mmI
D!!D1':D p c .2.itT~
33.000
300 ~ .2.rrTJ3000)
6)(1400)
8 ~ ..:r L (3.1x10
-- - - 4- . _ - - - 0.003774 rad (180
e/rr)
JG 33,000
[rr(~~o) ](80.000) T c 525.211 h-lb (12) ~ 6,302.53 in-lb
9 ~ 0.216 degree

PROBLEM:> (Oct. 2000) PIWBLEM 6 (Oct. 1998)


A 3 in diameter short shaft carrying pulleys close to the bearings transmit how much
Compute for the torsional deflection in degrees. of a 110 mm diameter, 1.4 m long
horsepower if the shaft makes 280 rpm?
shaft subjected to a twist moment of 3 x 1o" N.mm The torsional modulus of A 991 hp C. 199 hp
elasticity IS 83000 N/mm'.
B. 919 hp D. 260 hp
A. 0.27 C. 0.20
B. 031 D. 024
Em:!!JmD
Em!!ImD For short shaft:

TL (3 6
x 10 )(1400) P ~
D3N
-
Q)3J.280) 19895 hp
e/rr)
8 0.00352 rad (180 38 38
JG ~ [rr (~~0)4](83.000)
PI,OBLEM 7 (Oct. 1998)
8 ~ 0.202 degree What power would a spindle 55 mm In diameter transmit at 480 rpm stress allowed
2
lor short shalt is 59 N/mm ?
'R OELEM 4 (Oct. 2000) A. 98.68 kw C 68.98 kw
'ind the torsional moment (in Newton-rum), developed when the shaft delivers 20 kw B. 96.88 kw D. 68.89 kw
t 200 rpm
96 Shaft uvq g7
Shq{t ing
SOLUTION T 1,927.390 N.mm = 1.92739 KN.m
P 2 rt T N ~ 2 n (1.92739)(480/60) ~ 96.88 kw
o ~ 55 mm ~ 0.055 m
No correct answer In the given choices.
S ~ 59 N/mm' ~ 59 Mpa ~ 59,000 kpa

E'ROBLEM 10 (ME Bd. Apr. 98)


For shaft: s ~ 16T A small countershaft is 1 1/2 inch in diameter and has an allowable stress of 8500 psi.
)"[03 Find the horsepower delivered by the shaft at a speed of 15.7 rad/s.
A. 7.20 C 13.31
59,000 ~ _16T
B.l.4 O. 14.72
n (0.055)3
T = 1.927 KN-m
EE!!i it.q+
p~2 rt T N ~ 2 rt (1.927) (480/60) ~ 96.88 kw s ~ 16T
3
rrd
16T
8,500 ~ - 3
PH.OBLEM 8 (Oct. 1998) ,(1.5)
A hollow shaft has an inner diameter of 0.035 m and an outer diameter of 0.06 m. T ~ 5.63278 in-Ibs 469.4 It-Ibs
Compute the torque jf the shear stress is not to exceed 120 Mpa In N-m.
A. 4,400 N-m C. 4,500 N-m
N :::: 15.7 !ad x~q.~ec x __ ~r~

B. 4,300 N-m O. 4,200 N-m


sec min 2n:rad
N ~ 150 rev/min
SOLUTION
p ~ ~TN ~ 2,( 469.4 )(150) ~ 1339 h
33,000 33,000 P
For hollow shaft
PROBLICM 11 (ME Bd. Apr. 97)
_16TO o _ ... Determine the torque received by the motor shaft funning at 4250 rpm, transmitting
s ~
4 4 11 Hp, through a 10 in diameter. 20° involute gear. The shaft is supported by ball
n(Oo -0, )
bearings at both ends and the gear is fixed at the middle of 8 in shaft length.
120,000 ~ __16 (T)JO 06)_ A. 163 in-Ib C. 132 ft-Ib
rt [(0.06)4 - (0.035)4 J B. 167 ft-Ib 0 138 ft-lb
T ~ 4.50 KN-m ~ 4500 N-m
IEiI!!Iim
PROBLEM 9 (ME Bd. Apr. 97)
p ~ 2,HJ
r What power would a spindle 55 mm in diameter transmit at 480 rpm. Stress allowed
for short shaft is 59 N/mm z 33,000
A. 45.12 kw C 45 Hp 2nT(4250)
11 0 -----
B. 50.61 kw O. 39.21 kw 33.000
T ~ 13.594 tl-Ib ~ 163.128 in-lbs
EI!!Dm
E'HOlll.b:M 12 (ME Bd. Apr. 98)
s _ 16T A 16 It lineshatl has no bending action except Its own weight. What power in Hp can
the shaft deliver at a speed of 200 rpm. Consider that the torsional deflection Will not
nd 3
exceed 0.08/fllenglh.
16T A. 13.2 C. 244
59
"(55)3 B 158 1'16 o
9A Shafting Shafting 99
SOLUTION
, PROBLEM 14 (ME Bd. Apr. 98)
Compute the diameter of a solid shalt transmitting 75 Hp at 1800 rpm. The nature of
the load and the type of service is such that the allowable S, based on pure torsion is
From Machineries Handbook, 24th Ed. p.267 6000 psi.
A. 1 7{7" C. 1 5/16"
For line shaft with no bending action except its own weight. B. 21/16" D. 31/8"
0 2 /3 := ~-­
8.95
where: 0 = shaft diameter, in
SOLUTION
L ee shaft length, ft
02i3 = _~
8.95
o = 2.39 in
P = 2"TN
75(33,000) ~ 2 n T (1800)
4
Hp = 1--,
"0
\ 4.6 )
N
'1 T = 218.838lt-lbs
T = 2626.06 in-Ibs
N = speed, rpm S _ 16T
- nd 3
4
2.39 ) 200 6,000 = 16(2626.06)
Hp
( 4.6 n(d)3
Hp 14.58 hp d = 1.306 in

Therefore use = 1 5/6 in (standard size)


PROBLEM 13 (ME Bd. Apr. 98)
The torsional deflection of a SAE 1040 steel shaft is 0.8' in a length or 1/2 m. The / PROBLEM 15 (ME Bd. Apr. 98)
shear stress is 69 Mpa. Compute the diameter of the shaft in millimeter. Steel The shalt is subjected to a steady load of 36,000 Ibs at a shear stress of 10,000 psi.
modulus of elasticity is 79.300 Mpa or N/mm 2 . Compute the diameter of the said shalt in inches.
A. 51 C. 75 A. 1718 C. 3
B. 50 O. 62 B.21M D.23M

am:mD SOLUTION

S = 16T T = F x r = 36,000 (dI2) = , 8,000 d in-lbs


nd3 S 16T
=ncf3
69,000 = ~­
n (d)3 10,000 = 16(18,000d)
T = 13.548 d 3 KN.m n(d)3
TL
8 =- d = 3.027 in
JG

0.8 Ll
X 1t ~548d3)(1@_ PROBLEM 16 (ME Bd. Apr. 97)
180 0 4 Determine the torque received by the motor shaft running at 4,250 rpm, transrnittlnq
(nd )(79.300.000) 11 hp through a 10 in diameter 20° involute gear. The shalt is supported by ball
32
bearings at both ends and the gear is fixed at the shalt length.
d = 0.062 m 62 mm
A. 163 in-Ib C. 167 in-Ib
B. 132in-lb D.138in-lb
100 ShaJtiflq 1 (J 1
Shafting
SOLUTION torsional sheanng stress IS 28 N/mm"'.
A. 218 C. 312
B. 232 D. 380
P = nTN

11(33,000) = ZnT(4250) m!!Ji1:mI


T = 13.594 It-Ib (12) = 163.128 in-los P = 2nT N
750 = 2nT(1500/60)
PROBLEM 17 (ME Bd. Apr. 96) T = 4.775 KN-m
Compute the maximum unn shear in a 3 inches diameter steel shafting that transmits S ~ 16T
24000 in-Ib of torque at 99 rpm. nd 3
A. 4530 psi C. 4250 psi 16(4775)
B. 3860 psi D. 4930 psi s. 'n(004)3
379,982 Kpa 380 Mpa

IImI!!imD PROBLEM 20 (ME Bd. Oct. 95)


A hollow shaft has an inner diameter of 0.035 m and outer diameter of 0.06 m.
s = 16T Compute for the torque if the shear stress is not to exceed 120 Mpa In N-m.
nd 3 A. 4500 C. 4100
16(24,000) B. 4300 D. 4150
Ss = = 4527 psi
n(3)3
SOLUTION

PROBLEM 18 (ME Bd. Apr. 96) 16TDo


Compute the linear shalt diameter to transmit 12 HP at 180 rpm with torsional s. -2 2-
deflection of 0.08 degrees per foot length. n(D o ·0,)
A. 3 in C. 5 em
B. 2.35 in D. 62 mm 16(T)(0.06)
120,000.000
n[(006)4 ~(0035)4i
SOLUTION

T = 4500 N-m
P = 2nTN
12(33,000) = 2nT(180)
T = 350.14 It-Ibs = 4,201.68 in-los
PROBLEM 21 (ME Bd. Oct. 95)
Design the size of solid steel shalt to be used .or a 500 Hp, 250 rpm application If the
L=IIt=12in
allowable torsional detrection is 1 and the allowable stress is 10,000 psi and modulus
Q
J = l'td'/32 6
G = 12,000,000 psi for steel 01 rigidity IS 13 x 10 pSI.
A. 5" dia. C. 4-5/8" dia.
e = TL B. 4-7/8" dra. D. 4-3/4" dia.
JG
n 4,201.68(12)
0.08' x 180 0 4 m:m:mmI
(nd )(12,000,000)
32 Solving for the shaft diameter 0 based on stress:
d ~ 2.35 in P = Zrt TN
500(33.000) ~ 2 rt T (250)
PROBLEM 19 (ME Bd. Apr. 96) T ~ 10.504.226 It-Ibs
Compute the nominal shear stress at the surface in Mpa for a 40 mm diameter shalt T = 126,051 m-lbs
that transmits 750 K!N at 1500 rpm. Axial and bending loads are assumed negligible.
102 SfW/llill/ IO.~
Sh({!1 fJlq

s = 16T
3
Emmit-w'
n:d
16(126.051) p -
D"N
10.000 c. For short shaft-
nid)3 38
d = 4" Where D shaft diameter, In
N = speed. rpm
P = power. hp
Solving for the shaft diameter based on torsional deflection:
The shaft length IS not given The common practice for torsional deflection IS per 20d
length. (From machineries Handbook) (51! 25.4)N
120 = . _ .
TL 38
I)
JG N = 330 rpm
v = ,D N = IT (0 55 x 3 28)(330) 1870 fpm
126.051(20d)
x Jl!180 - ----- --

('~d2' )(13.000,000) PROBLEM 24 (ME Bd. Oct. 95)


A hollow shaft nas an Inner diameter of 0035 m and outer diameter of 0.06 m
d = 484" Determine the polar moment of mertta of the hollow sf-a-t.
A 1512x10hm~' C 1215x10bm~
4 b 1
Therefore use d 4-7/8" (standard) B1152x10- 6 m D.1125x10 m

PROBLEM 22 (ME Bd. Oct. 95) ~


A 2-m solid shaft IS dnven by a 36-',n, gear and transmits power at 120 rpm. II
allowable shearing stress IS 12 kSI, what horsepower can be transmitted? 4 ,
T:(d o -d, )
A 29 89 C. 35.89 J
B 38 89 D. 34 89 32
IT [(006)' - (0035)4]
J
mmm.mI 32

6
15T J = 1 125 x 10 m':
s
j"[d3
PROBLEM 25 (ME Bd. Apr. 95)
15T Compute the speed of the gear mounted all. a 52.5 mm diameter shalt receiving
12.000 = _.
n(2)3 power from a driVing motor With 250 Hp
T = 18850 m-Ibs 1.570.8 tt-lbs A 2182 rpm C 2071 rpm
8 2282 rpm D. 234 f rpm
p = 2n.~
550 ~
Where T = torque. f1-lb
N = speed, rps D 3N
p
p = 2'(1570 8)(120! 50) 80
3589 Hp
550
(52.5 25.4)3 N
250
80
PROBLEM 2:; (ME Bd. Oct. 95) N 2255 rpm
A short 61 mm diameter shaft transmits 120 Hp Compute the hnear speed of a pulley
55 em diameter mounted on the shaft. 1he: answer IS B (nearest)
A. 1795 lpm C. 1765 fpm
8. 1856 fpm D 2106 torn
10" Shuftinq 105
S}lU(lllln

PROBLEM 26 (ME Bd. Apr. 95) f'I<UIJLEM 28 (ME ae, Apr. 94)
A 3" diameter short shaft carrying 2 inches pulleys close to the bearings transmit how
A solid cylindrical shaft 48.2 em long IS used for a transmission of mechanical power
at a rate of 37 KW running at 1760 rpm The S~ IS 8.13Mpa. Calculate the diameter. much power If the shaft makes 280 rpm
A. 30 mm C. 35 mm A 199 Hp C. 198 Hp
B. 40 mm D 50 mm B. 200 Hp D. 210 Hp

SOLUTION ~
For short shafts
P~2rrTN
37 ~ 2nT(1760/60) D 3N (3)3(280)
P = = 199 Hp
T ~ 0.20 KN-m 38 38
S ~ 16T
rrd3 PROBLE:vI 29 (ME Bd. Apr. 97)
A step shaft made of SAE 1117 steel with an ultimate strength of 69.7 ksi The notch
8130 = 16(42.017)
sensitivity lactor IS 0.00435 IS the constant dependent upon the ultimate strength.
nd 1 check the radius stress raiser
d ~ 0.050 m = 50 mm A. 0.192 C 0250
B. 0386 D. 0775

PROBLEM 27 (ME Bd. Apr. 95)


~
A hollow shaft with outside diameter of 14 em and wall thickness of 0.8 em transmits
200 KW at 400 rpm What must be the angular deflection of the shaft If the length IS From Faires p 113
5 meters? The material of the shaft IS C4140 steel 1
A 0.84 dog C 1 83 deg q = - -- -
B. 1.14 deg D 1.50 deg
1 , (a! r)
1
0983 =
SOLUTION 1 I 0.00435

P = 2" T N = 0.25
200 = Zrr T(400/60)
T = 477465 KN-m PROHLEM 30
A shafl IS used to transmit 200 KW at 300 rpm by means of a 200 mm diameter
D, = D. 2 t sprocket Determine the force tangent to the sprocket
D = 014 - 2(0008) = 0.124 m A 60 44 KN C 60 88 KN
B. 60 33 KN D 6366 KN
J = rr/32 (D.' - D,') = rr/32 [(0.14)' - (0.124)"] = 1.45042 x 10" m"
~
From Vallance p 31. Table 2-6 for C4140
steel. P=2TITN
G ~ 12000.000 pSI = 12.000.000 (101325/147) = 82714.286 Kpa 200 " 2 II T 1300/60)
II c TL T ~ 6.366 KN-m
JG
T :::;- F x r
477465(5)
H 00199 rad (180" IT rad) 1.14 deg 6366 = F, (0 20/2)
145042,10 '(82.714286) F = 63.66 KN
lOb Shullilln 107
S/wjl illy
PROBLEM :J]
Find the diameter of a steel shaft which will be used to operate a 110 t0N motor
E·)!''ir.g+
rotating at 5 rps If torsional stress IS 90 Mpa.
A. 60.2 mm C. 38.3 mm 2"TN
P =
B. 583 mm O. 46.2 rr-m 33,000
2nT(1400)
50 --------
Em!!i:mD 33,000
T 18757 It-Ib x 12in/lt 2250905 m-lb
P=2nTN 16T
110 = 2 n T (5) S
r::d J
T = 3.501 KN-m
16T 16(2250905)
S 500 - ---- -----
J 3
rrd rId
d = 2.84 In
90000 = 16(35()1)
, J
'd P[,OIlLEM :14
d = 0.058298 m = 58.298 mm A hollow shaft that has a 100 mm outside diameter and 80 mm msrdc diameter IS
used lo transmit 100 KW at 600 rpm Determine the shaft stress
A 1373 Mpa C. 19.86 Mpa
PROBLEM 32 B 1682 Mpa D. 17.21 Mpa
What IS the speed of 63 42 mm shaft that transmits 75 KW if stress is not to exceed
26 Mpa? ...--r.lI.fkll.~
A. 550 rpm C. 650 rpm
B 660 rpm D. 700 rpm P=2,TN
100 = 2, T 1600/60)
E.:.t!!immI T = 15915 KN-m

16TO, 16(1.5915)1010)
S = 16T S 4 -- 4 13728.73 Kpa
nd J il:(Do -0 1
)
;[(010)" (008)']

16T S 13.73 Mpa


26 000 = ---- -
, n(0.06342)3
P],Ol3LSM 35
T = 1.3022 KN-m Determine the length of the shortest 2 mm diameter bronze wire which can be twisted
P=2rrTN through two complete turns Without exceertnq a shearing stress of 70 Mpa. G =
75 = 2 rt (1 3022)(N) 35,000 Mpa
No 9.166 rps x 60 = 550 rpm A. 6,283 mm C. 4.836 mm
B 5,484 mm O. 7.263 mm

PROBLEM 33
A steel shalt transrruts 50 Hp at 1400 rpm. If allowable stress is 500 PSI, find the shalt
E.:.t!!immI
diameter
16 T
A. 3.58 In C 1 65 In Ss
B. 2.84 In D 2.54 In TId:)

70 = 16 T
rt(2)3
T = 109.96 N mm
A = 2 turns = 2 rev = 2 (2TI) - 4"
I (),~ ,'-jlllllllll!)
~11(llrl!ln lOq
10996 L
4)1 = -- --
SOLUTION
IT (2)4
(35000)
32 ' lit x 1,3.281 _ 0.30478 m
D
L 6.283 mm
T = F(D!2) = 5(0.30478/2) = 0.7619 KN-m

PI,OBLEM 36 d 2,nxI/12xl/3.281 = 005079m


7
A hollow shaft developed a torque of 5 KN-m and shaft stress IS 40 Mpa If outside J rrd'i32 = n(O 05079)";32 = 6.533 x 10 m"
diameter of shaft IS 100 mrn. determine the shaft inner diameter. TL
8
A 68 43 mm C. 58 38 mm JG
B. 6328 mm D. 77.64 mm
T
ell
JG
Em!!ImD 0.7619
OiL 0.014043 rad/rn x 180 'ITrad
1§TD o (6533;; 07)(83xl 0 6 )
s 4
n(D o - D:') ell 0.805 -rn
16(5)(0 10)
40.000
n[(O 1())4 ~-(D,)1] I'HO!3LEM 39
(0 1)' D,' c 6 3662 x 10" What IS the minimum diameter of a steel shaft which will be used to operate a 14 KN-
6
D, ~ 0.07764 m = 77.64 mm m torque and will not be twist more than 3 In a length of 6 m? (G = 83 x 10 Kpa)
A 10064 mm B. 118.45 rnm
B 96.80 mm C 120.72 mm
l'I,OBUcM :17
What is the polar moment of Inertia of a solid shaft that has a torque ot 1.5 KN-m and
a stress of 25 Mpa?
EJ!!,it.'%I
A. 246 x 10 6 m'l C 3.46 X 10' m'
B 2.02 x 10- 0 m'l D 1.24 x 10'[ m 1
e = torsional deflection
TL
(J = ---
JG
Em:!!i:ir:mI 14(6)
3' (IT/180') - ------------

16T J(83Xl 0 6 )
s
nd 3 5
ITd 4
J = 1.93286 X 10 m'
25.000 = 16(1,5) 32
3
ITd _ red 11
1.93286 x 10" =
d 0.0673556 m = 673556 mm 32
4 4
rrd._. = rr(0.06755) = 2.02 x 10" m' d = 011845m = 118.45mm
J
32 32

PROBLEM :lH ['I,OIlLEM 40


A force tangent to a foot diameter pulley IS 5 KN and IS mounted on a 2 Inches shaft. A solid shaft 5 m long IS stressed to 60 Mpa when twisted through 4 Find the shaft
Determine the torsional deflection If G = 83 x 1Or, Kpa diameter If G . .:. 83 Gpa
A 0805 -rn C 0.768 '1m A. 95.46 mm C 101 32 mm
B 90.42 mm D. 103.54 rnm
B 0.654 -rn D 0.938 .m
j In Shu!1 inn .SIi(~lt!nq II1
IlE!!mmI Em:!!ii%.:D
s = 16T For rnam shaft:
3
i'Td

Solving for T in terms of d:


p
o3 N (50/25.4)(3001 ~ 2860 Hp
3S 80 80
T = rrd
16
3 PHOBLEM '+:'\
rrd (60.000)
T 11780.97 d
J
___ Eqn 1 A 1 5 In diameter short shaft IS used to transmit 44,4 Hp Determine the shah speed
16 A 500 rps C 400 rps
TL B. 833 rps 0 450 rpm
H
JG

4 x n:/180'
TIS) E:.'l!!IiI:.mI
J
IITd ](83x106) SOLUTION:
32
T = 113774606 d' _ Eqn. 2 For short shaft
03 N
Equate 1 and? /"
1 r f1 q.:. Cj-l c. p =
38
113774.606d' = 1178097,r (::I: :;;
d = 0.10354 m = 10354 mm (1.5)3 N
¢' ! Ii:; ;:;d Le.' 44.4
38
N = 500 rpm x 1160 8.33 IpS
PROBLEM .+ I
What IS the diameter of a line shaft that transmit 150 KW at 15 rps?
A 228 In C 1.62 in
PR01J!.EM 4-+
B. 3.54 In 0 2.04 In
A 3 in solid shalt IS desired to replaced a hollow shaft havnq 4 In outside diameter
Consider the strength to be the same. determine the mside diameter of hollow shaft
Em:!!ii%.:D A 2.5 In C 3.48111
B 3 0 In 0 4.0 In
For line shaft.

p =
O'N
- E:.'l!!IiI:.mI
53.5
For 50110 shaft
03(15x60)
150'0.746 = S = 15T 16T
53.5 0.1886T
nd 3 ,,(3)3
D , 2.28 in

For hollow shaft


PHOBLEM 42 16TO r
A main shaft has 50 mm diameter IS running at 300 rpm What power that could be
s - 4"' --- 4
"(0,, 0 1
)

delivered by the shaft?


A 3040 Hp C. 32 50 Hp 01886 T = 16T(4)
B. 2860 Hp O. 16.42 Hp rrl4 J 0')
(41',0,'=108
[J, = 348 In
112 Sh(~{t iun I L;
SlluJrll1()
P[{OULEM 45 D:'(954931
A motor IS used to drive a centrifugal pump thai discharges 3000 h.rnin at a head 01 150(095)
535
10 m. The pump efficiency IS 6SC'o and running at 550 rpm. Find the torsional stress of
D = 2 In
shaft If shaft diameter IS 35 mm.
A. 1385 Mpa C 1285 Mpa
['!,OllU':M 47
B 11 85 Mpa D 1487 Mpa
An 800 mm diameter circular saw blade IS driven by a 1800 rpm motor with belt
speed ratio of 1 8. Find the panphcra! speed of the blade
IlE:!!IiI:mI A 137.43 ft/sec C. 13243 IVsec
B 14065 ft/sec D. 13521 It/sec
Brakl Power
Solving for the power output of the pump"
Q = 3000 li/rrun = 3 m'!/min
P ~ wQ h IIE:!!iml3
P ~ 9 81 (3160)( 10)
NI / N;. _ 1.8
P ~ 4905 KW
1800 ; N. = 1 8
Solving for tho power Input of the pump: N.· = 1000 rpm
V • IT D rl ,(08)11000/60) = 4188 m/sec x 3 281 137.43 It/sec
Brake power = 4.905/068 = 7213 KW
P=2nTN
7.213 = 2, T(550/60) [,1'OBl.!':"1 4H
A machine shaft IS supported on bearings 1 m apart IS to transmit 190 KW at 300 rpm
T = 0.1252 KN·m
while subjected to bending load 01 500 kg at the center If maximum shearing stress IS
40 Mpa. determine the shaft diameter
16(0.1252)
s- ]
14876.63 Kpa 14.876 Mpa A. 100 mill C. 94 mm
n(0.035) B. 90 mm 0 98 mm

Hali,,-p'
PROBLEM 4(;
A circular saw blade has a circular speed of 25 m/sec and 500 mm diameter IS driven
by means of belt that has an effiCiency of 95% Find the required diameter of the
For Simply supported beam with load at center.
driven shaft If power transmitted IS 50 hp.
PL (500x000981}(1)
A. 1 In C. 3 In M
B 2 In D. 4 in 4 4
M 1.226 KN m p ~ 500 kg

E'.l!!iil.:'mI

V =rrDN
Sol\/Ing lor the torque developed"
P=2nTN
190 = 2 rt T (300·60)
I ~ PI
T 604i88KNm
25 = rt (0.5) N
N = 954.93 rpm For shaft subjected to torsion and bending load.
Based on shearing.
For line shaft:
16 "' ')
S _ \ M" - T"
D3N ;-rd\
P =
53.5 16 ') <'
J ~ 0 ()OO " ,'(1 226)" T (6.04788)
P lei = D N :r:cl
53,5 d 009227 rr; = 9227 mm
II" ShU/II/H; :-.,!Ui!llfU! I J 5
PROHLSM 49
A 100 mm diameter shaft \5 subjected to a torque of 6 KN-m and a bendinq moment
I!m!!iiI:mI
of 2.5 KN-m Find the maximum bending stress developed. For line shan
A. 4584 Mpa C 50.2B Mpa
B. 6025 Mpa D. 5546 Mpa
p
D"N
SOLUTION 53.5

For shaft subjected to torsion and bending IOrHL Based on compression, 0 3 (900 )
200
53.5
16 . . -', --,
31 = -- I'M~\"'M2 __ T.:' I D = 2 28 In or 2 7/16 III (standard)
Ted'

16 -- ?- - ') I PROBLEM 52 (ME Bd. Oct. 1996)


S, ='- ",125+\1 2 5 ;- - (6;- , A centnlugal pump IS dvectty coupled to a molar The pump rating IS 3600 liters per
n(0.1) J
minute against a total head of 8 meters of water. Pump efftcrency IS 65"~, at a speed
of SSO rpm Calculate the torsional stress Induced on the 40 mm diameter motor
S, 40,836.62 Kpa 4584 Mpa shaft.
A 11,193.45 kpa C. 10.01085 kpa
B. 13467.34 kpa O. 16,34567 kpa
PROBLl';M E)O
A round steel shaft transmits 1/2 Hp at 1800 rpm. Tile shear modulus of elastiCity is
12.000,000 PSI The torsional deflection IS not to exceed 1 deg In a Jer-otn equal to 20 Brake Power
times the diameter Find the shaft diameter.
A 1.04 In
B. a 257 In
C a 864 In
D 2 045 In
Im:!!mm
o - 3,000 Ills -.= 3 rn?s
1.t}!IJ'"
E'.l!!iiI:mI p,
WOh 981(3)18)
7,24 kw E = 65%
e 0.65
p = 2nTN P",2rrTN
33.000 7.24 2 n T (550'60)
, 2nT(1800) Tea 1208 KN-m
1,2 = - ,
33000 16T
S
T = 1.4589 f1-lb = 175071n-lb rrdJ
16(0 1258)
S ~ 10.01085kpa
H = TL n(O 04)3
JG
17507(20d) PROBLEM 5;,
("xrr/180'
( IT d')(12000000) A round steel shaft transmits 0.375 kw at 1800 rpm. Tho shear rnodu.us of elasticuy
32 IS 80 Gpa The tors.onal deflection is not 10 exceed 1 degree In a length equal to 20
d 025727,n rnaroe'c-s Fmd the shaft diameter
A 6621 mm C 12344mm
3 8342 mm o
16300 mm
PHOBLEM 51 (ME Bd, Oct. 1996)
A line shaft to transmit 200 Hn at 900 rpm. Find the diameter of the shaft.
IS
A. 23/16 m C 315/16 m
E.l!!imD
S. 27/16 In 0 13/16 In P 2,1 N
j)h
Sl](~fl i nq
.'-,}wfl inq 117
0.375 = 2 rrT (180060)
T = 0 001989 KN-m SOLUTION

TL m'I,JI ,~ -:.: 1/2 m",I'J


11=
JG m=wV
wV 1 1 i2wV 2
n 0 001989(20d)
x- -_
180 n:d4 6 V, = 1/2 V,
(80xl0 )
32
d = 00066206 m = 6.62 mm (rr/4)(D,' - D,')L = 1/2 (ni4)(d')L
,.., '") 2
0,," - 0' " 112 d
(1.5)' - Dr' = 1/2 (11"
PROBLEM 54 D. = 1 .322 In
A round steel shaft rotates at 200 rpm and IS subjected to a torque of 225 N.m and a
bending moment of 340 Nrn The allowable sheanng stress IS 40 MN/n/ the
allowable tensile IS 53,3 MNim;! Find the diameter I'ROBU:M ,,6
A 1/4 In diameter shaft IS designed with a working stress 01 7000 pSI In shear If it
A 23.45 mm C 28.56 mm
B. 3445 mm 0 4149 mm rotates at 1725 rpm and shear modulus of elasticity IS 12.000,000 pSI. what is the
torsional deflection In degrees per foot of length?
A 3.82" C 3.21'
E:.7:!!mmI B. 256 D. 423"

Based on sbeacr.o-
IIE!!Im3
16 r '--2
S. = _ \ M , T,l
nd' _
S = 16T
16 ' -- -, - -- r; J
n:d
40 x 10 = - _j
'l,(0.34 t ,(0.225t
nd ,
7000= 16T
d = 0.0373 m = 37 3 mrn . "(I! 4)3
T = 1.789 fI-lb
Based on compression.
16 2
T2
51 = ---, - M + vM nd 4rr(1 4)4
'I .

t
nd<l J= = _ 3 835 x 10' ff'
32 32
53 3 x 10' = 16 10.34 ,
;r:d 3
\(0::34l r (0.225)2 L·lf1=12In
TL (1789xI2,112) J/nrad
8= = 00559 rad x 180
d = 0 04149 m = 41 49 mm JG 3835xl0·'(12.000.000)
II = 3 21
Therefore choose the larger diameter, d ·,41 49 mm

PFOl-lLEM C,7
A hollow shaft has an outside diameter of 4 In and an Inside diameter of 3 In. Find
PROBLEM 55
the marnctcr of a solid shaft with equal strength In torsion
A 1 In diameter shaft IS to be replaced with a hollow shaft of the same rnatenal
A2121n B323,n
weighing half as much, but equally strong In torsion The outside diameter of the
C >1?3 In [) 3 S2 In
hollow shaft IS to be 1 1/2 in Find the inside diameter.
A. 1322 In C I 653 III
B. 2 123 In 0 3 123 In IIE!!Im3
S, "1 =- SI"...,.....
I] H J'. ( ·!I -, 11 q
....... llfl/llllq
16T 16TO"
3 ::::: --.-1 --1
IT d 7T(Oo D )
0,/ - 0,'1-:::. rj)
(4)'. (3)' 0 d' (4)
°
do 352 In

I'ROIILEM 58
A 1;4 In steel shaft transmits 30 In-Ib of torque The effective length of the shaft IS 12
in: the modulus of etasncuv In shear IS 12.000,000 pSI. Find the angular deflection in
degrees.
A. 234"
B 1,23'
C, 448'
0, 634"
GmD
mm:mD ~
1. Key - a machine member employed at the mtartacc of a pair of mating male and
ITd' . )4 ~
IT(~' 4 . = 3835 terna'c Circular cross-sectional members to prevent relative angular motion
>1
J = -' x 10ft
32 32 between these mating members
TL (30)(12)
H ::::: = 0.078 rad x 180"/mad 2. Keyway a groove ill the shaft and mating member to which the key fits.
JG 3835xl0" (12000000)
00448' 3. Splines - permanent keys made integral With the shaft and fitting Into keyways
broached Into the mating
PHOl3LEM SCI
A 1 1/2 In Monel shaft IS used In a torsional application. Based on operating speed of 4. Square key has a square cross-section with tlalf of ItS depth sunk In the shaft
100 rpm and safety factor of 12. How much horsepower can it transmit? Assume the ano half in the hub.
shearrng stress IS 3/4 of the ultimate tensile stress which is 100.000 pSI
A 657 hp C 234 hp 5. Flat key has a reqular cross section With a smaller om-cison placed In the
B. 345 hp 0. 5.23 hp radial direction with half sunk In the shaft and half In the hub and IS used where
the weakening of the shaft by the keyway IS serious
IIE!!imD
6. Round key - has Circular cross section
S" = 3'4 S,
S" = 34 (100,000) 0 75,000 pSI 7. Barth key IS a square key With bottom two corners beveled.

S _ SSLJ 8. Woodruff key consists 01 one-half of a Circular disk fitting into a rectangular
,- FS keyway In the female member and a semi-circular keyway In the male member
S,075,000'12
S, 6.250 pSI
0
9. GibMhead taper key - is a flat key With a special qrb-he ad to facilitate easy
dnvrnq and removal of the key
S, 0 16T
, 7Td 3
10. Saddle key IS a flat key used Without a 'Keyway In the shaft
16T
6250= -',
"(1.5r 11. Kennedy keys are tapered SqU,Hf' keys With the diagonal dimension In a
T -4141 751n·lb 0345.141!·lb circumferential direction.
P = ~"TN_ = 2IT(34514)(100) 12. Feather key is one which has a uqhl [II into one member and a loose Sliding lit
= 6 57 hp
33,000 33,000 In thr' 1Tl,1tltl(j member thus allowmq tile hub to move along the shaft but prevents
ILO {(I'II_", 121

rotation on the shalt

Types of keys:
1. square key 3. Compressive Stress (So) of key
2 flat key
3
4.
found key
barth key
Fe
Ih/ 2
5 woodruff key So
6 oro-nead key (h/2)L
7. saddle key
8 kennedy key
h
9. leather key
//

rmmmm Compressive area -~


4. Shearing Stress (5,) of key

s, = ~
wL
Key F where L -= tenqth of key
W -= width of key
h h - height of key
r
w
Shaft w

1. Power of key: 5. Relation of key and shaft for the same material:

D
A. P 21t T N, KW w L = 12 D
4

21tTN H
B P 6. Force tangent to pulley rim
33000' P F'

l?
0-
/

T=T'
2. Force transmitted, F R
F.r=F'.R T'
r
T T
F
d/2 ~ _ force tanqent to the key
F force tangent to pulley rim
where d shaft diameter R radn.s of pulley
122 tc: c-~ I ':> 1'23
PROBLE:vl I (ME Bd. Oct. 97) 7788

1
A 76.2 rnrn In diameter shafting of SAE 1040 grade, cold rolled, navinq a yield point of f- 28,8444 N
(0054/2) '1
50 kSI and with a 3/4 x 5 Inches key Compute the minimum Yield pomt In the 'Key In ;.<:
P' ,
order to transmit the torque of the shaft. The tactor of satety to use IS 2 and S
0.50 S/ For shearing ot key 1.5875 em <~;~. L
A 39120 kSI C 27920 kSI
F 1.11 em
B 42.130ksl D. 47.120 kSI
S.
wL
SOLUTION 28844.4
60,000000
00111(L)
For Key
L -c 00433 m = 4.33 em
w -= 3/;" ~ 0.75"'
L = 5"
~ For compression of key'
d = 762 mm 3 In
Sy ( )
S,. S, 2 F 54mm
FS h L
3/4"
50,000
S" " 25,000 pSI 2(28.844.41
2 90,000.000
o 015875(Lj
S = 16T L ~ 0.040377 m = 40377 em
rrd3 Therefore choose the longer length of key
L c 433 em
25,000 = 16T
n( 3)'
T 132,535 94 m-lb
T = F r I'H013LEM 3 (Ocl. 1998)
132,535.94 = F(3/2) A 76.2 mm cramercr shattmq of SAE 1040 grade, cold rolled, havno a Yield oom' or
F = 88,357.29Ibs 50 kSI and With a 3/4 x 3/4 x 5 In key Compute the minimum Yield pou-t In the key In
For sheanng of key order transmit the torque of the shaft The factor of safely to usc IS 2 and S,c, .;: 05

S,
F 88,357.29 S,
23,561 94 psi A 47 52 kSI C. 4712 kSI
wL 075(5)
B. 4725 kSI o 4721 ksi
Sy = S, x FS
S, = 23,561 94 x 2 = 47,120 PSI 4720 kSI
Em!!Ii[-':W
PHOflLEM 2 (ME Bd. Ocl. 97) For Key
w ::::: ':'~ " =-- 0 75"'
A keyed sprocket deliver a torque of 778.8 N m thru the shaft of 54 rnrn 00. The '",ey
thickness IS 1.5875 em and the Width IS 1.11 em. Compute the length of the same
- c-:
L - .,
d c:: 76.2 rnm = 3 In
key. The permissible stress value of 60 Mpa for shear and 90 Mpa for tension.
A 39.12 em C 52.22 em
B. 4.82 em 0 432 em S, 50.000 c 25,000 PSI
S"
FS 2
Em!!iiIm
S = 16T
F
T Tl"d 3
3"
16T
25,000 c
IT(3)3
124
T = 132.535.94 in-Ib K('l/S 125
T = Fr PROBLEM 5
132,535.94 = F(3/2) A metric M4 x 15 square key is used with a 16 mm shaft. If the allowable shearing
F = 88,357.29Ibs stress is 50 Mpa. How much torque can the assembly handle? The rotational speed
For shearing of key: of the shaft is 600 rpm.
F 88,357.29 A. 20 N-m C. 24 N-m
Ss = --- 23,561.94 psi
D. 34 N-m
wL 0.75(5) B. 12 N-m
Sy = Ss x FS
S, = 23,561.94x2 = 47,120 psi 47.20 ksi SOLUTION

An M4 x 15 square key, means w = 4 mm, 4mm


h = 4 mm and L = 15 mm
PROBLEM 4 (ME Bd. Apr. 98) 4mm
A keyed gears delivers a torque of 912.4 N-m thru its shaft of 635 mm outside For shearing of key:
diameter. If the key has thickness of 15.875 mm and width of 11.1125, find the length F
of the key. Assume the permissible stress values of 61.2 Mpa for shear and tension Ss= -
wL
at 99.8 Mpa. 2
A. 47.42 mm C. 42.22 mm Ss = 50 Mpa or 50 N/mm
B. 39.72 mm D 46.92 mm F
50 =
(4)(15)
~'
Em!!iImII F = 3000 N
T = F (d/2) = 3000(0.016/2) = 24 N-m
E
I 16 mm
)1
T = Fxr
912.4 = F (0.0635/2)
F = 28,737 N 15.875m _ PROBLEM 6
11.11 em A flat key is to be designed for 75 mm diameter shaft which will transmit 150 'r<YV at
Based on shearing of key: 400 rpm. If allowable shearing stress is 200 Mpa and key width is 15 mm, determine
F the length of key.
Ss = - -
wL A. 30.65 mm C. 33.75 mm
B 31.83 mm D. 32.85 mm
61200 000 =~37

" 0.0111125L
L = 0.04225 m = 42.25 mm Em!!iImII
Based on compressive of key:
P = 2rrTN
150 = 2rrT(400/60)
Sc = 2F T = 3.581 KN-m
hL
2(28,737) T = Fxr
99,800,000 ~ . _ - -

0.015875L 3.581 = F x (0.075/2)


I
L = 0.03628 m = 36.28 mm F = 95.493 KN IE )
I 75 mm

Therefore use the longer length to be safe: For shearing of key:


L = 42.25 mm F
So =
Lw

200,000 = _9~~~3

L(0.015)
L = 0.03183 m = 3183 rnrn
126 J,! '1/', 127
PROBLEM 7 1\ <+3 KN C. 45 KN
A rectangular key is used in a pulley to transmit 100 KW at 1000 rpm on 50 mm shaft B. 48 KN D. 46 KN
diameter. Determine the force required to remove the shaft from the hub if friction is
0.4.
A. 33.24 KN C. 28.35 KN K=I-15·UW..'
B. 36.85 KN D. 30.55 KN
For line shaft:
SOLUTION 3N
D
P =- -
53.5
3
P=2nTN 15/0.746 = .0
(600 )
100 = 2nT(1000/60) 53.5
T = 0.9549 KN-m o = 1.2148 in = 0.03085 m
T = Fxr L = 1 in = 0.025398 m
0.9549 = F x (0.050/2) w = 0/4 = 0.03085/4 = 0.007714 m
F = 38.197 KN F
Ss =
Lw
The friction on key experience both upper half on one side and lower half on the other
F
side.
230,000 = (0.001'714)(0.025398)
Force required to remove the shaft from the hub F = 45.061 KN
P = 2 f F = 2(0.4)(38.197) = 30.55 KN

PROBLEM 10
PROBLEM 8 A square key is to be used in a 40 mm diameter shaft and that will developed a 2 KN-
A 7/16 in height x 3 in length flat key is keyed to a 2 inches diameter shaft. Determine m torque. If bearing stress of the key is 400 Mpa, determine the cross sectional
the torque in the key if bearing stress allowable is 25 Ksi. dimension of square key to be used if key length is 30 mm.
2
A. 16,406.25 in-Ib C. 17.42 in-lb A. 324.80 mm C. 446.80 rnrn"
B. 15,248.56 in-Ib 0 246.75 in-lb B. 246.80 rnrn" D. 277.77 mrn"

&-i-"'5it-D "W"'mE'
w = 7/16 = 0.4375 in T = Fxr
7/16" 2 = F x (0.04/2)
F
Sc = ---~- F = 100 KN w
(h / 2)L
w
F F
25000 = ------ Sc = ---
, 3 (0.4375/2] (h / 2)L
F = 16,406.25 \bs
400000 = 100
, (0.030/ 2)(h)
T = Fxr
h = 0.01666 m = 16.6667 mm
T = 16,406.25 x (2/2) '" 16,406.25 in-Ibs w = h = 16.667 mm
A '" 16.667 x 16.667 = 277.77 rnrn"
!( )
, 40 mm
PROBLEM 9
A rectangular key is used in a pulley connected to a line shaft at 15 KW and 600 rpm.
If shearing of key is 230 Mpa, determine the force acling on key length if key width
one-fourth of shaft diameter and key length is 1 inch
KellS 129
12R
PROBLEM II EmmmD
A 100 KN force is acting on a key that has a length of 4 times its height. If bearing
stress of key is 400 Mpa, determine the height of key.
For the same shaft and key material:
A. 1054 mm C. 12.65 mm

B.1118mm D.15.25mm
w '" 1/4 0
w = 1/4 (0.12)
w :" 003 m '" h
SOLUTION
F
Sc'" - - ­
(h /2)L
F

s, '" (h/2)L
F
H
410,000 = - ­
w
(0.14)(0.03)
L '" 4 h
F '" 1722 KN
100
Fxr'" F'xR
400,000 '" (4h)(h /2)
1722(0.12) '" F' (1.2)
F' '" 172.20 KN
h '" 0.01118 m

h '" 11.18 mm

PROBLEM 12
A one meter pulley is fastened to a 100 mm shaft by means of 25 mm square key and
150 mm long. What force tangent to pulley rim will shear the key if shear stress is 200
Mpa?
A. 65 KN C. 75 KN
B. 70 KN D. 80 KN

EiD!Im'D
F

Ss '" Lw

25mm
F
200,000 '" 0.025(0.15) 25mm

F = 750 KN

Torque on key = Torque on pulley


r, '" Tp

F x r = F' x R

750(0.100) = F' (1)

F' '" 75 KN

'( )
I 100 mm
PROBLEM 13
A 1.2 m pulley is fastened to a 120 mm shaft by means of a square key with 140 mm
long. What force tangent to pulley rim will crush the key if bearing stress is 410 Mpa.
Assume key and shaft are of the same material.
A. 16042 KN C. 18042 KN
B. 16525 KN D. 172.20 KN
130 Couplif/S} Coupling 131

2. Total force transmitted(F)

8 Coupling

3. Force transmitted per bolt(F b )

~ where:
n = no. of bolts
Flange Dc = bolt circle diameter
---71 ~t
~
1. Coupling - is a mechanical device
• 4. Shearing of bolts(Ss)

which is used to connect length of


shafts permanently.

2. Rigid couplings - couplings that do


not allow angular, or rotational Shaft

• Shear Area

flexibility and used with collinear


5. Compressive stress on bolts and flange(Sc)
shafts.

Flange
I
3. Flange coupling - type of rigid coupling which consist of two halves of flanges
connected by each other by bolts. -4 l~t

~i
where:

4. Sleeve or collar couplings - rigid coupling which is a cylindrical collar pressed t = thickness of flange

over the ends of two co-linear shafts. d = bolt diameter

5. Flexible couplings - coupling which allows angularity to take care of


misalignment of the shafts
--+-­
mmmm d
Compressed Area

1. Power transmitted PROBLEM 1 (ME Bd. Oct. 97)


A flanged bolt coupling has ten (10) steel 25.4 mm diameter bolts evenly tighten
A. P = 2. n TN, KW around a 415 mm bolt circle. Determine the torque capacity of the connection if the
2
allowable shearing stress in the bolt is 50 MN/m .
A. 5995 KNm C. 46.15 KN.m
B P = 21tTN

33,000,HP
B. 52.6 KN.m D. 43.8 KN.m
132 Ccn ipl i iu]
COlLpiing 133
Em!!J:mD
For shearing stress in bolts SOLUTION
-1
s, = --~--2
(nI4)d
P = 2rcTN
where: Fb = force transmitted per bolt
60 = 2 rt T (180/60)
50,000 = . . ~.--Fb -'- T = 3183 KN-m
(n 14)(0.0254)2
Fb =' 25335 KN Torque on coupling:

T = F (OJ2)

F = total force on bolts = n x Fb 3.183 = F (0.180/2)

F = 10(25.335) = 25335 KN F = 353677 KN (total force)

T = F (--~) = 253.35(0.415/2) = 52.57 KN.m F 353677 = 4.421 KN


2 Fb = force transmitted per bolt - = - 8
n
Fb 4.421
= 14,542.67 Kpa 14.542 Mpa
PROBLEM 2 Sc = dt (0016)(0.019)
A flange bolt coupling consist of eight 20 mm diameter steel bolts space evenly
Fy .148
around a bolt circle 300 mm in diameter. If the coupling is subjected to a torque of FS -+--- = 30.8
15.1 KN-m, determine the maximum shearing stress in the bolts? Sc 14542
A. 40 Mpa C. 70 Mpa

B 38 Mpa O. 29 Mpa

PROBLEM 4
~i·l!iii[·H· A turbine is connected to a generator by means of flange coupling that has a bolt
circle diameter of 500 mm. The generator output is 40 MW, 3600 rpm and 90%
T =F x r etticiency. If there are 16 bolts, determine the force acting on each bolt
A. 26.41 KN C. 35.62 KN
15.1 = F (0.300/2) B 29.47 KN O. 3261 KN
20 mm
F = 10067 KN (total force) Lomm Emmir.m
Fb = force per bolt L 40000
Brake Power = .i.:»:
0.9

= 44444.44 KW

Fb = Fin =' 100.67/8 = 1258 KN P = 2rcTN


44,444.44 = 2 rt T (3600/60)
For shearing of bolt T = 117.89 KN-m

Ss ~ .~- -
12.58
--_.~._-_.-

40043.38 Kpa = 4004 Mpa

o co. 500 mm = 0.50 m

rc d2
rc 2 T = F (0/2)
- (0020)
4 4 117.89 = F(0.50/2)

F = 471.57 KN

PROBLEM :3 F b = Fin

A flange coupling having 180 mm bolt circle and 19 mm thick uses 8 bolts, 16 mm where: n = no. of bolts
diameter to connect two shafts. It is use to transmit 60 KW at 180 rpm. Determine the 71.57
F I , = ---- = 29.47 KN
factor of safety in bearing if yield point in compression is 448 Mpa 16
A. 156 C 30.8 /

B 185 o. 25.4

1:34 ( '0 I I l ' II 1111 COll/J[i,,,/ 135


PH.OBLEM 5 .,:r.]I{·.K·~
A 75 mm diameter shaft is transmitting 300 KW ill 600 rpm A solid coupling has 6
bolts each 18 mm In diameter. Find the required bolt circle diameter if shearing stress
in bolts is 27.5 Mpa.
A. 22740 mm Fb
C 25450 mm s,
B. 233.60 mm D. 27260 mm dt

T = F (0/2)

SOLUTION 15 = F(O.25/2)
25 mm
F = 120 KN
T 250mm
P=2rrTN

4=:.~
Fb = F = 120 = 15 KN

300 = 2 1l T (600/60)
n 8

T = 47746 KN-m
75mm


18 mm
S, = - ~ Solving for t based on compressive on key:
(rr / 4)d 2 15
Dc 15000 =
, (0.025)t
27 500 = f},__

t = 0.040 m = 40 mm
, rr/4(0.018)2

Fb = 6.9978 KN

F = 6(6.9978) = 41.9868 KN

T = F(D c/2)
4.774 = 41.9868(Oc/2)
Dc = 0.22740 m = 22740 mm

PROBLEM 6
The total force of 125 KN is required of flange coupling that has 5 bolts has also
shearing stress of 30 Mpa for bolt. Determine the required diameter of bolt.
A. 62.54 KN C 45.62 KN
B. 54.21 KN O. 32.57 KN

--i·'!!ImD
Fb 125/5 = 25 KN
Fb
Ss ------,-­
(rt /4)d 2

25
30.000 = -- ­
(rr/4)d 2

d = 00325 m = 32.57 mm

PROBLEM 7

A flange coupling with bolt circle diameter of 250 mm uses 8 bolts, 25 mm diameter.

the torque transmitted is 15 KN-m and compressive stress of bolt is 15 Mpa.

Determine the required flange thickness.

A. 25 mm C. 35 mm
B. 30 mm D. 40 mm
Pressure \', I :I
136 Pressure V, 'S.'>, '/
,.,>.,>/.[

,9 ~r~s;sur~
"~s.sel I'
P
rnnxrrnum pressure at the bottom
w h == (SG x ww) h
-<-t
h

For Spherical Pressure Vessel


1< )j
Di

(.. If efficiency of joint is not considered:


S ==!,]J~
4t , ,
, I
I ,

(. If efficiency of Joint is considered:


I
,, ,,
,

,, ,,,
S== POi ,
41 e t~, D'I ~t
(, If the ratio of wall thickness to the inside diameter (~ ) is less than 0.07 then ,

. OJ k ),
. ~ !
the cylinder is considered as thin-wall. (.. OJ :::: Do -2t
For Cylindrical Pressure Vessel where: e:::: joint efficiency
t

(.. If efficiency of Joint is not considered: If the ratio of t/D, is greater than 007, then the vessel is considered as thick-wall.
S, == tangential stress or hoop stress
S ",PDi " Using Lame's equation for internal pressure:

Ql'~St+~-1l
2t
SL longitudinal stress t ::::
PDf
2 SI-Ii J
Sl Ie ~ where:
4t Dr
1 == wall thickness SI tangential stress
D == inside diameter P, internal pressure
<. If efficiency of joint is considered:
PO PDt (. When the vessel is subjected to an internal and external pressures:
SI == ------1... s,
2te 4t e
where:
e == joint efficiency For Maximum Internal Stress:
S == tangential stress
2 2. 2
0, == inside diameter t == thickness of the wall S _ Pi(fo Hi ) ~2Poro
1/ ~ 2- ~

" OJ '" 0 0 - 2t
fo - rj
,,
I
where: t:::: wall thickness Do :::: outside diameter For Maximum External Stress: ~

2 •... ':'./ 2 2 .t< :I


): ~
" For a cylindrical vessel filled with fluid: SIO ~ 2Piri~Pb{rO + rj ) " r. t
ro 2 .-. r,2
I
:<
.
I
r
)1I
'
o
l0lJ Pressure \ ·(·ss,·/ Pressure \',',.,,-1 139

where:
l' 1« 1I\L1': M :\ (Oct. 1998)
St, = maximum internal tangential P, = internal pressure Deterrrune the bursting pressure of steel with diameter of 10 inches and make of
stress Po = external pressure 1/4 111 thick plate. The joint efficiency is at 70% and the tensile strength is 60 ksi.
SIO = maximum external tangential r = outside radius
0
A 4.020 psi C. 2.400 psi
stress r, = inside radius B. 4.002 pSI D. 4,200 psi
PROBLEM 1 (Oct. 2000)
SOLUTION
Find the thickness of a metal needed in the semi-spherical end of cylindrical
vessel 0.70 meter in diameter subjected to an internal pressure of 2.7 N/mm 2
The material is mild carbon steel and tensile stress is 69 N/mm. Using spherical vessel formula for thin wall:
A. 5.2 mm C. 7.53 mm
B. 6.02 mm O. 6.84 mm Considenng the efficiency,

SOLUTION PO,
8
4te
Using spherical vessel formula for thin wall: S 60 ksi = 60,000 psi
Two semi-spherical ends is considered as sphere. P(10)_
2
P = 2.7 N/mm = 2.7 Mpa = 2700 Kpa
2
60,000 = 4(11'4)(0.70)
S = 69 N/mm = 69 Mpa = 69,000 Kpa
P = 4200 psi

S = ~Oi

4t PROBLEM 4 (ME Bd. Oct. 97)


69,000 = .£2~~2) (0.7)_ Determine the safe wall thickness of a 30 inches steel with internal pressure of
4 (tj--- 7.82 Mpa. The yeild stress of material is at 275.48 Mpa. The factor of safety to
use is 2.0.
A. 3/4 in C. 21.6 mm
0.00685 mm = 6.85 m
B. 23.4 cm D. 5/8 111

PROBLEM 2 (Oct. 2000) E1I!DI':D


A cylindrical tank with 10 inches inside diameter contains oxygen gas at 2500
psi. Calculate the required wall thickness in millimeter under stress of 28000 psi. ~(.1~ 1,
A. 10.54 C. 10.24 Using thin-wall formula:
B. 11.34 O. 12.44 8, = PD~ P = 7.82 Mpa
2t
SOLUTION
~ = PO,
FS 2t
For cylindrical pressurized tank: !< ~
275.48 = 7.82@~) 30in

8 =~Oi
2 2t
2t t = 0.8516 in = 21.6 mm

28,000 = ~500~l
PROBLEM 5 (ME Bd. Oct. 97)
2t Determine the bursting steam pressure of a steel shell with diameter of 10 inches
t = 0.446 in (25.4) = 11.34 mm and made of 114 inch thick steel plate. The joint efficiency is at 70% and the
tensile strength is 60 ksi.
A. 4200 psi C 42.8 ksi
B. 105 kSI D 8500 psi
140 Pressure \lessel
Pr c ss urc \Ie sscl 141

Ei3!Iim3I
St = PO
-- ---'-
Steel shell usually spherical: 2t
Based on operating pressure:
POi 180(60)
Considering the efficiency, S 9500 = : - - - - -
4tTj 2t
t =: 0.568"
P(1Q)--
60,000 =: 4(0:25 )(0 70)
Based on pop-out pressure:
P =: 4200 psi
9500 c; 200(60)
2t
PROBLEM 6 (ME Bd. Apr. 98) t = 0_631"
A standard plate thickness of 5/8" (0.625") would be safe Solving for the
Compute the safe wall thickness of a 76.2 cm diameter steel tank. The tank is
bursting pressure ustnq a wall thickness of 5/8" (0.625"):
subjected to 7.33 Mpa pressure and the steel material has Yield stress of 215.4
Mpa. The factor of safety to use is 3. S, =: POi
A. 1 1/2 in C. 4.09 cm 2t
B. 3.89 cm D. 369 cm P(60)
65,000 = 2(-0.625)
SOLUTION
P = 1354 psi

Usually steel tank is cylindrical, ~j~t


PROBLEM 8 (ME Bd. Oct. 95)
Sy P = 7.33Mpa Determine the thickness of a steel air receiver with 30 inches diameter and
POi
pressure load of 120 pSI, design stress of 8000 psi.
FS 2t
A. 1/4 in C. 5/8 in
C 3/8 In D. 1/2 in
215.4 7.33(76.2) !< )1
--'-~-._--
76.2 em
3 2t SOLUTiON

=: 3.89 cm For cylindrical tank:


PO
St = ----,
2t
PROBLEM 7 (ME Bd. Oct. 95)
120(30)
A steel cylindrical air receiver with 5 feet diameter and pressure load of 180 psi, 8000 ---_.

design stress of 9500 psi maximum. The pressure vessel is to be provided with 1 2t
1/2" diameter drain valve Installed at the bottom of the vessel and safety t =: 0.225.
pressure relief valve installed either at the top most or at the side with prop-out
rating of 200 psi. assume a 100% weld joint efficiency. The lap welding tensile Therefore: Use 1/4" standard thickness
strength is 65,000 psi. determine the bursting pressure of this air receiver. (Kent's p. 6-03)
A. 1154 psi C. 1454 psi
B. 1354 psi 0 1254 pSI
PROBLEM 9 (ME Bd. Oct 93)
Ei3!Iim3I A compression ring is to be used at the junction of a conical head and shell.
Determine the required area of the compression ring if the pressure is 50 psi and
0, = 5 It =60 in the stress is 13,750 pSI. Assume an efficiency of the JOint IS 80%.

_.-
142 Pressure Vessel 143
PreSSlUT Vessel

Em!!iirf3 IED!DImI
20mm
'I-I~t Using thin wall cylinder formula:

PD,
k --- >I S
Do 2t
PD,
S = !< )j
2te 140,000 = P(0.50) 500mm
2(0.02)'
13.750 =J'0D;
2t(0.80)
P = 11,200 Kpa 11.20 Mpa
t = 0.00227 D,

Do D, + 2t
Do D, + 2(0.00227 D,) PROBLEM 12
A water tank 10m x 12 m is completely filled with water. Determine the minimum
Do 1.00454 D,
Area rr/4 [( 1.00454 D,2 - D,2 J thickness of the plate if stress is limited to 50 Mpa.
Area = 0.00715 D, in
2 2 A. 11.77 mm C. 13.55 mm
B. 12.66 mm D. 12.44 mm

PROBLEM 10 (ME Bd. Apr. 94) KJe1!ii«ml


A cylindrical tank with 10" inside diameter contains oxygen gas at 2500 psi.
Calculate the required wall thickness in (mm) under stress of 28,000 psi.
~t
A. 11.44 mm C. 11.34 mm
B. 10.6 mm D. 10.3 mm Solving for maximum pressure located at the
bottom of the tank. For cylindrical pressure
SOLUTION vessel
~·~t
P wh=9.81(12) 117.72 Kpa
PD, 1< >I
SI = .-. P = 2500 psi
10 m
2t
S = ~Di
28 000 = ~500J.1..Cl)_
2t
, 2(1)
50.000 = 2~7.72(10)

t = 0.4464 in = 11.34 mm 1< >I 2t


10in
t = 0.011772 m = 11.77 mm

PROBLEM 11 PROBLEM 13
Determine the internal pressure of a cylindrical tank 500 mm internal diameter, A spherical shell of 1.8 m outside diameter and 1.725 m inside diameter contains
20 mm thick and 3 m long if stress is limited to 140 Mpa. helium at a pressure of 10.4 Mpa. Compute the stress in the shell.
A. 1006 Mpa C 1120 Mpa A. 124.08 Mpa C. 96.48 Mpa
B. 10.52 Mpa D. 12.88 Mpa B. 119.06 Mpa D. 88.46 Mpa
Pressure Vessel ]45
]44 Pressure Vessel

§~= PO,
.i·'K:m:mI FS 2t
~20 = __ P (2)
Do -0, 4.5 2 (0.015)
2 P = 14 Mpa
1.8 -1.725
t = ------
= 0.0375 m t~ ~t
2 OJ = 1.725 m t)ROBL~M 16
S = ~~i 10.4 (1.72_51 = 119.60 Mpa Oo=1.8m The cylinder of a hydraulic press is made up of thick walled cylinder having an
4t 4(0.0375) inside diameter of 300 rnrn. It is subjected to an internal pressure of 40 Mpa
Determine the thickness of the cylinder without exceeding a shearing stress of 80
fvlpa.
A. 62.13 mm C. 48.92 mm
PROBLEM 14
B. 5861 mm D. 7240 mm
A 600 mm diameter spherical steel pressure vessel has a uniform 8 mm wall
thickness and an internal gage pressure of 10 Mpa. If the ultimate stress of steel
is 420 Mpa, determine the factor of safety with respect to tensile of failure. EJ'immi
A. 230 C. 2.69
B. 1.89 D. 148 Using thick wall formula: (Lame's Equation)

SOLUTION = l
~ J~+~ -1J
~t~t

P =40 Mpa

= Q~~~? U~~:~~~r~~:~~~ -1J


Solving first the inside diameter:
OJ = Do - 2t t.
0, = 600 - 2(8) = 584 mm = 0.584 m
8mm ~8mm = 010980 m = 10980 mm I( :;
Using the spherical vessel formula: 300 mmn

PROBLEM 17
S = P~j 10(0584) = 18250 Mpa The internal pressure of a 400 mm inside diameter cylindrical tank is 10 Mpa and
4t 4 (0.008)
tank thickness IS 25 mm. Determine the stress developed if joint efficiency is
420
FS ?"'- 2.30
95%.
S 18250 A 8010 Mpa C 86.75 Mpa
B. 84.21 Mpa D. 7842 Mpa

PROBLEM ]5 ~
Determine the largest internal pressure which maybe applied to a cylindrical tank
~~25mm
2 m in diameter and 15 mm wall thickness, if the ultimate stress of steel is 420
Mpa and a factor of safety of 45. PO,
A. 14 Mpa C. 18 Mpa I S 10 Mpa
2tT]
B. 2.6 Mpa D. 22 Mpa
--J.~~ 15 mm 10,000(040)
S ------_._"-----~-
84.210.526 Kpa
SOLUTION p 2(0.025)(0.95)
84.21 Mpa I( :;
400 mmn
P OJ
S
2t 1< )1
2m
146 Pressure Vcssd 147
Pressure Vessel

PROBLEM 18
A spherical tank 15 mm thick has an internal pressure of 5 Mpa. The joint IID!!JmD
efficiency is 96% and stress is limited to 46,875 Kpa. Find the tank internal
radius. Using thick-wall formula:
A. 540 mm
B. 200 mm
C. 270 mm
O. 300 mm
t= Qr iTs! + PI) - 1l
2l\i(St- Pi) J
SOLUTION 0.050 = .0. 30 r ((30,000 + Pi) -1l
2 l~ (30,000 - p;) J
S = ~~ ,~ . l
4111 0.333 = I(30,00~_~l 1J

I' ,,:
I

46 875 = .~ 5,000LCJ,L_
l \i (30,000 - Pi)
, 4(0.015)(096)
0, = 0.540 m = 540 mm
I

I(
I

I I
1333 = II (30~~~.:+:£JllJ
L~ (30,000-Pi )
r, = 540/2 = 270 mm I Di )! ~5mm
Squaring both sides:
000
1.333 = :3
+ p;
PROBLEM 19 3000 -PI
A cylindrical tank has an inside diameter of 5 in and is subjected to an internal 30,000 + P, = 53333.33 - 1.7777P,
pressure of 500 psi. If maximum stress is 1200 psi, determine the required P, = 8400 Kpa = 8.4 Mpa
thickness.
A. 1.0 in C 1.4 in
B. 1.2 In O. 1.6 in PROBLEM 21
A round vertical steel tank has an Inside diameter of 3 m and 6 m height. It
SOLUTION 3
contains gasoline with a density of 750 kg/m If the allowable tensile stress is 25
Mpa, find the minimum thickness required
To check if the problem is thin or thick wall: A 2.65 mm C. 3.65 mm
B. 2.85 mm O. 1.82 mm
S = POI
2t

1200 = .?g~(5)
wa·"'U(m11
2t
Solving for maximum pressure at the bottom of tank.
t = 1.04166 in
tID, = 104166/5 P w h = (750 x 9.81 Ii 000)(6) = 44.145 Kpa
tiD, = 0208> 007
POI
Therefore, use thick-wall formula: S
2t
By Lame's Equation for thick vessel
25.000 = 44·~~@1

t = l l
~ I~: +-~? -1J =~ J~~-ci~: ~ci~f -1] 1395 in
t
2t
= 2.648 x 10. 3 m = 2.648 mm
!'\-{013LEM 22
PROBLEM 20 A cylinder having an internal diameter of 16 in and external diameter of 26 inches
A thick wall cylinder has 50 mm thick and internal die-meter of 300 mrn. If stress IS subjected to 1500 psi external pressure and internal pressure of 9.000 psi.
is limited to 30 Mpa, determine the maximum internal pressure Determine the hoop stress on outer surface of cylinder.
A 8.1 Mpa C 8.3 Mpa A. 8,742.65 psi C. 9.400.62 psi
B 82 Mpa O. 8.4 Mpa B 7,642.85 psi 0 9,471.43 psi
148 Pressure Vessel
noll s & Power Screw J49

SOLUTION

r, = 16/2 = 8 in
ro = 26/2 = 13 in -+1
2 2 2 -+
2Pj r, - Po( r0 + r, )
Sto = --- ----.-
2 2
. 1500 psi
ro - r, -+1
-+
2(9000)(8)2 -1500(13 2 ... 8 2)
SIO ----_ -._---_._-._-
.. -+
13 2 .. 82
s;
~

7642.85 psi
I< >!
16in
k )1
26in

PROBLEM 23 m:mm:tmtJ
A cylindrical tank has a mean diameter of 40 cm and thickness of 10 ern. It is ~ Bolts and Screws are threaded fasteners which are used to hold together
subjected to an external pressure of 140 kq/crn" and maximum internal
machine members which require easy dismantling.
tangential stress is 900 kq/crn". Determine the maximum internal tangential
stress is 900 kq/crn". Determine the maximum internal pressure of the tank. ,. Pitch(p) = is the axial distance between adjacent threads.
2
A. 2947.5 kg/cm C. 1246.5 g/cm
2
B. 34265 kg/cm D. 1942.6 kq/crn"
~. Lead ' !S the axial distance a thread advances in one revolution.

Em!!JirmI (, Minor diameter - IS the smallest diameter of the threads.

r, = 40 - (10/2) = 35 cm ~, Pitch diameter - is the mean of major and minor diameters.


Io = 40 + (10/2) = 45 cm
2 2 2 ........ 140 <. Major diameter - is the outside diameter of the threads and is the nominal
Stl = fJ-,-~r~~~ - 2Poro diameter
r0 2 - r,
Stress area - is the area of an imaginary Circle whose diameter is the mean of
900( 45 2 + 35 2) - 2(140)( 45)2 the pitch and minor diameter.
SI' - --452 ' _ 35-
2 _.-.- t t t 2
1< )1 ..o., +D,'
SI' 2947.5 kq/crn" 40 em Stress area = IT

4. l 2
-_. i
)

Ie. Common types of bolts and screws:

1. machine bolts 5. stove bolt


2. stud bolt 6. cap screw
3. eye bolt 7. set screw
4. U-bolt

I.. Types 01 threads:

UNC(Unified National Course) - for general use, except where other types are
t,Ynrr.mended.
, UNF(IJ'lified National fine), frequently useo in automotive and aircraft work and
Wllt:lf; a line adjustment is requllf:cJ

~.;;
150 130/{ s &. Power Screw 151
Bolts & Power Screw
3. UNEF(Unified National Extra Fine) ~ used in aeronautical equipment and where o ~ nominal diameter
very fine adjustment is required
F, = Initial tension
Common forms of threads:
1. Acme thread
2. Sellers square thread

x
Co Pitch(p) ~IS the axial distance a thread
«': advances in one revolution.
1. Formulas from Vallance: 1
p =
No. of threads per inch
" Sw = C(A r ) 0 4 18
(. Lead (L)
L = p (for single thread)
Iv
(. Fa = C(A) 1418
L = 2p (for double thread)
L = 3p (for triple th read)
" Depth of tap = 1.5 0 (for cast iron)
1.25 0 (for steel) '" Linear velocity = (rotational speed)(Lead) GT
D v = NL
'" Initial torque = 0.2 Fa 0
where: c Lead angle(x)
Sw = permissible working stress, psi Lead
tan x =
Fa = applied load, Ib rrO m
A = stress area, in 2
C = 5,000 (for carbon steel) " Torque applied to turn the screw, T
C = 15,000 (for alloy steel)
o = nominal diameter For square thread:

yv¥~ l1t~ni ~~~~


2. Formulas from Faires:

"

" Fe
Sy(A s )1/ 2
Sd = ~--~------­
6
Sy(A s )3 / 2
= ------.--
7
Depth of Tap
T

T
=

For ACME thread:

= WO m
-~.-~. lcos<1>tanx+f]
------.---------
6 2 cos <1> - f tan x
€.- Depth of tap = 1.5 0 (for cast iron)
= 0 (for steel of wrought iron) where:
Om = mean diameter of screw
, Initial Torque = C 0 F, W = weight or load
f = coefficient of friction of th reads
where: <1J = 14.4
0

Sd = design tensile stress


Fe = tensile load '" Torque required to overcome collar friction, Te
As = stress area
Sy = yield stress r. = fs_~J~_::-S~ = fe W o,
2 2
C = 0.20 for as received
= 0 15 for lubricated where
0, = (Do + 0,)/2 = r + r,
ll
152 Holl s <v, Power Screw 1.5:~
Bolts &. Patne-r .screw
Dc = mean diameter of collar a-ieJ!,ureg,
r 0 = outside radius of collar
r, = inside radius of collar From Machineries Handbook:
fe = coefficient of friction of collar W '" working strength of bolt
2
W '" S, (055d . 0.25d)
(, Total torque to operate the screw, TT W '" 12,000[0.55(2)2 . 0.25(2)]
TT = T +- Te W '" 20,400 Ibs '" 2040 klb

(, Output power of screw(P o )


PROBLEM 3 (ME Bd. Oct. 97)
Po = Weight x linear velocity of screw An eyebolt is lifting a block weighing 350 Ibs. The eyebolt is of SAE C 1040 material
'" W xv with Su '" 67 ksi and Sy = 55 ksi, what is the stress area ( In inches square) of the bolt
if it is under the unified coarse series thread?
C Power input of screw(Pi) MOTOR Power Screw A. 0.1341 C. 0.0991
P, '" 2 IT TT N B. 0.1134 O. 0.1043

C Efficiency of power screwte). Em!!ImD


a. For square thread:
From Faires p. 150:
e Power Output/Power Input Fe = applied load on bolt
tan x(1 - f tan S A 3/2
e Fe = .X __s __
fO
tan x + f + (c_c )(1 ~ f tan x) 6
Om 213
b. For ACME thread: _ (. 6 Fe
A s - 1--
II
tan x(cos o - f sin x) \ SY )
e '" --_._---.._ . _ - - - - - - - - - - - - - - -..-
f 0 .
r~(350) t
tan x cos o + f cos x + (c_c_ )(cos<jl - f sin x) 3
2
Om Stress Area = 0.1134 in
1,55,000 I
PROBLEM 1 (Oct. 1998)
Compute the working strength of 1 In bolt is screwed up tightly in packaged joint when
the allowable stress is 13,000 psi. PROBLEM 4 (ME Bd. Apr. 97)
A. 3,6001bs C. 3,8001bs If the pitch of a screw is 2/9 find the thread per inch.
B. 3,7001bs D. 3,9001bs A. 0.34 C. 54
B 4.5 O. 17

EmI!BtmI
2
E'!!Jrm \<
1 in >\
F '" S(0.55d - 0.25d) '" 13,000[0.55(1)2 - 0.25(1)}

r~
F '" 3,9001bs
Pitch
No of thread per inch
1
PROBLEM 2 (ME Bd. Oct. 97) 2/9 = - . - - - - - -
No. of thread per inch
What is the working strength of a 2 inches bolt which is screwed up tightly in a packed
joint when the allowabl, working stress is 12,000 psi. No. of thread per inch = 45
A. 204 klb C 234 klb
B 224 klb O. 18 klb
]54 Bolts & Pot ncr Screw Boll s &. Power Screw 155
PROBLEM 5 (ME Bd. Exam.) at 1000 rpm and the coefficient of friction between the collar and the pivot surface is
Find the horsepower lost when a collar is loaded with 1000 Ib rotates at 25 rpm, and 0.15.
has a coefficient of friction 0.15. The outside diameter of the collar is 4 in and the A 08 Hp C. 0.5 Hp
inside diameter is 2 in. B. 0.3 Hp D. 1.2 Hp
A. 0.045 hp C. 0.089 hp
B. 0.89 hp D. 0.56 hp SOLUTION

Em!!iiImI T = f W rj
rl = mean radius of collar
ro + rj
T c -- -fcW(r2-~
o +r,) = 015(~OOO)(2+1) = 18.75ft-lb T = fW(-----)
- 2(12) 2
T = 0.15(100)(000981)(O.05CJ..:+-.Q:02?.1 0.00515 KN-m
Solving for the power lost from collar: ~ 2 )
P = 2 IT TN/ 33,000 P = 2nTN
P = 2(n)(18.75)(25) = 008925 h 1000 1
33,000 P
P = 2n(0.00515)(--)(----)
60 0.746
P = 072 Hp

PROBLEM 6 (ME Bd. Apr. 98)


Compute how many 3/8 inch diameter set screws required to transmit 3 Hp at a shaft PROBLEM 8
speed of 1000 rpm. The shaft diameter is 1 inch. Determine the permissible working stress of a UNC bolts that has a stress area of
2
A. 1 1/2 C. 3 0.606 in if material used is carbon steel.
B. 2 D. 1 A. 4055.5 psi C. 4675.5 psi
B. 5244.5 psi D. 4186.5 psi
Em!!iiImI
From Machineries Handbook, 24th Ed. p. 1452
IEm!!imD
H _ DNd 2 / 3 Sw = C (Ar)o 418
p - -,,--
50-
C = 5000 (for carbon steel)

where: D = shaft diameter, in Sw = 5000(0606)°418 = 4055.49 psi


d = set screw diameter, in
N = speed, rpm
PROBLEM 9
1(1 000)d 2 /3 2
The stress area of NC bolt is 0.763 in , if material used is carbon steel, determine the
3
50 applied load on the bolt.
d 0.4383 in A. 3407.141bs C. 4675.5 psi
0.4383 B. 5244.5 psi D. 4186.5 psi
Number of set screws 1.17 (say 2)
3/8
IEm!!imD
Fa = C (Ar)l 418
PROBLEM 7 (ME Bd. Apr. 95)
C = 5,000 (for carbon steel)
What is the frictional Hp acting on a collar loaded with 100 kg weight? The collar has
Fa = 5,000(0763)1"18 = 3407.141bs
an outside diameter of 100 mm and an internal diameter of 40 mm. The collar rotates
156 noll." 'v, Power SCrelL!
Boll.'-; ,"x. Power Screw l~7
PROBLEM 10
A 12 cm x 16 cm air compressor has 5 bolts on cylinder head with yield stress of 440 ~
Mpa. If the bolt stress area is 0,13 inc', determine the maximum pressure inside the
cylinder. v = linear speed In raising the screw
A. 546.71 psi C 742.78 psi v = NL
B. 671.96 psi D. 840.60 psi 8 = N(8 mm x 1/25.4 x 1/12)
N = 3048 rpm

SOLUTION Solvinq for the power input:


«': P, = 2nTN
Sy = 440,000 x (14.7/101325) P, = 211:(0,050)(304.8/60) IBfPm
Sy = 63834.196 psi P, = 1.5959 KW x 1/0.746
Sy(A s )3 / 2 P, = 2.139 Hp
Fe -----
6
F PROBLEM 13
,,-_ -'-'
(63,834.19)(013)3/2
---6- ,,-- A single square thread power screw has a lead of 6 mm and mean diameter of 34
Fe mm If it is used to lift a load of 26 KN and coefficient of friction of thread is 0.15,
Fe = 498.67 Ibs
determine the torque required to turn the screw.
F = 498.67(5) = 249337 Ibs
12cm A. 919 N-m C. 72.6 N-m
Pressure = F/A
B. 65.8 N-m D. 865 N-m
2493,37
Pressure = --,.-------- 671.97 psi
(n:/4)(12/2.54)2
~i-l!wmmI
PROBLEM 11 Solving for the lead angle of screw:
The cylinder head of ammonia compressor has core gasket area of 80 cm 2 and L 6
2
flange pressure of 90 kg/cm . Determine the torque applied on the bolt if nominal tan x = = - - - = 0.05617
n:D m n:(34)
diameter of bolt used is 3/4 inch and there 5 bolts.
A. 47628 in-Ibs
B. 586.28 in-lbs
C, 696.28 in-lbs
0, 666.26 in-lbs Solvinq for the torque required to turn the screw: 16mm
T = WD...'11...I· (tan..x..:+.fL I
E:1!!imD 2 L 1- f tan x J
26(0.034) I 0.05617 \- 0,15
Total initial tension = 90(80) = 7200 kg x 2.205 Ibs = 158761bs T -------1 --..--..-----.-....
2 L1-0.15(0.05617)
Initial tension per bolt = 15876/5 = 3175.2 Ibs

Solving for the initial torque applied per bolt: T = 0.0919 KN-m = 91.90 N-m
T = 0.2 F, D = 0.2(3175.2)(3/4) = 476.28 in-Ibs
PROBLEM 14
An ACME thread power screw that has a mean diameter of 25 mm and pitch of 5 mm
PROBLEM 12 IS used to lift a load of 500 kg. If friction on threads is 0.1, determine the torque

The total torque required to turn the power screw is 50 N-m. If linear speed of screw needed to turn the screw.
is 8 ft/min and lead of 8 mm, determine the Hp input of the power screw. A. 1030 N-m C. 13.10 N-m
A.282Hp C2.14Hp B. 1263 N-m D. 14.10 N-m
B 2.54 Hp D 238 Hp
.....r.J.·~·NII

L 5
tan x ~ 0.0636
«o., n(25)
158 Bolts &. Power Screll' Bolts &. Power Screw 159

T WOrn lcos(j)tanx+f)l
- - - - - _ _._-
-"'-_.- ..
For double square thread: L =0 2P
2 cose-t tan x 2(10)
tan x = - - - - = 0.039788
;c(80x2)
For ACME thread, <jl =0 14.5 0
W =0 500 x 0.00981 = 4.905 KN T = WOrn rJt~nx+!ll

2 L 1 - I tan x
W Om = 4.905 (0.025) =0 0.122625 KN.m
Om =0 2r
T =0 (0.122)[(COS14.50)(O.~636)+0.10J = 0.01030 KN-m W Om =0 80(008 x 2) =0 12.8 KN-m
2 cos 14.5°-0.1 0(0.0636)
T =0 10.30 N-m
T =0 I
(12.80) 0039788 + 0.13 I =0 1.0923 KN-m
2 11-0.13(0.039788)

PROBLEM 15 Solving lor the total torque:


TT =0 T + T,
The torque required to overcome collar friction of a 100 mm mean diameter collar
But: r, =0 0.20 TT
power screw is 50 N-m and collar friction of 0.15. Determine the weight lifted by the
screw. TT =0 T + 0.20TT
A 458.57 kg 0.80TT =0 1.0923
C 487.57 kg
TT = 1.3653 KN-m =0 1365.30 N-m
B. 478.57 kg D. 679.57 kg

SOLUTION
PROBLEM 17
The root diameter of a double square thread power screw is 0.55 in. The screw has a
Torque required to overcome collar friction: pitch of 0.2 in. Determine the major diameter.
Tc =0 !,"-W (r 0 +rJ A. 0.524 in C. 0842 in
2 B. 0.750 in D. 0961 in

Taking the relation between Om and r. SOLUTION


Do +0; 2ro +2r,
Om = -- =0 ---- r0 + r, For double square thread:
2 2
r o+ r, =0 0.100 L =0 2p = 2(0.2) = 0.4
Solving for the weight:
For square thread:
50 = (0.15)JWL~~9_01
Do = 0, + L/2
2 Do = 0.55 + 0.4/2 =0 0.750 in
W =0 6666.667 N x 1/9.81 =0 67957 kg
PROBLEM 18
A power screw consumes 6 Hp in raising a 2800 Ib weight at the rate of 30 ft/min.
PROBLEM 16 Determine the efficiency of the screw.
A double square thread power screw has a mean radius 01 80 mm and a pitch 01 10 A. 12.5% C. 42.42%
mm is use to lift a load of 80 KN. If friction of screw is 0.13 and collar torque is 20% of B. 16.8% D. 66.62%
input torque, determine the Input torque required.
A. 83076 N-m C 1246.30 N-m
B. 83576 N-m 0 136530 N-m Em!!ImD
SOLUTION Solvinq for the horsepower output:

L 'N x vei()c;,ty ,hp


tan x Hp, =0 -33~OOO

nD rn
Bolts & Power Screw 161
160 Bolts &. Power Screw
0.70 == ta~_x[1-0.10(tanx)]
30x2800
Hp., -------.--
tanx+0.10+0
33,000 2x
0.1tanx + 0.07 == tanx - 0.1 Otan
Hp., 2.545 Hp 2x·
0.1tan 0.3tanx + 0.07 == 0

Solving for the efficiency of the screw:


By using quadratic formula:
Po
Screw Efficiency - (-0.3) ± J(-0.3)2~ 4(0.1 0-)W07)
Pi tan x == _.~-- - -
Screw Efficiency == 2.545/6 == 42.42% 2(0.10)
tanx == 0.255
x == tan' 0.255 14.30°

PROBLEM 19
A square thread power screw has a pitch diameter of 1.5" and a lead of 1". Neglecting
collar friction, determine the coefficient of friction for threads if screw efficiency is
63.62%
A. 0.113 C. 0.146
B. 0.121 o
0.151

II:DmimDI
Solving for the lead angle:
L 1
tan x == ..--- == - == 0.2122
nOm n(1.5)

Using the formula of efficiency:


tan x(1 - f tan x)
e == . _ - ------
tan x + f + (fcOc 10 m )(1- f tan x)
Since collar friction is negligible (fc == 0), then the quantitytt, Dc IDm)(1 . ftanx) == 0
0.6362 == 0.2122[1-_fJ.9 212 ..?1l
0.2122 + f + 0
0.2122 + f == 0.333 - 0.070f
f==0.113

PROBLEM 20
A square thread screw has an efficiency of 70% when friction of threads is 0.10 and
collar friction is negligible.
A. 1430° C. 12.43°
B. 1037 0 o 16.45°

SOLUTION

Using efficiency formula'


tanx(1 ftanx)
e == ---- - .. _._~--
tanx I f I (f, 0, IOm)(1-ftanx)
I ():2 1"11/11'11 t ' 'I

2. Total weight of Flywheel(W)


FI.1jU l l W I '/ I fi:{
-
11 Flyvvheel· where
W := Wr + Wah

Wr =- weight of the tlvw-reel rim


W"h - weight of arm an the hub

3. Weight of the flywheel rim(W,)

W r =- Vxw
W, = (rr D b t)(w)

where Om
b =- Width of the flywheel nm
W"I'>
\ =- thickness of flywheel rim
w =- density of llywheel material

b
Om 4. Energy required to punch a metal(E)

E=1/2Ftp

E = 1/2(8," x A)l,
Shaft b
where:
l. Flvwheel a rotating energy reservoir which absorbs energy from a power A -=0 shearing area
source dunnq a portion ef the operattnq cycle and delivcrs that stored energy A =- IT d t p (for Circular hole)
as useful Work our.nq the other portion of the cycle. F _ average force needed to punch a t-ore
S~ =- uittrnate shear stress
I

(. Flywheel applications: " _ thickness of the plate


1
2
3
4
Punch press
Shears
Interna! combustion engines
Compressors
d =- hole o.ar-ieter
D; .,-- mea-t diameter of flywhee!

5. Power needed to punch a hole(P)


tI.
...
d
5 Reciprocating pumps P = ~nergy.

6 Steam engines Time needed to punch a hole

1. Kinetic energy released by the flywheel(KE). 6. Kinetic energy released by the fl'f/'lheel =- Energy needed to punch a
W 2 2 hole
KE = -(v, ··V2 )
2g
where 7. Coefficient of fluctualion(C;)
V' -maximum speoo = It 0 N.
v. = minimum speed = It 0 N2 ~-l,.- v2
C,
W := total weight 01 the tlvwheel v
9 =- acceleration due \0 gra',Ilty =- 9.81 Ill/SCC
v;=.v 1+v-;;. 2(v, -v 2 )
wnere C,
2 v, +v2
I f31 Flll/I'he!'!
j()~
Fllfll'!!t'd
PRO[JLE:Y1 I (ME Bd. Oct 95)
Solvmo for Hl(' hl1, \I V,.clqr1l
A cast Iron flywheel IS rotated at a speed of 1200 rpm and bavmq a mean rcldluc, o!
I W _ 120'.W-1
foot. If the weight of the rim IS 30 Ibs, what IS tile centrifugal force? Use laclG! C - -11 W 12<1181181-97417kg
A 14.8001bs C 7 ml
B 14.860 Ibs D 14 760 IrJS
PROBLEM :1
IIE!!Ii1ElII A press IS used to punch 10 holes per minute of 30 mm diameter hole trorn 25 mrr
thick plate. Determine the power needed to punch a hole If ultimate shear stress is
400 Mpa
Solving fer the tangential speed. A. 2.68 KW C 3.58 KW

v = IT D N = n(1 x 2)(1200,601 B 1.96KW D 096 KW

v = 125 664 tt/scc

E:I!!hir·W+
USing centrifugal force formula
1 min '-'- 60 sec
Wv 2
F. Lei t = time required to punch one hole
gl I = 60/10 c 6 sec
30(125 664)2

F c 14.7241bs
E energy requrred to shear a plate
32.2(1)

E _ 1'2 (S. x A)t"


A _ IT d I,. = rt (0 03)(0 025)

I'ROUU~M 2 (ME Bd. Apr. 94)

A flywheel for a punching press must be capable of furnishing 3400 N-m cl energ:1

dunnq the 25°,) revolution while tile hole IS being punched The flywheel maxumtr­

speed IS 200 rpm and the speed decreases 8.7 c,c our.nq the load stroke The mear

E = 1178 KN-m

P ower --=
0.002356 m'
E c 1'21400000 x 0 002356)(0 0251

Energy 11.78
1 963 KW
25 mmC:
.0= (J
30 mm

Time 6
radius of the run IS 1016 mm diameter and contributes 90~-o of t"le energy

requirements Approximate total weight cf the llywheel 10 be 1.20 times that 01 tile
rim Find the weight of the flywheel.
A 975 kg C I'RC)BIYM 4
B. 652 kg D The kmetrc energy needed to punch a hole IS 5 KJ wt-a: IS the maximum ttuck.iess
W'h
of a hole that can be punch If hole diameter IS 25 mrn ard ultimate shearinq stress of
~·::JX(fi[.1~. plate is 420 Mpa
A 12.61 rn-n C 1741 mm

B 1468m" D 19.62mm

v, 2rrR,N c 20(1016)',200,'60)
V· 21 279 rr-sec
V, 2rr(1 0161(200,'601(1 - 0 0871
Em!!iit.':+
Vi' 19,427 rn.soc
E energy reqUired to shear a plate
W" : 1 /2 (S",>( Al tr_

KE -_. (V t V -> ~) E
2g " E 1'2 IS, lie d I, Itr

Sorv.nq lor the welghl o! lhe r.rn t aserr on e'lergy reqlmer"-'en(:


5 1i2 j420.000!(n: x 0.025 X tl,)l"

t-, - 0.01 /~ll1 'T' 17.41 mm ­


3400lO 90) \"/!~ ,212791= 11] 42TI?J

981 2,'981,1"
tpL:.::.:
W-~ 81151 "'9 .<;-25 rnrn
1 ijl; n 1111'Il( '( ,I PI 1111'/1<',,1 II 7
I'f,UIlIX;V1 r, A. 45 r-im C. 60 mm

During iJ punching process o! 350 kg flywheel the speed vanes from 200 rpm to 180 B 30 mm o 55 rr-rn

rpm with 1 m mean diameter Deterrr.r-e the ktnettc onerqy released by the flywheel

A 364 KJ
B 6.28 KJ
C 451 KJ

0 5.62 KJ

Em!!ii[.g+
Sheared Area
E _ 1!2(S??xA)t
SOLUTION
10 = 1/2 (420000)(A)(0 020)
A c 0.0023809 m 20 mm
v· r: 0 N 1 ell )1200,601 1047 m/sec
v. "0 N2 "(1)1180,601 9.425 m/sec Let
'I- length of side cf a square hole
KE =-- kinetic energy released by Ihe flywheel
A = shear area
_ W , 2
A c 14x) t
Kt: -=- (v.: v.;")

2g
0.0023809 = 4x(0 02)
x c 002976 m = 29.76 mm
KE = 1350xO
--
009811 110
I
7?
4)
'Q 0C'
I,~ 4L.-:)) j

219.81)

KE = 364 KJ PR013LfcM "


A 1 m mean diameter flywheel. SOD kg weight changes Its speed from 220 rom to 200
rpm during shearing process. What average lorce is needed to shear a 30 rnrn th nk
f'r,OHLFM fi pta:e
A plate 200 mm wide and 25 mm thick ,...../Ilh strength cl 410 Mpa IS to be shear along A. 384 Kr-; C 234 KN
Its width During shear.nq process an 800 nom mean diameter flywheel changes its B. 653 Kr-. o 1082 KN
spe od Iro-n 200 rpm to 180 rpm Deter-ru-e the wc.qht 01 flywheel to be usc
A. 326584 kg C 3821.72 kg

B 3452.71 kg 0 3845.97 kg
EI::I!!im'I3I
v, ~ it (1 )(220/60) -::: 11 519 m/sec
EI!!:im'mI v:= :1(1)(200/60) 10472 m/sec

W = 500 x 0.00981 = 4905 KN

V, ,,0 N, .'10.8)1200/60) 8377 m'snc W , 2, 4905 [ 1 . 1,1 0.427 J~]

v: "ON '10.8)1180/60) 75398 rr/sec KE = (VI"" v'),1 = - \ 1.5191"


2g , 2(981)

Shear Area
E = 1,'2 IS., .<1.,;1): I KE = 5756 KJ
E - 1,'2 (410,000)10 2 x 0 025)10 025) E = 1 F I

E - 25625 KJ 2

~25mm 5.756 = 1,2 (FIIO 03)

KE w ('I.', v, ') c c 383.75 Kr-;

29 ­
200mm
If' .
25625 -= ". [1'8 37lt - (7.5398) PROBLEM Y
219811 .
The enercv required to puncf a ho.e IS 3 KJ frorr a II/wheel mean diameter of 800
W - 37729 KN x 1:0 00981 = 38"597 kJ mrn that slows Clown trom 33 rps to 3 rps during ouicunq. If Ii\'ei;)ht ,=,1 arm and hub
account 10"" of rm weight, cetetrrune the rim Vie 9hl
A 421 08 kg C 48268 kg

f'ROHLEM 7 B "5740 kg 0 41668 kg

The energy requ,'red 10 punch a square role from 20 mil thick plate IS 10 r(J It the
uiti'late strength 01 pla:e is 420 Mpa. detorrn.ne the maxnr-um sides of square that
can be punchec.
I fl,'-<, Fl!JlI'llcd 1 (j~)
FIL/Lt'lU'd
A 332-1~rnPl C 286.76 mm
I.E!!m:D B 24261 rrrn D. 29878 mm

v· ,1(0.80il3 3), 8293 m/scc

v n(O 80)(3) = 75398 rrsec


EE!!IiS3
Sorvmo for the weight of the Hywheal: b = 51
W ? 2 W, = (n D b II W
KE = -(v, -v,)
2g ­ 450 = ;r(2 x 045)[51)11)(7200) Dc = 900 mm
1 = 006649 m = 6649 mm
VV ~ ?
3 = .. --;[(8293)"-(7539811

2(9.81)

b 51
W 44358 KN x 1 '000981 503 147 kg b 5(66491
b 33245 mm
Solving for the nrn weight: k ,I
W = VV + VV d ,
VV ~ vv,
+ 10 ooVV,
503.147 = W, + 010W
PHOBLlc:vI 12
W = 4574 kg
The power required 10 shear a plate 50 mm thick for 10 seconds IS 5 KW. If ultimate
strength of plate material IS 420 Mpa How vade should the plate be?
A 8042 mm C 90?8 mrn
PHOBLE:vI 10 B 85 65 mm D 95 23 mm
A flywheel has a total weight of 500 kg and the weight of arm and hub IS 8°'0 of total
weight. It has a mean diameter of 1 m and width of 300 mm Deterrruno the thickness
of the fiy\vheel if density of material IS 7000 kgl"/
A. 60.65 mm
B. 6548 mm
C 6972 mm
D 75 42 mel
.,.'S!imD
Shear Area

E Power x time
E 51101 = 50 KJ
I.E!!m:D
E = 1·2 (S" x AI t
Solving for the nrn werqht 50 = 1'2 1420.000)[A)10 05)
A = 00047619 m w
VV ::;- W r -.... W,,'I

\V = W r + 8°'"W
A = wt
500 = W. + 008[500)
Dc = 1 m 0.0047619 = w(0.051
VI = 460 kg w = 0095238 m
IN ----' 95,23 m-e
Solving for the rim thtcxnes s .
Wc(oDbl)w
l'EOlJLE:Yl I:,
460 = c (1110 3)(11[70001
A 900 m-n mean diameter flywneel has a 'N,d:h Cit 350 m>, 7S rr.m thick <.nd dens ty
1 = 0.06972 m
of 7100 kq.rn''. \Jeg'ectInQ :he weight of arm nne hub, fl~d the cr-c-qv relcasoo if .ts
I ~_ 69 72 rnrr
k )1 speed cranges from 280 rpm to 250 rpm
A. 7.89 KJ C 548 KJ
B. 9 30 KJ D. 6 71 KJ
PROBLEM I I
A flywr.eel has a rim weight of 450 kC~ and mean rartus of 450 mm If rim Width IS 5
times the rim thickness ana materia! density IS 720C kq/m'. determine the Width of
Ilywheel
170 FIIJ/I"! II '('1 Fi'l" ,Ii "f'! 17\

~ A. 190 66 Ibs C 19866 Ibs


D 20077 Ibs
B. 195661bs
Solving for welghi of rim
W. = J( 0 b t w
W, = ,,(09)(0.35)(0.075117100) c 52696 kg
Em!!ImmI
V; v2
w = W,+W.I'r. Dc;:: 900 rnm v.;
W = W, + 0 = 526 96Kg 2

v, = ,,(09)(280/60) = 13195 m/sec 80 ~


Y1 - v;;

V2 = ;t10 91(250/601 = 11,781 m/sec 2

v, + Vi 160

"-
USing kinetic energy torrnura V2 160 - V,

W ' 2(V1 V2)

KE = 29(V," - v : IE >1 Cr
VI - V2
b = 350 mm
W = 526 96 x 0 00981 = 5 169 KN
009
21v, (160
-----
v,ll
'5169' 160

KE ,- \ .J [113195)' -111 781)!J 930 KJ


2(981) v- = 83.6 ft.sec

v' ~ 160 - 836 c 764fusec

W ::.' 2
PROIlLEM 14 KE = --(v, -v 2 )

2q

A flywheel welghmg 1000 kg has a rao.us of gyration of 1.5 m The normal operaunq
W I I
speed IS 160 rpm and ecett.ccn: of Huctuauor, IS 0.08, determine the energy released 3500 =--l(83.6)' 176.4)2 J

by the flywheel 2(322)

A 31.70 KJ C 4170 KJ W _c 19566 Ibs


B 3670 KJ D 4670 KJ

IIE!!'.'iiI:D
v- ,,(15 x 2)(160/601 25 132 rrvsec
2(v 1 - V?)
c
V 1 v:
008

2(25132 v,l
25 132 + VI'

25132 + v- = 628318 25v_


V; = 23 20 rnsec
W = 1000 kg (0 009811 981 KN
W, 2
KE IV, - - v" - )
2~ c

9_8 1
KE 1 ) 11251321' (232/] 4670 KJ
2(9.81) ,

1'!,OElLEM 1f>

A 5 11 mean diameter llywheel has to absorb 3500 ft-Ib of energy and maintain a

coettrcrent 01 uuctuanc- of 0.09 11 mean speed IS 80 n/soc. find the weight of

flywheel.

, ­
I -"',
.'-iJHIIl!J In
1. Stress of coil spring(S,)

12 Spring S, ~ 8KF~m

2 Stress factor(K)
ltd

K ~ 4C-1 + O~

4C-4 C

~ 3 Spring Index (e)


C ~ Om
Uses of spring: d

t
Where
Om = D, - d y,
1 To absorb energy or shock loads, as In automobile shock absorbers
2. To mamlam contact between machine members. as In valves and clutches

CL~
3 To act as a source of energy. as In clocks
Om = D: + d
4 To serve as measuring device, as In spring scales
4 Deflection(y) SL
3
Types of springs: BFC n
y ~

Gd
Om

a Helical compression spring where:

b Helical tenson F = axtal load

c. Torsion D·'1 = mean diameter

d. Spira!
e. Leaf spring
d = wire diameter

n = no. of active or effective coils


:+- d
I
Dc> )!
G =- modulus of rigidity

Materials used in spring:


5. Deflection at solid force(y,)
a Oil-tempered spring wire y. ~ Free length - Solid length
b. mUSIC wire
c. hard drawn spring wire

T_
d carbor steei 6 Spring rate(K)

e. chrome-vanadium steel K ~ Fly ~ F, I Y1 F,/ Y2 constant

f. chrome-silicon steel
9 stainless steel
F2 -F,
K
Y2 - Y,

t
Tabu!ated Data of springs:

Types of coil end 7 Impact load on spring:

Actual no, of coil Solid Length Free Length


Plam N (n ./ 11d np + d
GrourJd N nd np
Squared n -+- 2 in ~ F
3) d np + 3d W(h+y) ~ -y
Squared & Gro.mc n • 2 (n -. 2\ d np + 2d 2
where
F = maximum force acting on the spnrr;
y dcllecuon on spring
17,1 ,'-,/)) II If/ IT)

W ::- weight of the object t -= tfucxncas of plates


h ~ height of the object n,] = no of gradualed leaves
nl no of full length leaves
y = dallecuon of spring

PHOIlLEM I (Oct, 1999)


8 For series connected spring: A body weighing 1000 Ibs Ieus from a height of 6 in and strikes a 2000 lb/m spring
The deformation of the spring IS
A 2 C 3
Y == total elongation B. 4 D 5
SOLUTION
Y :;: Yj + Y2 + Y3
USing the formula of Impact load on spring'
y F, F
2 +---
3 F K1 F ~ 2000 lb/m tV) = 2000 V Ibs
--+---
K K2 K3 (2000 V)
1000 (6 + V) ~ " . V
2
K2
F total load ~ F, ~ F, ~ Fe 6 + Y == l
l-V-6~0
K3
By factoring.
(V - 3) tV + 2) ~ 0
9 For parallel connected spring:
V ~ 3 In: V = ,2 In (neglected)
F Therefore' y = 3 In
Y = total elongation
Y == Y1 == Y2 == YJ
*_ ""r'--'"

Pl{OBLF:M 2 (ME ac. Oct, 97)


y ~ £L~5-~F3 Compute the defl€ctlon of an 18 COils helical spring having a load of 100 kg The
K K2 K3 modulus 01 ciasncity in shear of spring IS 96.62 Gpa. 00 of 9256 cm and With Wife
K, K2 K3 diameter of 9.525 mm. The spnng IS squared and ground ends
A. 9 cm C. 11 em
F == F1 + F2 + F3
B 101 cm °
14cm
If the springs are of the same material
F, ~ F, ~ F 3 = F/3
m:m:mD
F
SolVing for Call mean diameter
11 Leaf springs:
0" ~ 0" d
0", ~ 9256,0.9525 c 8.3035 cm
18FL
8, = ,

2
--"~--

C = ~.. ~ .8.::l0 35 ~ 8.717


bt (2n g + 3n, ) d 0.9525

For square and ground ends


12FL3 Actual no of coils = n + 2
v
bt 3 (2n g + 3n,) Where n = no. of active coils
I
9.525 mm ~
where 18 = n + 2
n ::::: 16 coils I.
I I
0, )
r-I
I
51 ~ flexural stress I 0<-- I
F load at the supports Solvmq for the deflection' \ 9.256 em )1
Ie ,
L distance of force to produce maximum moments
b width of plates
lill ,'-,J> / 1/ I [I t77

BFC'n 8(IUOxO 00981,(8 717}'(161 k spring constant


y F
G~J 96 62x10"I0 0095251 k -
Y K, = 0.4 kg/mm
y 00903, m -= 9.037 em
F
Y =
FH()l--H,Ei\'i :1 (ME Bd. Oct. 97) k
K2 = 0.64 kg/mm
Compute the maximum ceuecto-i of a 20 coils helical sprr-q hEWing a load of 75 kqs. Y = total deflection
The spring is a square/ground ends w.th modulus of elasticity In shear of 79 84 Gpa. y = Yl+y~,+y:,,,::, ~- F, F3
ou.stde diameter of 101 6 rnrn. wtre drameter of 9.525 mm K, K2 K:J
A 140.7 rnm C 15 i7 mm
KJ = 0.64 kg/mm
1(1(1 11111 11111
B 112 7 rnrn D 126 7 rum
+ + = 5625 mm
Y
114(] 11,(,4 IU,4
IEm!!Im3 100 kg

D D rj
} +75 kg l'I,OULEM 5 (ME Bd, Apr, 96)
A high alloy spring aavmo squared and ground ends a'ld has a total of 16 Calls and
modulus of elashcity In shear of 85 Gpa Compute the Wahl factor The spnng

~
D, 101 6 - 9 525 92075 mm outside diameter IS 9 66 em Wire diameter IS 0,65 cm
A, 1058 C. 1.10

For square Ci'ld g:ound ends B 1 185 D 1 2

Actual no. of uctve coils = n + 2


?O ::: n +- 2
n = 18 co.ls (acuve) E'I!!iiI':.mI
I

C - Spring index
9.525 mm -7l i<­
0111 92075
,I l 0, f
C I Solvinq for spring Index' I
9,67 I( )
d 9,525 I
101.6 mm Dr" mean diameter 0.65 em -7l
D,,, = D, - d ~ 9 66 - 0 65 I r-I
I I 0, ) I

Solving for the deflection: D,- = 901 em I I( I

C = D,Jd = 901065 = 13,86 I 9.66 em


)1
I, ,
8FC"n 8175xO 00981)19671 3(18)
y c =- 012597 m 125.97 rr-m 4C 1 0,615
Gd 79 84x1 0 610009525) Whaal Facor c:­
4C4 C

4(1386)-1 o 61 5 1 1023

Whaal Factor =
I'J{OULEM 4 (ME Bd, Oct 97) 4(n86) 4 13 86

A three extenSion call springs are hooked In scncs tl..·at support a single weight of 100
kg The first spring ,'S rated at 0 400 kq.r-rm and the other 2 lower springs is rated at
O 64 kq/mrn Compute the total (jeflectlon. I'1,OULlcM (i (ME Bd, Oct 95)
A 563 mm C. 156 mrn A COil spring With 5 em outsde diameter is required to work under the load of 190 N
B 268 mm D. 250 mm The Wire diameter IS 5 mm. the spnng IS to have 6 act ve coils and the ends are to
be closed and ground. Detsrrnme the total number of Calls The modulus of rigidity IS
80 Gpa and the mean radius IS to be 23 mm. With 7 mm pitch of the SPWlg_
IIE:!!i:lm A 6.5 COils
B 8 5 Calls
C. 7,5 ous

0 9 5 Calls

For sor-es connected spring.

F = F, = F F, = 100 kg

17R
Sprilln l ,'q
IIE!!immI I"" lIILEM 'l
A spnnq has a rate 01 50 Ib with a spring Index of 8 If stress Induced IS 90 000 ~::;;'
For square and ground ends
determine the wire diameter.
A. 0058 In C 0452 In
Total no of coils = n + 2 .:.;. 6 + 2 ~ 8 coils or 8,5 calls B. 0.828 In D. 01157 In

PHOHLF:M 7 (ME Bd. Oct. 95) Em!IimI


A helical spring having square and ground ends a total of 18 cons and its matenal has
Solving for K'
modulus of elasticity In shear of 78,910 Gpa. If the spring has an outside diameter of
4C-1 0_615 418) 1 0615

1042 em and wire diameter of a 625 em, compute the maximum deflection that can K 0

be produced In the spring due to a load of 50 kgs 4C-4 C 4(81 4 8


A. 302 mm B 342 mm K 0 1184
C 490 mm D 322 mm
CoD,. d

IIE!!immI 8 c D, d

D", 8 d

Dm 0 D, - dolO 42 - 0625
0"1 = 9795 em USing stress formula
C 0D";d 9 795/0 625
0 15672 0 S 8KFD",

'. nd'

For square and ground ends


Actual no. of coils = n + 2 8(1 184)(50)(8d)
90.000
18=11+2 :-Ed J
n = 18 - 2 = 16 active calls d 0 0 1157 In
Jn
8FC
y 1
Gd O.625cm ~ PHOllLI::M 10
I
8(50 xO 00981)(15 672)3(16) I I D, )r-I Determine the maximum shearing stress of a helical spring composed ot 20 turns of
y -­ ---­ I I( I 20 rnrn diameter wire on a mean radius of 80 rnrn when the spring IS supporting a
(7891x 10 6 )(000625) I (
10.42 em )i, load 012 KN Go 83,000 Gpa
y 0490 m 0 490 mm A 10820 Mpa C 120.60 Mpa
B 9862 Mpa D 8868 Mpa

1
['HOBLF:M 8
IE!!imD
~2KN
A extension coli spnng IS to be elonqate 5 In under a load of 50 lb. What IS u-e spring
rate?
A s Jb-mn C 151b/mln 8KFD",
o s
B 10 lb/mm 20lb/mm r::d3

Em!iiriD
C D,,;d c (80 x 2)/20 _ 8 -~
4C1 0.615
K ­
4C 4 C
Spnnq Rate F.y
4(8) 1 0.615
K 1 184

Spnng Rate 5015 0 10 lb.m!n 4(8) 4 8 I

8(1184112)(008x2) 20 mm ~
s "1002)' --­
1 __ I
I 1 D, )1 I
1 I( I
s 120 603 Kpa - 120.603 Mpa
i( Do ) ,I
J so
,t..;pr-III!/ tHJ
l'kOULEM I 1
A helical spring IS made by wrapping steel wire 20 mm diameter around a forming
PI'OfJ\X:vJ 1:1
A spring sustam 200 H-Ib o! energy wrtf deflection of 3 tn Assume that the coil
cylinder 150 mm ITl diameter. Compute the elonqauon of the spnng without
diameter IS 7 times the wire diameter and allowable stress of 100,000 pSI. deterrrune
exceeding a shearing stress of 140 Mpa If It IS composed of 18 turns. Let G = 83,000
Mpa the wire diameter.
A. 0.416 In C.05681n
A. 9642 mm C 121 36 mm
B. 0 321 In D. 0672 In
B 1006 mm 0 89 62 mm

SOLUTION Em!!hit.g+
200
F = 800lbs
3 12
D». c 0, + d = 150 + 20 " 170 mm Om 7d
C ~ 7
C = D,,!d = 170/20 = 8.5 d d
4C1 0.615 4(7) ~ 1
K 4C ~1 0.615 0615
4C 4 C K = = 1 2128
4C 4 C 4(7) ~ 4 I
4(8.5) . 1 0.615
K 1.172 8KFD r ,

~F
4(8.5) 4 8.5 Sc ~
3
rtd
s ._. 8KFD Il1 }
8(12128)(800)(7d)
3
r: d 100,000

-~
J
nd
8(1.1721(F)(0 1701
140,000 ~ d 0 0416 in
"(0 02)3
F = 2207 KN
3 P](OBLEM 14
8FC n 8 (2.207) (8.5) l (18)
y A weight of 100 Ibs stru.es a COil from a height of 18 Inches and deflects the spring of
Gd (83,000.000) (0.02) I
6 Inches Find the average force acting on the spring.
y 11757 mm 20 mm -?>lI _ A. 600lb C. 800lb
I I 150 mm )1 I B 700 Ib D. 900 Ib
I IE I
I
\ , 0
0 .I
~ I
IIE!!ii1:mI
PHORLEY! 12
It IS found that a load of 50 Ibs on extension COil spnnq deflects 8 5 In What load will Tall
the spring deflects 25 In?
A 10 64 Ib
B. 12 48 Ib
C 13 48 Ib
0 14 70 Ib
USing the formula of Impact load on
spring.
18;n
4-i·,,··it·H' Wlh + y)
F
~y

2
The spring rate of spring is constant:

K, K:,
100(18 + 61 = ~ (6)
2

FI / y, F2 / Y:I F 800 Ibs

50'85 Fe' '2 5

F- 14 70 tbs
IR2
.'i[!1 ifill u·n
PHOBLEM 15 A 27 60 rnm C 3250 rnm
Three corl spring are hooked In senes and support a w8rglll 01 70 kg One spring has B. 2980 rnrn D 3460 mm
a spring rate of 0,209 kg/min and the other two have sprinq rates of 0.643 kq/mm.
Find the deflection
A 346.71 mm C 55265 mm EI:'I!!iil':.lD
B 389.30 mm

~--r.TIIljit.] ~.
D. 416.58 mm
D" ~ D, 'd
D", = 117 ' 13
OTT - 104 mm
1
C :::: 0",1 d
y = lola] defleclion C = 104/13
y = y + y? + v> C " 8
8Fc ln
y = I
For series connection of spring.
Gd 13 mm~
F, = F i · = F s = 70 kg I
K FlY I -,
K1 = 0.209 kg/mm 8(1 11(8)3(75) I I
Y = F/K Y = 6)(0.013) IE Do = 117 mm ))
(80xl0
y F; K· + F~/K;: + F:,iK J Y = 003249 m
K2 = 0.643 kg/mm
y 700 209 + 70/0643 + 70/0 643 y _ 32.49 mm
y 55265 mm
PHOHLE:vI 1H
KJ = 0.643 kg/mm A concentric helical spnng IS use to support a load of 90 KN. The Inner spnng has a
rate of 495 8 KN/m and outside spring IS 126.5 KN/m If initially the Inner spring IS 25
mm shorter than the outer spring. find the percent load car-reo by Inner sprrnq
PROBLEM 16 70 kg A. 3465"" C. 6825""
B 5586"" D. 768P"

1
Four compression coil spring In parallel support
a load of 360 kg. Each spring has a gradient of 0.717 kg/mm. Find the deflection.
A 125.52 mm C 138.52 mm SOLUTION

IIE!!1:mmI
B. 132.52 mm D. 145.52 mm

v,
K
y, + 0025
=
Fly
F, - v. K, c 495 8 Y
I
Ty,
2s mm
F=~KN YI

:~ :: :~ :~ F, 126,5 YL
~-
For parallel springs'
y, = y = YJ = y.,
i~~ i=
!- i :!- I !"~
F, + F L :.:: 90

F1 F2 FJ F4 495.8 v, t 1265 YL = 90
F, 4958y + 126,5(y, + 0025) = 90
F2 = F', = F, = 360/4 90 kg
495 Sv- t 126 5 v: + 3 1625 c 90
y, F, i k, ::: 9010 717 +360 kg v, = 01395 m

y, F. = 4958(0 13951 = 69 185 KN


125,52 mm Y?::: YJ y, = y
69185
'J" Load Carried =: 76 8r,0

PROBLEM 17 90
A force of 1 1 KN IS acting on a 75 active coils With wire diameter of 13 mm. The
outside diameter of corlrs 117 rum and G ::: 80 GN/m 2 Find the coil deflection. PROl3LEM I~)

How long a wire IS needed 10 make a helical spring havmq a mean diameter of 1 Inch
If there are 8 active coils?
Sprlllfj lR5
Ik-\
4C 1 0.615 4(8) 1 o 615 1 184
A 25 13 In C 30211n K
B 2665 In D. 3234 In 4C ,1 C 4(8)4 8
S stress at solid length
IIiE!!DIiD S -
8KFD r·)
rrrJ
L wire length
8(1184)(15)(010)
S, c 2.315.544Kpa = 2.315S4Mpa
L Circumference x No of calls ,,0.0125) 3

L = cD(n) '11)(8)- 2513,n 1>J«()13Lt:~l 22


A squared and ground ends spring has a pitch of 20 111m wire diameter of 12.5 mm.
If there are 12 actual number of coils. lind the deflection when the spring IS
compressed to Its solid length
FI<OBLEM 20 A, 78 mm C 77 mm
A 008" diameter sprnq has a length of 20 in if density of spring is 0 282 Ib,',ln:; B. 75 mm o 79mm
determine the mass of spring
A. 00395 Ib C 00485 Ib IIiE!!DIiD
B. 0.02831b D. 0.06861b

~'it.]:. For square and ground ends


Actual no or coils = n + 2
V : .: Volume of spring where: n = no. of effective coil
V c (,/4 de) L = 1'/4 (0 081"](20) o 10053 In" 12 = n+2
n = 10 calls
1 20 mm

Solvmq for mass


w mN Free Length np ·t 2d
m = V w (0.10053)(0282) o 0283 Ib Free Length 10(20) + 2(125(
Free Length 225 mm

Solid Length c_ (n + 2)d


PROBLEM 21
Solid Length = (10 + 2)(12 5) = 150 mm
A square and ground ends spring has a free length 01 250 rnm. There are 10 active
y. = FL - SL c 225 - 150 = 75 mm
calls with Wife diameter of 12.5 mm If the spring rate is 150 KN,irn and mean
diameter IS 100 rnm. determine the solid stress.
A 2.3155-\ Mpa C 7.8427 Mpa
l'I~CJLlLI-:M 2:;
B 7.6548 Mpa D. 8.432.9 Mpa
A spring with plain ends has 15 active calls. diameter or 6 mm and Pll!,)) of 10 mm If
spring rate IS 100 KN/m. determine the solid force
A. 4 KN C 6 KN
~iit·W' B 5 KN D. 7 KN

For square and ground ends:


Solid Length = (n + 2)d ~ (10 + 2)(12 5) 150 mm IIE!!ImD
y, = solid length deflection
For plain end type of spring'
y = Free Length SOlid length 250 - 150 100 mm
Solid Length = (n + l)d
F, ~ force at solid length = k Y'>
SOlid Length = (15 + 1)(6)
F,. ~ 150(0100) = 15 KN
SOlid Length =- 96 mm
C = D-! d = 100/12.5 = 8
Free Length = np + d = 15(tO) + 6
IHG I ,~7
Le( I/o..,
156 rum

13
v. = FL, SL = 156, 96 60 mrn
F, = kyo = 100(00601 = 6KN
(3.E3Clr
PRO[JLE\1 24
A spring has a spring rate of 30 KN.im. If wire dIameter IS 10 rnrn with mean diameter
of 70 rnrn. determine the number of active cors G --= 80 G~~/m2.

A. 654 C. 842 (.. Gears - are machine elements that transmit motion by means of successively
B. 782 D 972 engaging teeth

Em!!ImD
~
8C'n
y = I. Addendum height of tooth above pitch Circle: or the distance between the pitch
Gd circle and the top of the tooth
C = D, / d 70/10 7
3 2 Addendum Circle the circle that bounds the outer ends 01 the teeth
v =
8C n
F Gd
Arc of action arc of the pitch Circle through which a tooth travels from the first
8(7)'J n pornt of contact With the mating tooth to the prtcn POint
1/30
180xH/'J(O 0101
4. Arc 01 approach - are of the Circle through which a tooth travels from the point of
n : : : 9 72 corls contact Wittl the mating tooth to the pitch POint

;-l. AXial plane - In a pair of gears It IS the plane trial contains the two axes. In a
PH OLlLEM 2" single gear, It may be any plane containing the a xts and the giver, pomt
A spring has a diameter 0125 mm and 1,2 active coils if a load of 10 KN IS apoheo it
deflects 75 mm Determine the mean diameter of the spring If G = 80 Glv.m". b, Arc of recess arc of the pitch Circle through wtuch a tooth travels from Its
A 12465mm C 134.65mm contact with the mating tooth at the prtch pomt to the point where tho contact
B 129.65 rum D 14065 rnrc ceases.

IE!!immI 7. Backlash the amount by w-uch trc wrrith of ttle tooth space exceeds the

T
75 m
IF = 10 KN
thickness of the engaging tooth on the prtcr, circles.

y
8C'n
GeJ 1 •
8. Base Circle the circle from which an Involute tooth ',$ generated or developed.

q. Base hehx angle - the angle, at the base cylinder If an mvolute gear. that the
8(10IiC)J(12)
0075 = tooth makes With the gear axis
80xl0"10 025)
C 5.386 10. Base pitch In an Involute gear It IS the pitch on the base Circle or along UlC lmc
of actron
C D"
o I
25 mm~ I I. Non-tnt hase pitch - IS the base pitch In the normal plane
D, ,
5386
25 ,
,
-,, 1 Z. AXlat base pitch IS the base pitch tn the axtal plane
, ,
0 :=. 13465 mm
11
k Do ) I,
I J. Center distance the distance between the parallel axes 01 spur gears and
parflltpl hehcal gears. or between tt-o crossed axes of helical gears and worm
IHH (;('111 .'>
(;('(1 ,-,.., lH:J
gears.

14. Central plane ~ In a worm gear this IS a plane perpendicular to tt-e gear aXIS
:12. Fd,lel curve HIe concave portion of the tooth profrle where It JOins the bottom of
and contains the common perpendicular of the gear and thp worm axrs the tooth space The approximate radius of ttus curve IS called the fillet radius

:j I. Flank of tooth - that surface which IS between the Pitch eucre and the bottom
1:1. Chordal Thickness length of the chord sub tended by the Circular thickness
arc land

:~-l-. Helix angle - the angle that a helical gear tooth makes the gear axis
] h. Chordal addendum the height from the top of the tooth to the chord subtending
the circular-thickness arc
3:1. Internal diameter - the diameter of a Circle comcrd.nq With the tops of the teeth
on an Internal gear.
17. Circular pitch - length of the arc 01 the pitch Circle be/ween the centers or other
corresponding POints of the adjacent teeth
:H-j. Internal gear a gear With teeth on the Inner cylindrical' sur-nee
I ~ Normal circular pitch IS the Circular pitch In the normal plane
J7. Involute - the curved formed by the path of a pomt on a snalghl line called n18
generatrlx, as It rolls along a convex base curve
1q Circular thickness the lenglh of the arc between UW two sides of a gear tooth.
en the pitch Circles unless otherwise specified
:1.'-;. Top land IS the top surface of the tooth
~() Clearance - tile amount by which the dedendum exceeds the addendum of the
:~q. Bottom land - IS the surface of the gear between the fillets of adjacorit teeth
rnatmp tooth.
:2 J Cer-t-ai dlameler - the smallest diameter on a gear tooth With which the mating
gear makes -l-O. Lead - the distance a helical gear or worm would thread along Its (DOS one
revotuton of it were tree to move axially
')'1 Cor-tact ratio the ratio of the arc of action to the circular pitch
4- 1 t.me of action the path of contact In Involute gears It IS a strarqht line passing
through the pitch POint and the tangent to the base cuc.cs.
:l:J. Cyclo«l the curved formed by the path of d pomt on a Circle as II rolls along a
stra-qturne
·12 Module IS the ratio of pitch ciar-ieter In millimeter to the numoer of teeth mm
:l.l, Dedendum - the depth of tooth space below the pitch Circle or the radial
1:). Outside diameter - the diameter olthe outside Circle
dimensron between the pitch Circle and the bottom of the tooth space

~.). Drarnetral pitch - the ratio of the number of teeth to the number of millimeters of ·1-I. Pitch the distance between Similar, equally spaced tooth surfaces. In a given
pitch oiarneter. direction and alonq a given curve or line.

l(;. Normal drarnetral prtch - IS the dtametral pitch calculated In the normal plane --1::; Pitch diameter th8 diameter of the pitch Circle
and IS equal to the pitch divded by the cosine of hehx angle
·1 t>. Pitch Circle - a Circle the radius of which is equal to the distance from the gear
L '{. Effective face Width - that portion of the face Width that actually COmes Into axrs to the pitch pomt
contact wtlh matmq teeth as occasronalty one member o~ a pair of gears may
have a greater face Width than the other 4-7. Pressure angle the angle between the tooth profile and a radical line at Its
Pilch po.n: In Involute teeth the angle between the line of action and the line
:2g . Efucrencv - ttlP actual torque ratio ct a gear set divided by Its gear ratio tangent to the pitch Circle

-+~. Roll angle the angle subtended at the center of the base Circle from teeth
29. External gear a gear With teeth on the outer cylindrical surface.
onqm uf an Involute to the pont of tangency of the generatnx from any pomt on
the ~dme Involute.
:~() Face 01 tooth - that surface of the tooth which IS between the pitch eucre 10 \'le
top of the tooth
4~) Tip rl ' lle, 1 an arbitrary modrrc.uon of d tuoth profile whereby a small amount 01
:~ 1 Face Width the length of the teeth In c))(1<11 plane mdtelldlls rernoved near the trp of the qear tooth.
]90 (;('(n'·;
Gears \9 l
50. Tooth thickness - the width of tooth measured alonq the pitch circle. 5. Module
5]. Tooth space - the space between the teeth measured along the pitch circle. M module
52. Whole depth - the total depth of a tooth space, equal to addendum plus M
o M
25.4
dedendum, also equal to working depth plus clearance. T OP
where:

D '" pitch diameter, mm

~
6. When two gears turning in opposite direction:

C '" center distance

. ­
7. When two gears turning in the same direction..
N2
1. Speed and diameter relation:

0 1 Ni'=: D2 Nz C ° 2.;.D
2 1

where:
2. Speed and no. of teeth relation: D '" pitch diameter
T '" number of teeth
T1N, = TzNz N '" speed
T2

3. Pc = circular pitch

8. Pitch line velocity, V


V=TTON c

C = center distance
9. Dynamic forces on meshing gears:
T 1 = no. of teeth of pinion

T2 = no. of teeth of gear

A. Power transmitted
4. DP = diametral pitch
p= 21tTN, 'r0N

where:
T '" torque, KN-m
N '" speed, rps

where: HP _ 2rrTN

- 33,000,HP

D = pitch diameter, in
T = no. of teeth where:
T '" torque, ft-Ibs
N = speed, rpm
Gears 193
192 Gears
p
1. Pn == 2. tan$n == tane tan'P 3. Fa == F1 tan\fl
D. r, Ft x r1 cos \fI

where:
N
4. Nv
F[ = tangential force
cos 3 'II
T 1 = torque developed on driving gear

where:
rl = radius of driving gear

\fI = helix angle


C.T2==F t x r2
P = diametral pitch
P n = normal diametral pitch
where:

4> == pressure angle

T 2 = torque developed on driven gear

<l>n = normal pressure angle

r2 = radius of driven gear

F1 = transmitted or tangential load

Fa = axial load or end thrust

D. Total load, Tangential load and separation load relation N == actual number of teeth

t:«
Fn = vF, + Fr
2 N v = virtual number of teeth

f = face width

Fn = l

case
5. Strength of Helical Gears
where:

Fn = total load or tooth pressure between teeth


SwfY 78
F - -;-,--.-.­
Fr = resisting load or separation load
\ - P-78+-JV
e = pressure angle

6. Dynamic Load on Helical Gears

10. GEAR TOOTH PROPORTION TABLE


Qi(J5.V(.CfC.OS.~'l'+FI)COS.'l'
Fd== Ft + .
O.Sy + eCf cos 2 'l' + Ft ) 1/2
'GeM-Parts-----mr14-ij2-~---··-·~+ 200 ---·_--·---1 where:

~~~:;=
i Working depth
=rl~57~_._~_~
±2/P
_ _ ~/P
2/P
- J
,
v == pitch line velocity == nON

~t1.ole..sJePth ~_ ~57/P .~. ._.J 2.25/P .__ ~

, Clear.?nce.. t.Q~-w:.__________
O.25/P _. . ~

r~~~?ffi;~eter------·- ~~~7P---n-----n- -l~~2)/P--·-·--=1

rToo!.QJ~lckn_e.s~.___ 1570~_______ 1 .5708/P-=-­ 1


Worm gears are used where high speed ratios(10:1) and above are desired.

Where: P = diametral pitch N == number of teeth 1. Worm Gear Nomenclature

P = linear pitch = distance between adjacent threads


ltD
P
~
T
~
where: T = no. of teeth

2. Lead = the distance from any point on one thread to the corresponding point on
Helical gear nomenclature: the next turn of the same thread.
194 Gears Gears 195
Lead = p (for single thread) PROBLEM 1 (Oct. 1998)

= 2p ( for double thread)


Compute the tooth thickness of 14 1/2 deg. spur gear with diameter pitch of 5.

= 3p (for triple thread)


A. 0.34116in C.041416in
where:
B.0.31146in D.0.31461in
x = lead angle =
the angle between the tangent to the pitch helix and the plane of
rotation.
<1l = pressure angle IEm!!Dm
<pn = normal pressure angle 15708
Tooth thickness
DP
15708
2. v ::; velocity Tooth thickness
5
Note: No. of threads on the worm is equal to 1 for single threaded and 2 for double Tooth thickness 0.31416in
threaded
PROBLEM 2 (ME Bd. Oct. 97)

3. Tan x::; Lead A 36 tooth pinion with a turning speed at 300 rpm drives 120 tooth gear of 14 1/2

nO degrees involute full depth pressure angle. What would be the speed of the driven

gear.

A. 1000 rpm C 90 rpm


Strength of Worm Gear: B 100 rpm D.140rpm

The worm gear is weaker than the worm, therefore the design for strength is based
on the worm gear. IEm!!Dm
1. F =. SPty(120t)' :••...J.
t w l1200 + V : Using speed and teeth relation formula:
where: F1 = tangential pitch line load on the gear
Sw = safe stress, Table 12-2 T, N, = T2 N2
P = circular pitch
f = face width 36(300) = 120(N 2)
Y = form factor, Table 11-2
N2 = 90 rpm 36 T 120 T
V = pitch line velocity of the gear

Efficiency of Worm gear


tan x(cos <Pn- f tan x) PROBLEM 3 (ME Bd. Oct. 97)

A triple thread worm has a pitch diameter of 3 inches The wheel has 25 teeth and a

cos<Pn tan x + f
pitch diameter of 5 inches. Material tor both the worm and the wheel is at phosphor

bronze. Compute the helix angle (tan a).

where: coefficient of friction A. 0.20 C 040


B. 0.30 D. 14

~ IEm!!Dm
~
Pc ci rcular pitch
Bevel gears - are used to connect intersecting shafts, usually at right angle.
Pc nD _11.(5t = 0.6283 in
T - 25
Number of teeth and Speed relation

T, N, = T2 N2 Lead 3 P (tor triple thread)


Lead 3 x 0.6283 = 18849 In
196 Gears 197
Gears
tan ,1 = helix angle

SOLUTION
L 1.8849

tan <X = -- = '-'.- = 0.20


ITD rr(3)
From Vallance, p. 282

tan <Dn = tane cos x

tan14S = tano cos45'

PROBLEM 4 (ME Bd. Oct. 97) ¢ = 20.1"

Find the tooth thickness on the tooth circle of a 20 degree full depth involute tooth
having a diametral pitch of 3, circular pitch of 1.0472 and whole depth of tooth at 0.60. PROBLEM 7 (ME Bd. Apr. 96)
A. 10,7 mm C. 101 mm Two idlers of 28T and 26T are introduced between the 24T pinion with a turning
B. 13.2 mm D. 7.9 mm speed of 400 rpm driving a final 96T gear. What would be the final speed of the
driven gear and its direction relative to the driving gear rotation?
SOLUTION A 120 rpm and opposite direction
B. 80 rpm and same direction
Using the table of gear tooth proportions: C. 100 rpm and opposite direction
1.5708 D. 100 rpm and same direction
Tooth thickness

DP

N4
1.5708 ~
Tooth thickness
3 ~
Tooth thickness 0.5236 in 13.29 mm

PROBLEM 5 (ME Bd. Apr. 98)


A pair of gear/pinion of 42 tooth and 18 tooth with a diametral pitch of 0.7874 96 T
teeth/cm and the addendum is 0.80/p and the addendum l/p. The gear pressure
angle is 20° Compute the center distance of the pair of gears in meters. Since they are tangent to each other, then
A.0.5026m C.0.3516m
B. 0.3426 m D, 0.4013 m T 1N 1 = TzNz = T 3N3 = T4N4

T1Nl = T4N4
E'I!!DI:D
24(400) = 96(N4)
For gears turning in opposite direction:
_ T +T N4 = 100 rpm (opposite direction)
C - .g-..­ p
2 (DP)

T = no. of teeth

42 +18
PROBLEM 8 (ME Bd. Apr. 96)
C = --,.--- = 38.1 cm = 0381 m A spur pinion rotates at 1800 rpm and transmits to a mating gear 30 HP, The pitch
2(0.7879) diameter is 4" and the pressure angle is 14 1/2. Determine the tangential load in Ibs.
A. 495 C. 525

PROBLEM 6 (ME Bd. Apr. 96) B,535 D 475

A helical gear having a 14 1/2° normal pressure angle and transverse diametral pitch
of 23622 per cm. The helix angle is at 45' and has 8 teeth. Compute the
transverse pressure angle in degrees.
A. 22.2' C. 19.3° &N!!mmI
B. 189 0
D. 20.1°

Solving for the torque developed:


198 Gears Gears 199
P = 2n:TN A. 08095 C. 0.7825
30(33,000) = 2n:T(1800)
Ft B. 0.8035 O. 0.8085
T = 87.535 ft-Ibs

T = 1050.4 in-Ibs
E1!l!1i(.):.
Solving for tangential force:

Force = T/r

Force = 1050.4/2
Using D and N relation:
Force = 525 Ibs
0, N, = 02 N2

Nj Pc
O2 01 (--)
N2
PROBLEM 9 (ME Bd. Oct. 95) 18 T 72 T
Compute the pitch angle of a bevel gear given the pinion's number of teeth of 14 and 02 = 40, 1< )1
42 teeth on the gear.
A. 18.40 C. 28.4" For gears tl'~ning In opposite direction:
B. 33.4 0 O. 38.4 0 OJ +0 2
C = - "......­
2
OJ +40 j
~ 10.23 =

From Machinery's Handbook p 844: 0, 4092 in

T Pc _1t....~ = ~ n:(4.092)/18
Pitch Angle = tan -1 ---"
T
T
g
Pc 0.7854 in.
Pitch angle = tan' 14/42 = 18.4 0
PR, 3LEM 12 (ME Bd. Oct. 95)

The tooth thickness of a gear is 0.5 inch and its circular pitch is 1.0 inch. Calculate

PROBLEM 10 (ME Bd. Oct. 95)


the dedendurn of the gear.

Compute for the tooth thickness of 14 1/2 0 spur gear with diametral pitch = 5.
A. 0.3183 C 1.250
A. 0.3979 C. 31831 B. 0.3979 O. 0.1114
B. 003141 0 0.31416
SOLUTION
SOLUTION
Using the relation of diametral pitch and circular pitch: Pc (OP) =: n:
Using the table of gear tooth proportions: OP =: n/P,
OP =: n:I1
Tooth thickness 1.5708/DP OP =: n:
Oedendum =: 1.25/DP
Tooth thickness 1.5708/5 Oedendum =: 1.25/n:
Oedendum = 0.3979"
Tooth thickness = 0.31416"
PROBLEM 13 (ME Bd. Apr. 95)

An internal gear is set up with a 5 in diameter pinion and center distance of 18 inches.

PROBLEM 1 1 (ME Bd. Apr. 96) Find the diameter of the internal gear

Compute the circular pitch of a pair of gears having a ratio of 4 and a center distance A. 36" 8 215"
of 10.23 Each gear has 72 teeth and pinion has 18 teeth. C 26" O. 41"
200 (;{'(If,~ Gears :.lot
1'1~()I\I.i':1\!1 I ()
SOLUTION Find the tli:;1, incc between centers of a pair of gears, one of which has 12 teeth and
the other :J I teeth The diametral pitch is 7,
A. 3.0 In C. 4.0 in
B. 3.5 In D. 4.5 in
For internal gear:

SOLUTION
D2 .' D,
C=
2 For gear turning in opposite direction:

18 = D 2. :- 5

C = D, + D2
----­
2
2

D 2 = 41" DP, =
T,


D,

D, = 12/7 = 1.7143 in

18in 12 T 37 T
PROBLEM 14 (ME Bd. Apr. 95)
The minimum whole depth of spur gear of 14-1/2 deq. Involute type with diameter
D2 = 37/7 = 5.2857 in I( )1
pitch of 24 and circular pitch of 0,1309:
Solving for center distance:
A 0,09000 B, 0,08900
C, 008987 D. 0.08975 _ 17143+5.2857
C - - - - ~ - - -

3.5 in
2
SOLUTION

From Vallance p 262, Table 11-1: PROBLEM 17


Determine the pitch diameter of a gear with 28 teeth, 4 diametral pitch.
h =~~7
A. 7 in C. 9 in

Pd
B.8in D.10in

2.157
h 0.08987
24
Em!!JmD
PROBLEM 15 (ME Bd. Apr. 94) DP =T/D
The minimum clearance allowed for meshing spur gears with a circular pitch of
0.1571 and diametral pitch of 20. The spur gear have 25 teeth. D = TIDP
A 0.007855 B. 0.007558
C. 0008578 D. 0007585 D = 28/4 = 7 in

Em!!JmD PROBLEM 18
Two parallel shafts have an angular velocity ratio of 3 to 1 are connected by gears.
From gear tooth proportions table:
the largest of which has 36 teeth. Find the number of teeth of smaller gear.
0.1571
A 10 C. 12
Clearance = - .
B. 11 D. 13
DP

Clearance
01571
0007855

Em!!JmD
20

T, N, T2 N2
202 ( ;( )( II c,
:20;1
Gears
N 1 / N 2 0= T 2 / T 1

3/1 = 361T,
UlII N, N = 1.6

T 1 = 12 teeth
[1 1 6 D l

D1 j 1 6D 1 = 5.2

0, -t- 2

PROBLEM 19 DP, T , / D 1

The parallel shafts have a center distance of 15 in. One of the shaft carries a 40­ 10 = T 1 /2

tooth, 2 diametral pitch gear which drives a gear on the other shaft at a speed of 150 T, = 20 teeth

rpm. How fast IS the 40-tooth turning?


A 600 rpm C 150 rpm
B. 300 rpm D. 75 rpm
PROBLEM 21
E:m!IImI A spur gear 20 degrees full-depth involute teeth has an outside diameter of 195 mm
and a module of 6.5. Determine the number of teeth.
A. 20 C. 28
For gear turning In opposite direction: B. 25 D. 41

D, +D 2
Em!!lmD
C 0= ------­

2
To convert module to DP, use this relation:
D1 T 1 / DP, 1< :J
OP = 25.~ or M = 25.4

D, 40/2 20 in

0= M DP

20+D 2 25.4
15
DP 3.90769
2
6.5
D2 10 in
. di N+2
Using the formula of Speed and Diameter relation:
O utside iarneter = ----­
DP

0 1 N1 0= D2 N2

20(N 1) = 10(150)
Where: N = no. of teeth

N, = 75 rpm
N+2

195/25.4
390769

N + 2 == 30

PROBLEM 20
N == 28 teeth

A pair of meshing gears has a diametral pitch of 10, a center distance of 2.6 inches,

and velocity ratio of 1.6. Determine the number of teeth of smaller gear.

A. 10 C. 30 PROBLEM 22

B. 20 D. 40 What is the pitch diameter of a 40-tooth gear having a circular pitch of 1.5708 in?

A. 20 in C 30 in
E:m!IImI B. 25 in D. 35 in

For gear turning in opposite direction: E:m!IImI


C = [)~E2

2
Pc ltD
:::: --­
2(2.6) = D, + D2
T
D, + D2 = 52

1.5708 == ltD

D, N, 0= D2 N2
40

D,(N, / N2) = D2 D == 20 in
204 Gi'urs
PROBLEM 23 Gears 205
How many revolutions per minute is a spur gear turning if it has 28 teeth, a circular 25.4
pitch of 0.7854 in and a pitch line velocity of 12 It/sec? M =
DP
A. 378.44 rpm C. 954.66 rpm
B. 643.82 rpm D. 392.88 rpm
25.4
M = ------

8
Em!!imD
M = 3.175
Pc = ~~
T

0.7854 = reD
PROBLEM 26
28 The diametral pitch of a gear having a module of 23 is:

D = 7 in A. 9 C. 10
V=reDN B. 11 D. 12
12 = re(7/12)N
N = 6.548 rps x 60 392.88 rpm EiI!!imD
Use this formula to convert module to diametral pitch
P~BLEM 24
DP = ~~4
How many revolutions per minute is a spur gear turning If it has a module of 2, 40 M
teeth, and pitch line velocity of 2500 rum/sec?
A. 59683 rpm C. 476.85 rpm 25.4
B. 386.83 rpm D 312.83 rpm DP
2.3

SOLUTION DP = 11

M = module PROBLEM 27
D A gear turning 300 rpm has a diametral pitch of 8. If there are 40 teeth on the gear,
M = ­
T find the pitch line velocity of gear.
D A. 654 fps C. 7.45 rps
2 = ...
B 834 fps D 9.45 fps
40

D = 80 mm

EiI!!imD
V=reDN

2500 = rt (80)(N) DP = 2'

N = 9947 revlsec (60) = 596.83 rpm D

PROBLEM 25 8 = <1:.~
D
The module of a gear having a diametral pitch of 8 is: D = 5 in
A. 5.234 C. 4.23
B. 3175 D 2.34 v = re D N = re (5/12)(300/60) = 6.54 ft/s

SOLUTION
PH.ODLEM 28
Two gears meshing each other have 18 teeth and 30 teeth. If circular pitch is 023 in.
Use this formula to convert diametral pitch to module. Find the center distance between gears
A. 523 in C 323 in
B. 2.34 in D. 1.76 in
2 Of) G('(/1 S Gears :.!()/

SOLUTION
EmiGr·U'
Solving for the diametral pitch of the smaller gear: o
1(0 M -
Pel = --- T

T
o ~ T M
1( (0,)
0.23 = - - ­

18
0, T,M
0, = 1.317in 0, = 20 M

Solving for the diametral pitch of the larger gear: 02 = T2 M


1(0 O2 = 60 M

P c2 = -.

T C =01+02
2
023 = ~'21l 200 = 20 M I- 60 M
30 -'--" ­

02 = 2.196 in
2
M = 5

c = 0 1 + O2 1.317 + 2.196
--_.,--­
176 in
2 2
PROBLEM 31
0
A gear has a tooth thickness of 0.1308 in. Find the addendum of the gear at 145
involute.
PROBLEM 29
A. 2.12 mm C.3.12mm
A gear with pitch line velocity of 6.126 m/s when turning at 600 rpm. If module is 3, B.4.12mm O. 5.12 mm
find the number of teeth on the gear.
A. 60 C. 65
B. 50 0 70
SOLUTION

Em!!ImD . 1.57
Toot h thickness = - ­
V=1(ON
OP
6 126 = 1( (0) (600/60)
0.1308 = :!.57

0= 0195m=195mm OP

OP = 12
M =
o
T 1
Addendum 1/12 = 0.0833 in (25.4) = 2.12 mm
195 OP
3 =
T
T = 65 teeth PROBLEM 32
A gear has an addendum of 0.10 in. What is the dedendum of the gear at 20°
pressure involute.
PROBLEM 30 A. 0.037 in C. 0345 in
Two gears meshing each other has a center distance of 200 mm. The pinion has 20 B. 0.125 in O. 0235 in
teeth and gear has 60 teeth. Find the module of meshing gears.
A. 2
B. 4
C. 3
O. 5
Em!!ImD
0.80
Addendum
OP
201-\ (;('urs (;('(1 r s 209

0.80 0.3183 = F1 (0.2/2)

0.10
F1 = 318 KN

DP

Fr
DP = 8 tans = ..--

Ft

1
Dedendum 0.125 in tan 14.5° = ..£r
8 318

Fr = 0.823 KN

PROBLEM 33 200 mm
A gear is use to transmit 12 kw at 450 rpm with 26 teeth. If the diametral pitch of the
gear is 10, find the force tangent to the gear. PROBLEM 35
A.1234KN C15.42KN The pressure angle of the 12 in diameter gear is 20 degrees. The total load of the
B. 1424 KN D. 1645 KN gear is 5 KN and is turning at 750 rpm. Find the power delivered by the gear.
A 43.23 KN C. 5623 KN

SOLUTION B 6423 KN D. 3423 KN

P = 2ITTN SOLUTION

12 = 2 11 T (450/60)
Cos 8 = -­Ft
FN
T = 0.5093 KN-m
Cos 20° = ~
o = 26/10 = 2.6 in 5
F1 = 4.698 KN
T = Fxr
T = Ft x r

r
2.6
0.5093 = F l2"(39.37)
J T = 4.698 12(3~237) J
T = 0.7159 12in
F = 1542KN
P = 2 IT T N = 2 IT (07159) (750/60) = 56.23 kw

PROBLEM 34
A gear is use to transmit 20 kw at 600 rpm to a driven gear. The pinion has a pitch PROBLEM :36
diameter of 200 mm. If pressure angle is 14.5 degrees, find the load that tends to Three gears meshing each other has a driving power of 30 kw with 900 rpm. The
separate the two gears. speed ratio is 1:3:5 and each meshing gear has an efficiency of 96%. Find the torque
A. 0.234 KN C. 0763 KN developed of the driven gear, KN.m.
B. 0534 KN D. 0.823 KN A. 440 C. 5.60
B. 7.60 D. 8.20
SOLUTION
SOLUTION

P=2ITTN Po = power output of driven gear


20 = 2 IT T (600/60)

T = 03183 KN-m
Po = 30 (0.96) (0.96) = 27648 kw
T = F x r

210 Gears Bear, uti 211


No speed of driven gear
900
- - = 60 rpm

14
No
3 (5)

To = torque output
Po = 2 IT TN
Bearing

27648 = 2 IT T (60/60)
T = 4.40 KN-m

1. Bearing - a machine member which supports, guide or control the motion of


another.

2. Lubricant - any substance that will form a film between the two suriaces of a
bearing.

3. Babbit - a tin or lead base alloy which is used as bearing material.

4. Sliding Bearing = type of bearing where essentially sliding friction exists.

5. Ball Bearing - type of rolling-element bearing which uses spherical balls rolling
elements.

6. Roller Bearing - type of rolling element bearing which uses cylindrical rollers as
rolling elements.

Classification of Bearings according to load applications:

1. Radial bearing (journal bearing): supports radial load

2. Trust bearing: carries a load collinear to the axis

3. Guide bearing: primarily guides the motion of machine member without specific
regard to the direction of load application.

4. Viscosity = resistance to flow or the property which resist shearing of the


lubricant.

5. Absolute viscosity - viscosity which is determined by direct measurement of


shear resistance.

6. Kinematics viscosity - absolute viscosity divided by the specific gravity.


212 Rearing
lkuril/(/ 213
Units of Viscosity:
r ~-
Journal radius, in

dyne - sec L = axial length of bearing, in

2
cm n, = journal speed, rps

C, = radial clearance, in

Sliding Bearings:

Cd = diametral clearance D-d


Heat dissipation in journal bearings (Vallance p 240)
H ChLD

C. = radial clearance 778

where:
Diametral Clearance Ratio H = heat dissipated in Btu/min
C h = heat dissipation coefficient, of projected area. It-Ibs/rrun-in"
L = length of bearing, in
D = diameter of bearing, in
where:
Ball and Roller Bearings
p = unit loading or bearing pressure
p = F/LD
e = eccentricity " Bearing Sizes and Designation
e = radial distance between center of bearing and the displaced center of
Example of bearing designation:
the journal

D = diameter (bore) of the bearing

d = diameter of the journal


SAE or IS 314 is 300 series, NO.14
L = axial length of the journal inside the bearing

F = radial load
" Tabulated data on ball and roller bearings Vallance: Table 9-2 p 206

" Bearing Capacity based on stresses (Vallance p 205)

nLD
where: Fr Fr K2 (for roller bearings)
u = viscosity in reyns 5
n = speed in rps
p = unit loading, psi where:

Fr = total radial load, Ibs

(,. Frictional torque in bearings (Vallance p 231) n = number of balls or rollers

D = ball diameter or roller diameter, in

L = length of rollers, in

k 1 = 550 for unhardened steel

700 for hardened carbon steel


where:
1000 for hardened alloy steel on flat races
Ttl = frictional torque
fb coefficient of friction 1500 for hardened carbon steel
F = radial load D bearing diameter 2000 for hardened alloy steel on grooved races
k2 = 7000 for hardened carbon steel

(.. Petroff's equation for frictional torque (Faires p 302) 10,000 for hardened alloy steel

, ,g3<:
Tf:s~unr:.l,.ps (, Radial Load Catalog Capacities of Ball and Roller Bearings (Vallance: pp

Cr 207-213)

where:
T, = frictional torque, in-Ib (. Tabulated catalog capacities of ball and roller bearings:
u = viscosity, reyns (Fig. AF 16, P 595)
Table 9-7, P 212 and Table 9-8, p 213
214 Bcaiin q Bell t i T/9 215
PROBU:M 2
ks kt Fr A 2 in horizontal shaft rotates in a sleeve type bearing The coefficient of friction IS
o 10 and the shaft applies a load of 500 lb. Find the frictional resistance.
where: A. 20 Ibs C. 30 Ibs
Fe = Catalog rating of bearing, Ib (tables 9-7 and 9-8) B. 40 Ibs D. 50 Ibs
Fr = actual radial load on the bearing, Ib
H, = desired life of bearing, hrs of use
He = catalog rated life of bearing, hr Em!!iit-H'
ka = application factor takinq into account the amount of shock (Table 9-4)
F = frictional resistance
F=fN

But: N = W
where: F = 0.10(500) = 50 Ibs
ko = oscillation factor
= 1.0 for constant rotational speed of the races
= 0.67 for sinusoidal oscillations of the races PROBLEM 3
kp = preloading factor A bearing sustains a load of 4450 N. The shaft diameter is 100 mm, the coefficient of
= 1.0 for non-preloaded ball bearings and straight roller bearings sliding friction is 001, and the shaft speed is 400 rpm. Find the horsepower lost in
k- = rotational factor the bearing.
= 1.0 for bearings with fixed outer races and rotating inner races A. 9320 watts C. 83.45 watts
krel = reliability factor, Table 9-3 B 76.34 watts D. 45.23 watts

ks=. 3.~.. the ""~


.. ~KTNa···
. N '
~,,,;;:._. ..
C SOLUTION
kt = thrust factor
1.0 if there is no thrust-load component Solving for frictional force:
F=fN=fW
F = 0.01 (4450) = 44.50 N
PROBLEM 1
A 1 in horizontal shaft rotates at 500 rpm in a sleeve-type bearing. The coefficient of Solving for torque due to friction:
friction is 0.15. Calculate the horsepower lost in the bearing if the reaction between T = F (r) = (44.50)(0.05) = 2.225 N.m
the shaft and the bearing is 800 lb.
A. 0.2634 hp C. 0.0925 hp Solvinq for the power lost:
B. 1.2344 hp D. 0.4759 hp P = 2 IT TN = 2 IT (2.225)(400/60) = 93.20 watts

SOLUTION
PROBLEM 4
Solving for the frictional force on bearing: A 50 mm diameter shaft supported by two sleeve bearings carries a load of 13.3 MN.
F = frictional force The shaft rotates at 150 rpm. If the coefficient of sliding friction between the shaft and
F = f N = 0.15 (800) = 120 Ib bearings is 0.1, how much power is lost in friction?
r = 1/2 = 0.5 in = 0.041666 ft A. 261 kw C.345kw
B. 643 kw D. 108 kw
T = torque developed due to frictional force
T = F x r = 120 (0.04166) = 5 tt-lb &miit-H'
Solving for the hp lost due to friction:
P=2 IT TN = 27t(5)(500) = 0.4759 h Load for one bearing = 13.3/2
33,000 P Load for one bearing = 665 MN
6
Fr = frictional resistance = (0 10)(665 x 10 ) = 6.65 x 105 N
[3eu t: i WI 217
216 Bearing
2 Usmg the beanng stress formula: 2
Fr = 6.65 X 10 KN
S = PIA = 2000/1000 = 2 N/mm or Mpa
Solving for the tangential speed:
PROBL~M 7
v = IT 0 N = IT (0.050)( 150/60) = 0.3927 m/s
A 22 mm diameter shaft is supported by sleeve bearing at a distance of 0.50 m. A
Power Lost = F r v load of 2.2 KN is applied at 0.2 m from the left end. The sleeve bearings have an LID
Power Lost = (6.65 x 10
2)(0.3927)
= 261.14 kw (per bearing) ratio of 1.5. Find the maximum bearing pressure.
A. 1.82 Mpa C 2 12 Mpa
Solving for the power lost of 2 bearings: B. 3.45 Mpa O. 5.23 Mpa
Total power lost = 2(261.14) = 522.30 kw
Em!!mmI
PROBLEM 5 2:M[J = 0
A 36 mm shaft uses a sleeve beanng that sustain a load of 4000 N. If the allowable 05 RA = 0.3(2.2)
2
bearing pressure IS 1.3 MN/m . Find the length of the bearing. RA = 1.32 KN = 1320 N
A. 75.47 mm C. 89.23 mm
B. 23.44 mm O. 85.47 mm A=LO
But: UO = 1.5
L = 1.5 0
Em!!mmI A = (150)(0) = 150
2

F = 4000 N = 4 KN F F 1320 2
3
P= - ' = " - -~ ' - -
= 1.818 MN/mm or Mpa
S = 1.3 Mpa = 1.3 x 10 Kpa A 1.50" (1.5 )(22)2
Using bearing stress formula
F
Sb = ._..
A PROBLEM 8
3 4 A sleeve beanng is to have an LO ratio of 1.0 and an allowable bearing pressure of
1.3 x 10 = - 2
A 0.5 MN/m . Find the inside diameter and the length of the bearmg If it is to sustain a
A = 0.0031 m
2 load of 2550 N.
A. 23.45 mm C. 45.34 mm
Solving for bearing length: B.71.41mm O. 9834 mm
A=LO
00031 = L (0.036) Em!!mmI
L = 0.08547 m =, 85.47 mm
F = 2550 N
F = 000255 MN
PROBLEM 6 UO = 10
A 20 mm shaft uses sleeve bearings. The total load per bearing is 2000 N. An UO L=O
ratio of 25 is desired. What is the bearing pressure?
A. 1 Mpa C. 2 Mpa Solving for the area:
B. 3 Mpa O. 4 Mpa S = F/A
2
A = F/S = 000255 I 0.5 = 00051 m
Em!!mmI Solving for the diameter'
A = LO
LID = 2.5
00051 = 0(0)
L = 2.50 = 2.5(20) = 50 mm
0=007141 m =71.41 mm
Solving for the area:
A = LO = 50(20) 1000 rnrn"
218 Bearing Bells 219
PROBLEM 9
A sleeve bearing has an outside diameter of 1.50 in and a length of 2 in. The wall
thickness is 3/16 in. The bearing is subjected to a radial load of 450 lb. Find the
bearing pressure.
A. 100 psi
B. 150 psi
C. 200 psi
D 250 psi
15 BE3I-ts
SOLUTION

ro = Do/2 = 1.5/2 = 0.75 in

r, = ro - t = 075 - 3/16 = 0.5625 in


D, = 2 r, = 2(0.5625)
D, = 1125 in
Types of belt transmission
Solving for the bearing pressure:
F 450
P = --- = -.----- = 200 PSI
. (. Flat belt - used with flat pulleys and allows long distance
A 2(1.125)
<. V-belt - used with shave or grooved pulleys and provides stronger grip at short
distance between shafts.
PROBLEM 10
A thrust washer has an inside diameter of 12 mm and an outside diameter of 75 mm. <. Toothed belt - paired with toothed pulleys and used as timing belt where speed
If the allowable bearing pressure is 0.6 Mpa, how much load can it sustain? ratio must be maintained
A. 2582.9 N C. 329344 N
B. 1235.5 N D 863445 N Materials for transmission belts:

SOLUTION (. Oak-tanned leather is the standard material for flat belts.

Solving for the cross-sectional area: (. Chrome leather is used where very pliable material is desired.

A= lJ J =~ [(0.075)2.- (0.012)2 ]
(D/ - D;2) (. Rubber belt is used when exposed to moisture, acids and alkalies.
2
A = 0.00430476 m t, Fabric and canvas belts are used for light power transmission.

Solving for the load applied:


S = F/A
F = S A = (10.6 x 10
F = 2582.90 N
6)(0.00430476)
mmmI
For Open Belt Connection:
e

'" Pulley diameter and speed relation


D,N, = D2 N2
to, Belt length:
(D -D )
L = 1.57(0 2 + D1) + 2C + - 2- - - 1-
4C
220 Belts
Be/ts 221
" Angle of contact:

e=tc ± 2sid (R 2 ~ R1
1
J radians
(.. If Centrifugal Tension is Considered
1"1 - Fe
---
1"2 -Fe
= eIII
where:
O 2 -0 1 Fe = centrifugal tension
2
12wbtv /g
9 = rt ± ,radians 3
w = belt weight, Ib/in
b = belt width, in
Note: Use + sign for larger pulley t = belt thickness, in
Use - sign for smaller pulley v = belt velocity, fUsec

For Crossed Belt Connection (, Net belt pull (tangential force on pulley)
I" = 1"1 - 1"2
(, Pulley diameter and speed relation:
0 1 N1 =02 N:z (. Stress in Belt:
S _ 1"1
(., Belt Length: e W - bt

2C +
. D2 + D1 where:
L = 1.57 ([)2 + D,) Sw = working stress = 300 pSI for leather belts
4C
where: L = length of belt
(.. Power Transmitted and Torque Relation by Belt:
0 1 = diameter of smaller pulley
O 2 = diameter of larger pulley
R = radius of larger pulley
T = (1"1 - 1"2) r = I" x r
r = radius of smaller pulley
P = 2nTN, Kw
e = arc of contact, radians
C = center distance
where:
P = power, Kw
t,.. Angle of contact: T = torque, KN-m
e =n + 2slo,1 N = speed, rps

(.. Horsepower transmitted and stress relation


D 2 -D 1
e n + -----, radians
P (1"1 - 1"2 )V, hp bt = [V(Sw_~J2WV2)]
550 ls
C [(ets) I(e - 1)]
550
Belt Tension 9

" If Centrifugal Tension is Neglected


~
-1"1 = e fs
1"2
where: Formulas:
1"1 = tension in tight side
1"2 = tension in slack side (.. Belt Length
f = coefficient of friction
e = arc of contact, rad L = 1.57(0 + d)+2C + (0 - d)2
4C
222 Belt s Bells 223
'- Center Distance PHOBLEM I
Find the angle of contact on the small pulley for an open belt drive with a 72 In center
distance The pulley diameters are 6 in and 12 in.
A 175.22° C. 185.34°
B. 17034° D. 165.34°

c. Arc of contact on small sheave


IE:.1!!.immI
For smaller pulley:
8 = angle of contact
where:
L = pitch length of belt 8 = 180 - 2sln
.1 ID - D
1 2
l-2C-J
l
C = center distance
D = pitch diameter
d = pitch diameter of large sheave 8 = 180 - 2sin' [12 -6J = 175.22°
2(72)
b = 4L -o.28tOtt:l)

'- Standard Pitch Length and Designation of V-Belts: PROBLEM 2


Table 3.3, p 21(PSME Code) Find the belt length at a 72 in center distance connected In open belt. The pulley
diameters are 6 in and 12 in.
Example: A. 162.34 in C 123.55 in
B75 IS Section B v-belt with length of 76.8 inches B. 173.45 in D. 172.39 in

l. Horsepower Rating for v-belts:


HP = XS09 1 -)is"Z$3
ki-I!i';r-UM
de L = belt length
where:
PH = recommended horsepower IT (0 D)2
L= -(D +D )+2C+ 2- 1
X,Y,Z are constants (Table 3.6) 2 1 2 4C
S = belt speed in thousands of feet per minute
de = equivalent diameter of small sheave which is equal to pitch diameter
(12-6)2
multiplied by small diameter factor (Table 3.9) L = -IT (12 + 6) + 2(72) + _._--
2 4(72)
'" Design Procedure in Determining the number of V-belts required:
Given: Size of belt, sheave diameters, speed, power transmitted L = 172.39 in
1. Find the length of the belt from Table 3.3.
2. Solve for the center distance and the arc of contact. PROBLEM 3
3. Find the value of X, Y & Z from Table 3.6. A 6 in diameter pulley turning at 600 rpm is belt connected to a 12 in diameter pulley.
4. Solve for the speed ratio and find the small diameter factor from Table 3.9, then If there is 4% slip, find the speed of the 12 in pulley.
solve for de. A. 187 rpm C. 203 rpm
5. Compute the HP rating per belt. B. 223 rpm D 288 rpm
6. Find the length correction factor from Table 3.7 and arc of contact correction factor
from Table 3.8, then solve for the corrected HP rating per belt. __-r-1.IlI.r-J~
7. Find the service factor from Table 3.5, then divide the corrected power transmitted
by the Hp rating per belt.
USing the diameter and speed relation:
D1 N, = D2 N2
6(600) = 12(N z)
Nz = 300 rpm
Belts 225
224 Belts
F (030)(165.63x IT )
1 e 180
Considenng the 4% slip F2
N2' = 300 (1 - 0.04) F 1 = 2.38 F2
N 2 ' = 288 rpm S = F/A
F=SA

PROBLEM 4 F=S (~xd2) = 400 l(~' XO.25 2 : = 19.631bs


4 4)
For a given belt a manufacturer gives a horsepower rating of 2.5 hp per inch of width
based on a belt speed of 2600 ft/min. The drive is to handle 10 hp and the and the F2 = 19.63/2.38 = 825 Ib
arc of contact correction factor is 0.90. Find the width of belt needed. Assume that
belt widths are available in 1 inch increments. n(1)(100) .
v = '.-. - . - = 26.179ftl min
A. 3 in C. 4 in 12
B. 5 in D. 6 in
HP = ~Fl_= ~;0\1 = (196~_=_8.25)(?~179J = 0.00903 hp
33,000 33.000
Em!!ImD
PROBLEM 6
Solving for the actual hp rating A 3/8 in flat leather belt is 12 in wide and is used on a 24 in diameter pulley rotating
3 0
600 rpm. The specific weight of the belt is 0.035 Ib/in The angle of contact is 150
P = 25 (0.90) = 2.225 hplin If the coefficient of friction is 0.3 and the allowable stress is 300 psi how much hp can
it transmit?
10hp A. 34.5 hp C 78.5 hp
Belt width = 2.2_5~E

B 69.5 hp D 54.5 hp
in

Belt width = 4.44 in


Im!!iit·a.
For flat belts:
Therefore: Use 5 in width of belt
pv 2 e fe_1
F1 - F 2 = b t (s - - - ) ( - - )
2.68 e f8
PROBLEM 5 v = IT (24/12)(600/60) = 62.83 ft/s
A 1/4 in round endless belt connects a 1 in diameter pulley with a 2 in pulley; spacing
between pulleys is 4 in on centers. The allowable stress is 400 psi. If the 1 in pulley e '8 = e 'o
30)l150)(rr!180) = 2.193
is the driver and rotates 100 rpm. How much horsepower can be transmitted?
Pv2 0035(62.83)2
Assume the coefficient of friction to be 0.30 for each pulley. ----- = .. = 51.55
A.0123hp C 0.0115hp 2.68 2.68
B. 1.234 hp D. 00090 hp
12.1932 -1l
F1 - F2 = (12)(3/8) [300-51.55JI-' - -
Em!!ImD L 2.1932 --'
F1 - F 2 = 608.26 Ibs
Since both have the same coefficient of friction, smaller pulley will be the basis of
computation.
HP =(1"".1..- F2 )v (60826t(62.83x60) = 69.50 hp
33,000 . 33,000
8 = angle of contact
l
.,
8 = 180 - 2sln

F1
.-.-- ~ e f8
l
D2-D1
--2C-J 180 - 2 sin
1
(~1J
l2(4 )
= 165.63°

F2
.6.MJ Belts l uak c: 227
PROBLEM 7
A 5 mm round belt connects a 20 mm pulley with a 40 mm pulley. The center
distance is 150 mm. The 20 mm pulley rotates at 100 rpm and the coefficient of
friction of the belt is 0.25. Find the horsepower capacity for this arrangement if the
2
allowable belt stress is 2.6 N/mm .
A. 0.12 kw C 000282 kw
B. 0.23 kw D. 0.054 kw

SOLUTION

F
~
S = -'.
A
F = SA Brake· is a device which is used to regulate or stop the motion of a body.
F = (2.6) (nI4)(5)2 = 51.05 N
For larger pulley: Types of Brakes:

8B = 180 + 2 sin'1l-~2 ~_f3l J= 180 + 2 sin' 20.-=-2.~ 1= 187.65° = 3.275 rad 1. Mechanical Brakes
150 ) a. band d. disk
b. block e. spot

For smaller PUII:(:R 2 -R J '1(20-10J °


c. shoe

85 = 180·2 sin l'-C'-- 1


= 180·2 sin -1'50- = 172.35 3 rad 2. Hydrodynamic Brakes:
a utilize fluid friction

For laroer pulley: 3. Electrical Brakes:


e 10 = e(15
25)(3 275) = 2.27
a. utilized the strength of electromagnetic fields
For smaller pulley:
ere= e(O 25)(3) = 2.12 Simple Band Brake

Therefore smaller pulley governs the design.


F1 18
Tension in the band f1. = e f8
-=e F2
F2
where:
~05=2.12 F1 =: force on tight side f =: coefficient of friction
F2 F2 =: force on slack side 8 = angle of contact. rad
F2==24.10N
v = n (0.02)(100/60) = 01047 rn/s
P = (Fl' F2 ) v == (51.05 . 24.1 )(0.1 047) = 2.822 watts =: 0.002822 kw
Brake Torque Developed:

T =: (F1 - F2)r

where:
T = brake torque
r =: radius of friction surface on the drum

Actuating Force Required: (by taking moment about the pivot point)
Fa = aF2
L
228 Brake Brake 229
Maximum Unit Pressure:
where: W = weight lowered
h = total distance traveled
where:
w = width of the bond
PROBLEM 1
Stress in band: The band brake of a band brake has 210 degrees of contact with its drum. By
laboratory tests, it is found that the pull on the tight side is 800 Ibs and the pull on the
slack side is 285 lb. What is the coefficient of friction?
where: A. 0.281 C. 0 186
B 0.753 D. 0.453
t = thickness of the band

Differential Band Brake ~


By taking moment about the point: Using tension ratio of belt:
F, fO
-=e
F2
IT
800 1(210° x_ _)
Self-Locking Differential Band Brake: ___ = e 180
285
A differential band brake is self-locking when Fa is zero or negative. 2.807 = e3665f

Block Brake (Vallance p 364) Take In both sides:


In2.807 = In e36651
In2.807 = 3.665f (In e)
f=0.281

PROBLEM 2
2
The inertia load on an electric brake is 32 Ib-ft and the drive shaft is rotating at 2000
where: rpm. What average torque is required to bring this load to a complete stop in 12 sec?
T = braking torque A. 173.61 ft-Ibs C. 12345 tt-lbs
F r = radial force between the drum and each shoe B 153.45 ft-lbs D. 237.23 ft-Ibs
f = coefficient of friction
h = effective moment arm of the friction force
SOLUTION
r = radius of the friction surface of the drum
8 = angle of contact
Using the formula of torque in electric brake:
P max = maximum normal pressure between block and drum
2
w = axial width of block T= J!v.r)N _
308(1.2)
Automotive Shoe Brake (Vallance pp 366-370)

Heat Dissipated in Brakes (Vallance p 374) (32)(2000) = 173.16 ft-Ibs


T = (308)(1.2)
Icl ""fFrV
where: H = heat dissipated
f = coefficient of friction PROBLEM 3
F r = radial force The band of a band brake has a contact angle of 180°. It is found experimentally that
V = surface velocity pulls on the bands are 275 kg and 100 kg respectively under certain operating
conditions. Find the coefficient of friction
For Brake used in lowering of a weight: A. 0.973 C 0197
H·.. ;=i•. Wh B. 0.567 0 0322
230 Brake elwell 2:31

Ki·)!WDI
F1 Ie

17 Clutch
-=e
F2

275 f(180ox no)


--- = e 180
100
nl
2.75 = e
In2.75 = Itf (In e)

~
f = 0.322

PROBLEM 4
A brake requires 900 in-lb of torque to stop a shaft operating at 840 rpm in a period of
3.5 seconds. What is the inertia load expressed in Ib-ft 2 ?
Clutch - is a machine member which is used to connect shafts so that the driven
A. 90.23 C. 34.45
shaft will rotate with the driving shaft. and to disconnect them at will.
B 74.34 D. 9625
Types of clutches:
Em!!Iir.mI
1. Jaw clutches - jaws or teeth in the two elements interlock
2)N
T= (wr
2. Friction clutches - the drivinq force is transmitted by friction; the major types are'
308 (t)
plate or disk clutch, cone clutch, band clutch, block clutch and expanding-ring
T = 900/12 = 75 ft-Ib clutch.
2)(840)
75 = lwr
308(3.5) 3. Hydraulic clutches - the torque is transmitted by a moving fluid
w ~ = 96.25 ft2 -Ib
4. Electromagnetic clutches - the torque is transmitted by means of a magnetic lieid.

Disk or Plate Clutch


Power transmitted:

P = 2rrT N

T = n f Fa fl

where:
T = torque transmitted
n = number of pairs of mating friction surfaces
f = coefficient of friction
Fa = axial load
r, = mean friction radius

Mean frictional Radius, rf


For uniform pressure disc clutch

fr =
2.'
-
[f O
3

3 ro 2-'--. rI 2
__ rj3 ]
Clutch 233
2:32 Clutch

For uniform wear clutch (worm clutch) (0~)(550) (0.200/2)


T =-sln 100
T = 142.53 N-m

PROBLEM 3 (ME Bd. Apr. 97)


Cone clutch: 2
If the inertia is 90 Ib-ft and the speed of the driven shaft is to be increased from 0 to
2500 rpm in 5 seconds, shaft material is of SAE 4140 with yeild strength of 110,000
T:;: Fa trt lb/in", find the clutch starting torque in It-lb.
sine A. 159 C 146
B. 128 D. 191
PROBLEM 1
0
A cone clutch has an angle of 10 and a coefficient of friction of 0.42. Find the axial Em!!immI
force required if the capacity of the clutch is 7 kw at 500 rpm. The mean diameter of
the active conical sections is 300 mm.
Torque = I ex
A. 234.45 N C. 368.49 N g
B 186.45 N D. 492.45 N
(0;2 - ~
ex = - - - -
SOLUTION t
rev 1min 2nrad
W2 = 2500-- x ----- x --.-- 26180 rad/s
P=2nTN min 60 sec rev
7 = 2 n T (500/60) (L = (~6~Q) - 0 = 52.359 rad/s"
T = 0.13369 KN-m 5
T = 133.69 N-m
(90Ib· ft2 )(52.359rad 1S2)
Torque ---32~21t1S2----
Using uniform wear method:

T= l
~~~ ~; ri J= sif;ex (D2m J Torque

Alternate Solution:
146 lb-tt

133.69 = (0.42~ (0.300/2) 2


sin 10 T= (~)f'J = 900(2.?OO) =146.10ft-lb
F = 368.49 N 308 (t) 308(5)

PROBLEM 2 PROBLEM 4
How much torque can a cone clutch transmit if the angle of the conical elements is 10 Find the frictional radius for a disc clutch. The disc clutch has an outside diameter of
degrees the mean diameter of the conical clutch sections 200 mm and an axial force 10 in and an inside diameter of 6 in.
of 550 N is applied? Assume the coefficient of friction is 0.45. A. 454 in C 5.34 in
A. 123.23 N-m C 142.53 N-m B. 4.08 in D. 8.44 in
B. 34.56 N-m D. 234.56 N-m
SOLUTION
SOLUTION
ro = 10/2 = 5 in
r, = 6/2 = 3 in
T= ~~~;l~trl.J 3
l
l
3
2 ro - r,.3 2 r 53 3 1 = 408 in
fF r'=:3 ~J

a , 3l5" 3"]
T= ~--- (rm )
Sin ex
234 Clutch ("{llldl 235

PROBLEM 5
In a clutch the outside diameter is 8 in and the inside diameter is 4 in. An axial lorce
4i·"mtmI
of 400 Ib is used to hold the two parts together. If the coefficient of friction of the Using uniform wear method:
mating materials is 0.4, how much torque can the clutch handle?
A.123.45in-lb C.213.34in-lb
B. 497.80 in-lb D. 563.23 in-Ib
P = 2]"(TN_
33,000

IIIEm!!IilimI 20 = 2]"(T.(1 00~2

33,000
r, = 4/2 = 2 in T =105.04 ft-Ib
ro = 8/2 = 4 in

Solving for the torque:


T = s;;a[~-?I )
IF
2 fP [r 3 _ r3 T = -.--(rm )
] sin c:
T= 3 r:2~2-
10504 = iO.3Q)~1} (16 12)
sin 10"
2
T = -(0.40)(400) l(4)3_(2)3] .
--_._-~- = 497.8 in-lb F = 91.202 Ib
3 (4)2 -- (2/ . F
slnr~ = -
r,
sin 10° = 91 202/F"
PROBLEM 6 Fn = 525.21 Ib
A disc clutch has 6 pairs of contracting friction surfaces. The frictional radius is 2 in
and the coefficient of friction is 0.3. An axial force of 100 Ibs acts on the clutch. The Fe = Fn (sina + I casu)
shaft speed is 400 rpm. How much horsepower can the clutch transmit? Fe = 525.21 [(sin10o + 0.3(cos100)] = 246.37 Ib
A. 2.28 hp C. 3.23 hp
B. 1.23 hp D. 4.23 hp
PROBLEM 8
SOLUTION In a band clutch, the ratio 01 the pullan the tight side of the band to that 01 the slack
side is 4:1. The band contacts the drum for 250 degrees. What is the coefficient of
Using a uniform pressure method: friction?
T = f P (r,) n A. 0.123 C 0234
T = (030)(100)(2)(6) B. 0.318 D. 0.462
T = 360 in-Ib

~
P = 3TCT~ 2]"((360/12)(400) = 2.28 hp
33,000 33,000 F I 1 F2 = 4/1
F,
-- = e
fO
F2
PROBLEM 7 4/1 = e l(250 x ,,/180)
A cone clutch has cone elements at an angle of 10° The clutch transmits 20 hp at a
speed of 100 rpm. The mean diameter of the conical friction sections is 16 in and the In 4 = In e436331
coefficient 01 Iriction ;s 0.3. Find the axial force needed to engage the clutch.
A. 246.37 Ibs C. 234.56 Ibs
f = 0318
B. 212.561bs D. 346.781bs
236 Clutch
('1111 ell 237

PROBLEM 9
0
A band clutch has an angle of contact of 270 on a 15 in diameter drum. The F1 = F2 e 1o
.' IT
rotational speed of the drum is 250 rpm and the clutch transmits 8 hp. The band is 0.4(250 x ----)
1/16 in thick and has a design stress of 5000 psi. How wide should the band be? 750 =, F2e 180'
Assume a coefficient of friction of 0.40. F2 = 13094 Ib
A.2.123in C.1014in
B. 3.234 In D. 4354 in T = (F 1 - F2) r
T = (750 - 130.94)(16/2) = 4952.48 in-lb = 412.707 It-lb
SOLUTION
p = 2ITTN 2IT(412.707)(35_0) = 27.50 hp
F1 = S b t 33,000 - 33,000
T = (F 1 - F2 ) r
p = 2n:TN
33,000 PROBLEM II
8 = _2_ITT_(_25_0_) Find the power capacity under uniform wear of a cone clutch with mean diameter of
33,000 250 mm if the conical elements are inclined 8 degrees and the axial force is 450 N.
The rotational speed of the driver is 200 rpm and the coefficient of friction is 0.20.
T = 168.067 ft-Ib = 2016.811 in-Ib
A. 1683 kw C. 1.287 kw
B. 2340 kw D. 3234 kw
F
.-1 = e to
F2
EellUi[']~1
fe
F 1 = F2 e
" It
0.4(270 x - - ) Using uniform wear method
F1 = F2e 180C = 6.586 F2
T =_.t£...1 !o-t:.i] = .lSl:.?1~450) (0.25/2) = 80.8345 N-m
sin (X L 2 sin 8°
Substitute to the equation of torque: T = 0.0808345 KN-m

2016.8114 = (6.586F 2 - F2)(15/2)


P = 2 IT TN = 2 IT (0.0808345)(200/60) = 1.683 kw
F 2 = 48.14 Ib
F 1 = 6.586 (48.14) = 317.0471b
PROBLEM 12
F1 = S b t
Determine the power capacity of a cone clutch under uniform pressure and assuming
317.047 = (5000)(1/16)(b)
the following conditions: major diameter = 250 mm; minor diameter = 200 mm; length
b = 1.0145 in
of conical elements in contact = 125 mm; rotational speed = 870 rpm; coefficient of
2
friction = 0.30; and allowable pressure = 70,000 N/m .
A. 12.34 kw C. 14.56 kw
PROBLEM 10
B. 19.09 kw O. 23.45 kw
The angle of contact of a band clutch is 250 degrees. The cross section of the band
is 1/16 in x 1.5 in. The design stress for the band material is 8,000 psi. If the drum is
SOLUTION
16 inches in diameter and rotates at 350 rpm, what is the horsepower capacity of the
clutch? The coefficient of friction is 0.4.
1
l
A. 20.34 hp C. 22.34 hp 3
r - r3
B. 1523 hp D. 27.50 hp T = P b IT 00f(2/3) °2. . '2J
ra - r,
"''-e]''!I[eg' I 0.25t-..-'-.
0.2 } [' (0.125)3 - (0.10)3 J
T = 70,000(0.125)(IT) I -- 0.3)(2/3)1 -----~-----
F 1 = S b t = (8000)(1.5)(1/16) = 750 Ib
,2 l
(0.125)2 - (0.1 0)2
T =209.603 N-m = 0209603 KN-m
23H CLutch [I.[u,11 ineries 239

PROBLEM 13
= 2 IT T N = 2 IT (0,209603)(870 160) = 19,096 kw
1 B Machineries
Find the power capacity under uniform wear of a cone clutch with the following
specification speed = 870 rpm; length of conical elements in contact = 125 mm; major

~
diameter = 250 mm; minor diameter = 200 mm; coefficient of friction = 0,30; and axial
operating force = 500 N,
A. 7,69 kw C, 9,34 kw

B, 5,23 kw D. 3.23 kw

SOLUTION
~
. r0 - r, Polar Moment of Inertia
sin u. = ---
b
. 0.125-0.10 Using English units:
sino; =
0,125 Jrr:= p.L. J
a = 11,537° 9
2
Where: Jm = polar moments of inertia 01 masses, ft-lb-sec
2
p = Ib/ft
T= ~-~ L = length, ft
sina
g = 322 tt/sec"
500(0.3)(g~_135_!_0.10) J = polar moment 01 inertia 01 area with constant cross-section, ft4
= 84375 N-m = 0.084375 KN-m
Sin 11 ,53r Using SI units:

P = 2 IT TN = 2 IT (0.084375)(870/60) = 7.69 kw J m = p LJ
2
Where: J m = kg_m
PROBLEM 14 3
p = kg/m
Assuming uniform wear, find the power capacity of a single disc clutch with an outside L = length, m
4
and inside diameter of 200 mm and 100 mm respectively, a rotational speed 01 1160 J =m
rpm, a coefficient of friction of 0,35 and an axial operating force of 800 New1ons.
A. 4,23 kw C. 6.32 kw Radius of Gyration
B. 2.55 kw D. 8.23 kw
Using English Units
SOLUTION
Ko = radius of gyration
Using uniform wear:
I
r + rj
T = fFl--2-~
0 l Ko =~Jmg
W
. •.·•·.

Where: J m
Jm= W
g
k~
= polar moments of inertia 01 masses, tt-lb-sec"
T = (0.35)(800{ 0.10; 0,05 J = 21 N-m = 0.021 KN-m
Ko = ft
g = 322 It/sec"
W = weight, Ib
P = 2IT TN = 2IT (0.021)(1160/60) = 2,55 kw
,\Tn r II i fI ('I WS 241
240 ModI ineries
Using 51 Units !(l~etic Energ:y

J m == Kinetic Energy of Translating body


2
12 Wv
KE == ~mv KE == ~-
Where: J m == kg_m2
2 2g
3

p == kg/m
Ko = m
Kinetic Energy of Rotating body
m == mass, kg KE== Vz Jrno}

Radius of Oscillation Where: J m == moment of inertia, ft-lb-sec'


(J) == angular velocity, rad/sec

For isosceles triangle: 3/4 of height

For circle: Total Kinetic Energy


5/8 of diameter

For parabola: 5/7 of the height


2 2
KE == 172 mv +% J m co
Center of Percusion
Force of a Blow:
Q == distance from axis of rotation to center of percussion
WS
Average force of blow == ­
d

Where: r == the distance form axis of rotation to center of gravity of body Where: 5 == total height, It

W == weight of driver in Ibs

Formulas relating Torque and Angular Acceleration o == distance in feet which pile IS driven

Using English units: Linear Impulse and Momentum

" . . .... ... W


Lmear momentum = rnx v.= ,,- v

Where: To == torque in pounds-feet Linear impulse: F xt


J m == moment of inertia, ft-lb-sec 2
K, == radius of gyration, ft Where: F == force

a == angular acceleration, rad per sec" t == time

Using 51 Units: Angular impulse and Momentum

Where: Angular momentum == Jm ill

To == torque, N-m
J m == moment of inertia, kg_m2 Angular impulse == To x t

a == angular acceleration, rad per sec 2


Angular impulse == Change in angular momentum

To x t == J m(rot - Ulo)
242 ATClchincries Mochinerics 243
Centrifugal Force For cast iron having a tensile strength of 19,000 psi the bursting speed would
be:
In English units: v == ~1 0 x 19,000 =; 436 fUsee

F =0 centrifugal force
Thickness of Cast Iron flywheel:

For solid rim:

y== ~FR9
W

Where: W weight, Ibs


=0
For jointed rim:
v velocity, ftls
=0

R radius, ft
=0

N speed, rpm
=0

In 51 units:
F= 0.01097 m R N2
Where: t thickness, in
=0

Where: N speed, rpm


=0
d diameter, in
=0

R = radius, m n = number of arms


m == mass, kg v peripheral speed, ftls
=0

F = force, N
Spokes and Arms of flywheels:
F = 0.01097 h1 R N2
The strength of the arms should equal three-fourths the strength of the shaft In
torsion.
FLYWHEEL
If W equals the width of the arm at the hub and 0 equals the shaft diameter. then
W==1';30 for a wheel having 6 arms. For 8 arms, W:;i1:2D
Energy of Flywheel due to change of Velocity:

2 2
E== W(v 1
2g-­
- V2 )
~
For simple pendulum
Where: W = weight, Ibs
v = velocity, ftls

Centrifugal Stress in Flywheel rims: T periods, sec

=0

y2
L = length of pendulum, ft

S == -'---­ g = 32.2 ftls


10
Where: S = tensile strength of rim material, psi For physical pendulum:
v = nrn speed, His
2
T == 2rr Jk o
gr
L,+,+
/\1u('11 incries
ko = radius of gyration, It Mue/tillcrics 245
r = radius, It
g = 322 ft/s 2
P = internal pressure, psi
Thick-walled cylinders of ductile material; closed ends.
Section Modulus of square beam, Z
Clavarinc's equation is used:
3
Z =a /6

Where: a = side of square beam

ft = poisson's ratio
~
Thick-walled cylinders of ductile material; open ends.
Strength of Taper Pins: Clavarinos equation is used:

Using English units:

d=113 JOST ,l1 = poisson's ratio

Where: T = torque, Ibs


Collapsing Pressure of Cylinders and Tubes subjected to External Pressures.
S = safe unit stress, psi

HP = horsepower
p == 50,210,000 (1/0)3

N = speed, rpm
P = collapsing pressure, psi

o = shaft diameter, in t = wall thickness, in

d = tapered pin diameter, in o = outside diameter of cylinder or tubes


Using SI units:

~
Where: d = tapered pin diameter, mm
N = speed, rpm 1. Torsional deflection of circular shafts:
0= shaft diameter, mm
P = power transmitted, watts
S = unit stress, N/mm 2 a

where: (X = angular deflection, deg

T = torque, in-lb

L = length, in

D = shaft diameter, in

For low pressure cylinders of cast iron: G = 11,500 psi (for steel)

' ,DP 2. Shaft diameter for 0.08 degrees per foot of length of shaft
t = t h.ckness, = --,'~.,,'~
In
2500 deflection.
o = inside diameter, in
Mochin eries
247
24fi !'v[Ucll illcrics
For English units: L (A + 0) xA xB xK

D = 0.29 ·iff it Where:


B = width between drum flanges, in
Where: D = diameter, in 0= diameter of drum barrel, in

T = torque, in-Ib H = diameter of drum flanges, in

P = horsepower K = factor from table

N = speed, rpm Y = depth no filled on drum

For SI units: Rope Load due to Bendi ng

1. St> = bending stress =

Where: D = diameter, mm

P = power, watts Pb"'· Sb>A

N = speed, rpm

T = torque, N-mm E = 12,000, 000 psi (average value)

o = sheave diameter
20 times its
3. Shaft deflec tion of 1 degre e for a length of
2. dw = wire diameter
diame ter.

0.10

P = power, hp
o = diameter, in 3 A=
N = speed, rpm

T = torque, in-lb

4. Linea r deflec tion of shafti ng


e)
own weight
a. Shafting subjected to no bending action of pulleys except its
8:~5,9JE2
b Shafting subjected to bending action of pulleys, etc

Where: L = shaft length, ft


o = shaft diameter, in mmm
Stren gth of Chain s
~ W20 54,000 0
2

Lengt h of Wire Rope W = breaking loads, Ibs


o = diameter of bar, in

A = depth of rope space on drum, in

/11' \20(H "-.D .: 2.Y)/2

L = length of wire rope, It


248 Mach i tier ies Machi/writ's 249
Milling machine feed rate:

f m ", It nt N

Milling Cutters fm = milling machine feed rate, in/min

ft = feed rate, in/tooth

nt = number of teeth of milling cutter

Number of Teeth of Milling cutters N = speed, rpm

T",,6.3D
W
Estimating Planer Cutting Speeds
T = number of teeth
Vc = cutting speed, fpm

o = cutter diameter, in
So = number of cutting strokes per minute

W = width of cut in inches

L = length of cutting stroke, ft

A = helix angle of cutter

o = depth of cut, in
Planning Time
For high speed milling with sintered carbide:

T= W[LX(~,' +1)+o,o25]•
T =.'. ,K H~
FNelW
L" l v: v,
T = time, min

T = number of teeth
W = width of surface to be planed, in

H = horsepower
L = length of stroke, It

F = feed per tooth, in


V c = cutting speed, fpm

N = revolutions per minute of cutter


V r = return speed in fpm

D = depth of cut, in

W = width of cut, in

Cutting Speed ~
Size and types of rivets:
v= nDN
Rivet diameter falls between:
N = speed, rpm
D = diameter, ft
v = cutting speed, fpm D= 1:2jt'· to 1.4jt

Where: t = thickness of plate


Cutting time for turning, boring and facing Joint Strength

J=h F = safe tensile load

IN F = n x Ar x 5 s

T = cutting time, min

f = feed rate, in/rev


N = number of rivets

N = lathe spindle speed, rpm


A r = cross-sectional area of rivets

Ss = allowable shearing stress

250 IIIarhineries Muchincries 251


Working Strength of Bolts

W ==St(0.55d2 -0.25 d)
~
W == St (A - 0.25d) Horsepower of friction wheels:
W = working strength of bolt, Ib
1. HP = - - - ­
St = allowable working stress, psi
d = nominal outside diameter of bolt, in
Where:
A = area at the root of the thread, in 2 D = diameter of friction wheel
N = speed, rpm
Holding Power of Set Screw:
W = width of face, in
DNd~·3 f = coefficient of friction
P = ,hp
P = force in Ibs per inch of width

T = 1250 0 d 2.3 , ltHb 2.


3.1416 Pxf
C = factor = 33,000)(12
Where: D = shaft diameter. in
d = set screw diameter, in 3. HP DxNxWxC
N = speed, rpm

Tensile Stress Area of Thread or Screw


S ur Gear
a. For steels up to 100,000 psi ultimate strength

Gear set Center Distance:


At == U.ll::lo41 u - - ­

b. For steels over 100,000 psi ultimate tensile strength


C = center distance
At = Dp = pitch diameter of pinion

Additional Gear Formulas from Machineries Handbook


D = basic major diameter, in
n = number of thread per inch 1. Gear set Center Distance:
Dm n = minimum pitch diameter of external thread, in
2
At = tensile stress area, in
C ;"Qp(fP~~(jHatiO·t1)
2

Sharp V - thread
C = center distance

Dp = pitch diameter of pinion

D = depth of thread
2. Circular Pitch for Given Center Distance and Ratio
0= P xcos30o

D= 0.866

no. of thread per inch

p = pitch, in
C = center distance
T = number of teeth

L
252 Machineries
Moe h i ucvie s 253

3. Circular Thickness of Tooth when Outside Diameter has been Enlarged

Wrapped-spring Clutches
t = tooth thickness
Pc = circular pitch 1.
e = amount outside diameter is increased over standard
¢ = pressure angle
HP = horsepower

N = speed, rpm

4. Circular Thickness of Tooth when Outside Diameter has been Enlarged


2. Clutch starting torque

ft-Ibs
t = tooth thickness
Pc = circular pitch
e = amount outside diameter is increased over standard 2
I = W R moment of inertia, Ib-ft
2

¢ = pressure angle W = weight, Ib

R = radius of gyration, ft

t-N = final rpm - initial rpm

5. Chordal Thickness of Tooth when Outside Diameter is Standard


t = time to required speed in seconds

3. Heat Generated

tc = chordal thickness o = heat generated


o = pitch diameter

N = number of teeth

6. Chordal Thickness of Tooth when Outside Diameter is Special 2 2

WR = total inertia, Ib-ft


N 1 = final rpm

N2 = initial rpm

T, = clutch torque, ft-\b

t = circular thickness T1 = torque load, tt-lb

PROBLEM 1
7. Chordal addendum A car moving at 60 mph when the brakes are suddenly locked and the car begins to
skid. If it takes 2 seconds to slow the car to 30 mph, at what rate is it being
decelerated, how long the car comes to a halt, and how far will it have traveled?

.i-j'·"[-RI
a = addendum
va = 60 (5280/60) = 88 ft/s

VI = 30 (5280/60) = 44 ft/s

A. VI = Va + at

44 = 88 + a (2)

a = -22 ft/s 2

254 MGchineries Machineries 255


2
A. 4.23 rn/s" C. 3.23 m/s
B. Va 44 ft/s
'= 2
B. 5.23 m/s" D.2.19m/s
VI 0 (stop)
'=

VI '= Va + at
SOLUTION
o '= 44 + (-22) t
t '" 2 sec
T '= total time W,= 50 (9.81) '= 490.5
T '= 2 + 2 '= 4 sec W
T '= W + -a
g
c. S '= Va t
2
+ Yz a t
S '= 88 (4) + 112 (-22) (4/ 600 '= 490.5 + 490.5 a
S '= 176 ft 9.81
a '= 2.19 m/s"
PROBLEM 2
A flywheel on a press rotating at 120 rpm is slowed to 102 rpm during a punching
process that requires 34 sec for the punching portion of the cycle. What angular PROBLEM 5
deceleration the flywheel experience? A flywheel has a diameter of 3 ft and weighs 1000 pounds. What torque must be
A. -2.52 rad/s" C. -1.23 rad/s" applied, neglecting bearing friction, to accelerate the flywheel at the rate of 100
B. -5.45 rao/s" D. -8.45 rad/s" revolutions per minute .per second?
A. 265.80 ft-lbs C. 365.80 ft-Ibs
B. 565.80 tt-Ibs D. 665.80 ft-Ibs
II'm!!Dm
SOLUTION
WI + a. t
'= Wa
Wa '= 120 (2rr/60) '= 12.57 rad/s
a. = 100 rev/sec-min (2rr/60) = 10.47 rad/s"
w, '= 102 (2rr/60) '= 10.68 rad/s
10.68 '= 12.57 + a. (3/4) 2 W 2
T = (Yz m r ) (112 -
0.= r ) a.
a. '= -2.52 rad/s" g
T = \/2 (1000/32.2)(3/2)2 (10.47)

PROBLEM 3 T = 365.80 ft-lbs


A 100 Ib body is being hoisted by a winch, the tension in the hoisting cable being kept
constant at 110 lb. At what rate the body is accelerated? PROBLEM 6
C. 2.22 ft/s 2
2
A. 1.22 ftls A flywheel has a diameter of 1.5 m, and a mass of 800 kg. What torque is needed to
2
B. 3.33 ft/s D. 4.44 ft/s 2 produce an angular acceleration of 100 revolutions per minute, per second?
A. 5356 N.m C. 3356 N.m
II'm!!Dm B. 4356 N.m D. 2356 N.m

T '= W + FR 4-NiIDi1m
W
T '= W + -a
g a. = 100 rev/sec-min (2rr/60) 10.47 rad/s"
T = (112 m (') a.
100
110 '= 100 + - - a T = 112 (800)( 1.5/2/ (10.47)
32.2 T = 2356 N.m
2
a '= 3.22 ftls

PROBLEM 7
PROBLEM 4 A 12 inch cube of steel weighing 490 lbs is being moved on a horizontal conveyor belt
A body of mass 50 kg is being hoisted by a winch, and the tension in the cable is 600 at a speed of 6 miles per hour (88 ft/s) What is the kinetic energy of the cube?
N. What is the acceleration? A. 323.45 ft-Ibs C. 534.34 ft-Ibs
B. 489.22 ft-Ibs 0 633.34 ft-Ibs
256 Mo ciiin er ie« Moc h i fl cr ies 257
SOLUTION SOLUTION

W 2 PE = potential energy
KE = 1'2 -V Potential energy = Work produced
g
PE = W
KE = '12 _490 (8.8)2 = 489.22 tt-lbs m (z + d) = F x d
32.2 100 (10 + 0.3) = F (0.3)
F = 3433.33 kg (0.00981) = 33.68 KN
PROBLEM 8
If a cube of mass 200 kg is being moved on a conveyor belt at a speed of 3 mis, what
is the kinetic energy of the cube? PROBLEM 11
A. 800 J C. 850 J A 1000 Ib block is pulled up a 2-degree incline by a cable exerting a constant force F
B. 900 J D. 950 J of 600 Ibs If the coefficient of friction between the block and the plane is 0.5, how
fast will the block be moving up the plane 10 seconds after the pun is applied?
A. 12.4 mph C. 17.3 mph
Em!!ImD B. 10.2 mph D. 14.3 mph

KE = % mv 2 % 200(3)2 w..ie],IIIWl'
KE = 900 J R = F - P = -W(~ cosu. + sin o.)
R = 600 - 1000(0.5 cos2° + sin 2°)
R = 600 - 535 = 65.41 Ibs
PROBLEM 9 W W
A pile driver weighing 200 pounds strikes the top of the pile after being fallen from a Rt = --vr--vo
g g
height of 20 ft. If forces a pile into the ground a distance of 1'2 foot. What is the
average force of the blow? 1000 1000
65.41 x 10 = ~-vf -~-(O)
A. 8,2001bs C. 8,4001bs 32.2 32.2
B. 8,6001bs D. 8,8001bs v, = 21.06 ftls = 14.3 6 miles/hr

Em!!ImD PROBLEM 12
A 500 kg block is pulled up a 2 degree incline by a constant force F of 4 KN. The
PE = potential energy
coefficient of friction between the block and the plane is 0.5. How fast will the block
Potential energy = Work produced
be moving 10 seconds after the pull is applied?
A. 27.6 rn/s C 30.4 rn/s
PE = W
B. 34.2 m/s D. 38.3 m/s
m (z + d) = F x d
d = Y2 It = 0.5 ft
200 (20 + 0.5) = F (0.5) Em!!:immI
F = 8200lbs
R = F - P = -mg(~ coso. + sin u.)
R = 4000 -500 x 9.81 (0.5 cos2° + sin 2°) 1378 N 1.378 KN
PROBLEM 10 R t = mv, -mv o
A pile driver of mass 100 kilograms falls 10 meters and moves the pile a distance of 1378 x 10 = 500(vr - 0)
0.3 m. What is the average force of the blow? Vj= 27.6 m/s
A. 23.45 KN C. 5423 KN
B. 33.68 KN D. 43.23 KN
258 Machineries Muchincries 259
PROBLEM 13 PROBLEM 16
A flywheel having a moment of inertia of 25 lbs-ft-sec" is revolving with an angular A casting of mass 150 kg is lifted 4 meters in 15 seconds by means of a crane. What
velocity of 10 radians per second when a constant torque of 20 Ibs-ft is applied to is the power?
reverse its direction of rotation. For what length of time must this constant torque act A. 392 watts C. 456 watts
to stop the flywheel and bring it up to reverse speed of 5 radians per second? B. 523 watts D. 634 watts
A. 12.34 sec C. 18.80 sec
B.14.34sec D.16.34sec ID.IDIilmI
P '= W xv = W x (Sit)
SOLUTION P '= (150 x 0.00981)(4115)
P '= 0.392 kw = 392 watts
Angular impulse '= Change in angular momentum

To t = J (Wf - wo) PROBLEM 17


20 t '= 25 ([10 - (-5)] A cast iron flywheel with a mean rim radius of 9 inches, is rotated at a speed of 800
t '= 18.8 seconds rpm. If the weight of the nrn is 20 Ibs, what is the centrifugal force?
A. 3169.341bs C. 3269.341bs
B. 3262.34 Ibs D. 4269.34 Ibs
PROBLEM 14
2
A flywheel with a moment of inertia of 20 kg_m is revolving with an angular velocity of
10 radians per second when constant torque of 30-N-m is applied to reverse its
ID.IDIilmI
direction of rotation. For what length of time must the constant torque act to stop the
flywheel and bring it to reverse speed of 5 radians per second? Wv 2
Fe = - . -
A 5sec C.10sec gR
B.12sec D.16sec V = 2 It R N = 2 It (9/12)(800/60) = 62.83 ftJs

SOLUTION Fe = 20 (62.83)2 = 3269.341bs


32.2(9 I 12)

Angular impulse '= Change in angular momentum

To t '= J (Wf - wo) PROBLEM 18


30t '= 20([10-(-5)] A steel pulley with a mean rim radius of 120 mm is rotated at a speed of 1100 rpm. If
t = 10 seconds the mass of the rim is 5 kilograms, what is the centrifugal force?
A. 5344.50 N C. 6432.30 N
B. 7961.50 N D. 8734.56 N
PROBLEM 15
A casting weighing 300 Ibs is to be lifted by means of an overhead crane. The SOLUTION
casting is lifted 10ft in 12 seconds. What is the horsepower developed?
A. 0.45 hp C. 6.5 hp mv 2
B. 8.5 hp D. 95 hp Fe = --
R

Em!!DrmI V = 2 It RN = 2 It (0.12)(1100/60) 13.82 mls

Fe = 5(13.82)2 = 796150 N
mv m(S/t) 0.12
HP '= 550 '= - 550

300 (10/12) 0.45 hp PROBLEM 19


-5~
'=
HP = A round bar made from SAE 1025 low Carbon steel is to support a direct tension load
of 50,000 Ibs. Using a factor of safety of 4, and assuming that the stress
260 Machineries
concentration factor k = 1. What is the suitable standard diameter. Yield stress is Machineries 261
40,000 psi.
SOLUTION
A. 3 Yz in C 2 in
1),
B. 2 9/16 in D. 1 Y2 in
Using the formula of strength of tapered pin:
SOLUTION 8 =0 2~?7T
Dd 2
F T = Fxr = 200 x 800 = 160,000 N.mm
8 a ll
A
8 = _1.27 (160,000) = 406 N/mm 2 or Mpa
8 a ll s, I F8 = 40,000/4 10,000 psi 50 (1 0)2
10 000 = 50,~00
, A PROBLEM 22
A = 5 in If a shaft of 50 mm diameter is to transmit power of 12 kw at a speed of 500 rpm, find
2
the mean diameter of the pin for a material having a safe unit stress of 40 N/mm .
2
A = rr/4 D A10mm C.12mm
B. 14 mm D. 16 mm
2
5 = nl4 D

D = 2.523 in &1·l!iiiGD
Therefore, use D = 2 9/16 (standard)

PROBLEM 20
d = 110.3 ~~-S' mm
A lever secured to a 2 inches round shaft by a steel tapered pin (d = 3/8") has a pull
of 50 Ibs at a 30 in radius from shaft center. Find the unit working stress of the pin. 112,000
A 6770 psi C. 6790 psi
d = 11 0.3 ~ 500 x 50 ~ 40 12.09 mm
B. 7433 psi D. 5234 psi

EmmmD PROBLEM 23
Find the thickness of metal required in the hemi-spherical end of a cylindrical vessel,
Using the formula of strength of tapered pin: 2 feet in diameter, subjected to an internal pressure of 500 psi. The material is mild
steel and a tensile stress of 10,000 psi is allowable.
8 = 2·27~ A. 0 10 in C. 0.3 in
Dd 2
B. 0.5 in D. 0.7 in

T = Fxr
SOLUTION
T = 50 x 30 = 1500 in-lb
8 =0 2.
27 (1500)
Using the formula of spherical shell because of two hem i-spherical ends.
2(3/8)2
8 =0 6770 psi P o,
8
4 t

PROBLEM 21 P o, 500 x 2 x 12
0.3 inch
A lever secured to a 50 mm round shaft by a steel tapered pin ( d =0 10 mm) has a pull 48 4 x 10,000
of 200 N at a radius of 800 mm. Find the working stress on the pin.
A. 32.3 Mpa C. 45.20 Mpa PROBLEM 24
B. 40.6 Mpa D. 56.34 Mpa Find the thickness of metal required in the hem i-spherical end of a cylindrical vessel,
2
750 mm in diameter, subjected to an internal pressure of 3 N/mm . The material is
2
mild steel and a tensile stress of 70 N/mm is allowable.
262 Machineries Machineries 263
A.4mm C.6mm A. 78 mm C. 93 mm
B.2mm D. 8 mm B. 81 mm D. 85 mm

Wf':rv.:m:D SOLUTION

Using the formula of spherical shell because of two hem i-spherical ends. For main shaft:
3
D N
P o, P =--
S 80
4 t
3
P o, 3 x 750
~~= D (500)
--- 8.04 mm 0.746 80
4S 4 x 70
D = 3.18 in = 80.78 mm

PROBLEM 25 PROBLEM 28
What would be the diameter of the line shaft to transmit 10 horsepower if the shaft What power would a short shaft, 50 mm in diameter, transmit at 400 rpm?
makes 150 rpm? A. 50 kw C. 55 kw
A. 9/16 in C.29/16in B.60kw D.65kw
B. 39/16 in D. 1 9/16 in
SOLUTION
SOLUTION

D3 N
D3 N P
P = ~- 38
53.5
3
(50/25.4)3 (400)
10 = D
(150) P 80.29 hp = 60 kw
38
53.5
D = 1.53 in = 1 9/16 in (standard)
PROBLEM 29
Find the torsional deflection for a solid steel shaft 4 inches in diameter and 48 inches
PROBLEM 26 long, subjected to twistinq moment of 24,000 in-lb.
What horsepower would be transmitted by a short shaft, 2 inches in diameter, A. 0.122 deg C. 0.23 deg
carrying but two pulleys too close to bearings if the shaft makes 300 rpm? B. 0.052 deg D. 0.43 deg
A. 60 hp C. 65 hp
B. 75 hp D. 70 hp
IEm!!Im3
SOLUTION
Using the derived formula for deflection:

3
D N 584 TL d
P = 38 (X = -4~-' eg
D G
(2)3 (300) = 63 hp where: ex= angular deflection, deg
P = - 38 T = torque, in-Ib
L = length, in
PROBlEM 27 D = shaft diameter, In
G = 11,500 psi (for steel)
What would be the diameter of power-transmitting shaft to transmit 150 kw at 500
rpm?
264 Machineries Much inL'ries 265
A = (H - 0 - 2Y)/2
a = 584 (24,009)( 48 2= 0.23 degree
(4)4 (11,5000,000)
L = length of wire rope, ft
L = (A + 0) x A x B x K
PROBLEM 30 Where:
Find the torsional deflection of a solid steel shaft, 100 mm in diameter and 1300 mm B = width between drum flanges, in
long, subjected to twisting moment of 3 x 106 N-mm. The torsional modulus of o =' diameter of drum barrel, in
elasticity is 80,000 N/mm 2 . H = diameter of drum flanges, in
A. 0.122deg c. 0.234 deg K = factor from table
B. 0.285 deg O. 0.543 deg Y = depth no filled on drum

Em!!ImD A = (30 - 18 - 0)/2 = 6 in


L = (6+18)x6x24x0.741 = 2560ft

584 TL 584 (3 x 10 6 )(1300)


ex 0.285 degree
[)4G (100)4 (80,000) PROBLEM 33
Find the bending stress and equivalent bending load due to the bending of a 6 x 19
PROBLEM 31 (Fiber core) wire rope of % in diameter around a 24 in pitch diameter sheave.
A. 14561bs C. 19831bs
Find the diameter of steel line shaft to transmit 10 hp at 150 rpm with a torsional
B. 1590 Ibs O. 1763 Ibs
deflection not to exceeding 0.08 degree foot length.
A. 0.834 in C. 1.23 in
B. 3.234 in o. 2.35 in E:mimD
Em!!ImD For 6 x 19 Fiber Core rope:

d., = 0.063 d = 0.063 (0.5~ = 0.0315 in


Shaft diameter for 0.08 degrees per foot of length of shaft deflection. 2 2
A = 0.450 d = 0450 (0.5) = 0.101 in
E = 12,000,000 psi (average value)
For English units:

0=4.6 {f Sb
E dw
D
= (12,000,000) (0.0315)
24
15,750 psi

Where: 0 = diameter, in Pb Sb A = 15,750 (0.101) = 1590 Ibs


T = torque, in-Ib

-
P = horsepower
N = speed, rpm PROBLEM 34

m
Find the breaking load in pounds of a wrought iron chain crane if the diameter of bar
10 . from which links are made is 2 inches.
o = 4.6 4__ = 2.35 Inches A.216,000Ibs C.316,000Ibs
150
B. 416,0001bs O. 516,0001bs
PROBLEM 32
Find the length in feet of 9/16 inch diameter rope required to fill a drum having the &i-)''tmD
following dimensions: B = 24 in, 0 = 18 in, H = 30 in. (K = 0.741) 2
A.1873ft C.2874ft W = 54,000 D
B. 2560 ft O. 2645 ft
Where: W = breaking loads, lbs
4i·)!!ImD o = diameter of bar, in
W = 54,000 (2)2 = 216,0001bs
A = depth of rope space on drum, in
266 Machineries Machineries 267
PROBLEM 35 SOLUTION
The specific gravity of cast iron is 7.2. Find the weight of 5 cubic inches of cast iron.
A. 1.1 Ibs C. 1.2 Ibs Using the relation of D and N:
B. 1.31bs D. 1.4lbs
D1 Nl = D2 N2
Em!!ImmI 24(100) = D2 (600)
Weight of any material = 0.0361 (SG), Ib
02 = 4 in.
Weight of any material = 0.0361 (7.2) = 1.2996 Ib
PROBLEM 39
PROBLEM 36 How many Y2 inch diameter set-screws would be required to transmit 3 horsepower at
a shaft speed of 1000 rpm if the shaft diameter is 1 inch.
The weight of a cubic inch of gold is 0.697 lb. Find the specific gravity.
A. 13.45 C. 17.23
B. 15.34 D. 19.31 A. 1 C.2
B. 1.5 D.2.5
&-,e],'''[.];''
SOLUTION

Weight of any material = 0.0361 (SG)


Holding Power of Set Screw:
0.697 = 0.0361 (SG)
P _ ONd2 .3
SG = 19.31 - ,hp

PROBLEM 37 P '" 1(1000)(1/2)2.3 = 4.1 hp


If the diameter of driving pulley is 15 inches and its speed, 180 rpm. The diameter of 50
driven pulley is 9 inches. Find the speed of driven pulley. 3
A. 100 rpm C. 200 rpm
No. of set screw = -4.1 = 0.731
B. 300 rpm D. 400 rpm Use 1 set screw

Em!!ImmI
PROBLEM 40
Using the relation of D and N: How many 3/8 inch diameter set-screws would be required to transmit 3 horsepower
at a shaft speed of 1000 rpm if the shaft diameter is 1 inch.
D1 N1 = D2 N2 A. 1 C. 1.5
B.2 0.3
15(180) = 9 (N2)
SOLUTION
N2 = 300 rpm
Holding Power of Set Screw:
PROBLEM 38
If the diameter of driving pulley is 24 inches and its speed, 100 rpm, and the driven 23
pulley is to rotate 600 rpm, find the diameter of driven pulley. P = DNd .
A. I in C. 2 in 50 ,hp
[1. 3 in D. 4 in
268 Machineries Much incrips 269
p = 1(1 000)(3/8)2.3 = 2.1 hp
50 26(N 1) = 4(800)

No. of set screw = -3 =1.428 N1 = 123 rpm


2.1
Use 2 set screws
PROBLEM 44
PROBLEM 41 If the dnving gear has 20 teeth and rotates 80 rpm, and the driven gear has 40 teeth,
What is the working strength of a 1 inch bolt which is screwed up tightly in a packed then the speed of the driven gear is:
joint when the allowable working stress is 10,000 psi? A.10rpm C.20rpm
A. 3000 Ibs C. 3500 lbs B. 30 rpm O. 40 rpm
B. 4000 Ibs D. 4000 Ibs

Ei1!!mmI E'm!!IilmI
Using the relation of T and N:
W = workinq strength of bolt
T1 N, = T 2 N2
W = S1 (0.55d 2 ~ 0.25d)
20 (80) = 40 (N2)
W = 10,000[0.55(1)2 - 0.25(1)] = 3000 lbs
N2 = 40 rpm
PROBLEM 42
42. If the diameter of driven pulley is 36 inches and its required speed, 150 rpm, and PROBLEM 45
the speed of driving pUlley is 600 rpm, then the diameter of driving pulley is: If the pitch diameter of the driver is 8 inches, its speed, 75 rpm, and the pitch
A. 5in C.7in diameter of the driven gear, 20 rpm is:
B. 9 in O. 11 in A. 20 in C. 30 in
B.40in 0.50in
. SOLUTION
SOLUTION
Using the relation of 0 and N:
Using the relation of 0 and N:
0, N, = 02 N2 0 1 N, = O2 N2

0, (600) = 36 (150) 8 (75) = O2 (20)


0, = 9 inches O2 = 30 inches
PROBLEM 43 PROBLEM 46
If the diameter of the driven pulley is 4 inches, its required speed, 800 rpm, and the 2,
If the inertia is 80 Ib-ft and the speed of the driven shaft is to be increased from 0 to
diameter of the driver is 26 inches, then the required speed of the driver is: 1500 rpm in 3 seconds, find the clutch starting torque in Ibs.
A. 112 rpm C. 123 rpm A. 100 rt-Ib C. 120 tt-lb
B. 134 rpm O. 145 rpm B.110ft-lb 0.130ft-lb

SOLUTION
E'm!!IilmI
Using the relation of 0 and N: Te = clutch starting torque, ft-Ibs
T = Ix,A.~
0, N, = 02 N2 c
308t
270 Machineries Mac iiin eti es 271
2
I = W R = moment of inertia. Ib-tt 2 f = coefficient of friction
W = weight, Ib P = force in Ibs per inch of width
R = radius of gyration, ft C = 3.1416 (150) (0.2) = 0.00024
tlN = final rpm - initial rpm 33,000 x 12
t = time to required speed in seconds
HP = 10 x 200 x 2 x 0.00024 = 0.96 hp

r, = 80308x 1500
(3)
= 130 tt-Ib
PROBLEM 49
A body weighing 28 Ibs rests on a horizontal surface. The force required to keep it in
PROBLEM 47 motion along the surface is 7 Ibs. Find the coefficient of friction.
If the inertia is 80 lb-ft", and the speed of the driven shaft is to be increased from 0 to A. 0.12 C. 0.25
1500 rpm in 3 seconds. Calculate the heat generated for each engagement if clutch B. 0.45 D. 0.85
starting torque is 10 in-lb.
A. 20.50 Btu
B. 41.50 Btu
C.
D.
30.50 Btu
51.50 Btu
4-t.l'iimD
Fr = frictional resistance
E·] "imB Fr = f N = f W
7 = f (28)
Q = heat generated f = 0.25.
2 2 2
Q = Tc x WR X (N1 - N2 )
Btu PROBLEM 50
(Tc - T1 ) x 4.7 x 10 6 Measurement M = 3.495 inches at the gaging notch of a 3 in pipe thread and the wire
2 2
WR = total inertia, Ib-tt diameter is 0.07217 in. Find the pitch diameter. (P = 0.125 in)
N1 = final rpm A. 1.23 in C. 2.34 in
N2 = initial rpm B. 0.34 in D. 3.39 in
Te = clutch torque, tt-Ib
T 1 = torque load, tt-Ib SOLUTION
Q = 130X80x[(1500)2_ 02]
41.50 Btu
(130-10)x4.7x10 6 M = E -(0.86603x P)+3 x W
1.00049
PROBLEM 48 E = effective pitch diameter, in
Find the horsepower transmitted by a pair of friction wheels; the diameter of driving
wheel is 10 inches, and it revolves at 200 rpm. The width of the wheel is 2 inches. 3.495 = E - (0.86603 x 0.125) + 3 x 0.07217
The force per inch width of face is 150 pound and the coefficient of friction is 0.20. 1.00049
A. 0.96 hp C. 1.2 hp E = 3.3885 in
B. 023 hp D. 1.6 hp

SOLUTION PROBLEM 51
Find the tooth thickness on the tooth circle of a 14 Y2 degree full depth tooth of 12
C = 3.1416 P x f diametral pitch.
A. 0.131 in C. 0.455 in
33,000 x 12
B. 0.234 in D. 0.864 in
HP = 0 xNxW xC

Where: FM'rmmII
o = diameter of friction wheel
N = speed, rpm 1.57
Tooth Thickness = 1.5708/P
W = width of face, in 12
272 Mach in eties Machineries 273
Tooth thickness = 0.1309 in
t = Pc - e tan <j>
2
PROBLEM 52
Find the tooth thickness on the tooth circle of a 20 degree full depth involute tooth t =Q.:6283 -(0.2746xtan14.5 0 ) 0.2432 in
having a diametral pitch of 5. 2
A. 0.863 in C. 0.108 in
B. 0.314 in D. 0563 in PROBLEM 55
A pinion has 15 teeth of 3 diametral pitch. Find the chordal thickness at the standard
SOLUTION pitch diameter.
A. 0.653 in C. 0.863 in
B. 0.523 in D. 0.234 in
Tooth Thickness = 1.5708/P
1.57
Tooth thickness
5
Em!!ImD
Tooth thickness = 0.3142 in
Chordal Thickness of Tooth when Outside Diameter is Standard
PROBLEM 53
The outside diameter of a pinion having 10 teeth of 5 diametral pitch and pressure tc = D sin (90° 1N) = 5 sin [9
10:
J = 0.5226 in
angle of 14 V2 degrees is to be increased by 0.2746 in. The circular pitch equivalent
to 5 diametral pitch is 0.6283. Find the arc tooth thickness at the standard pitch
diameter.
A. 0.385 in C. 0.543 in
B. 0.863 In D. 0.534 in

SOLUTION

Circular Thickness of Tooth when Outside Diameter has been Enlarged


t = tooth thickness
P
t = ~ + e tan o
2

t = Pc + e tan ¢ = (0.6283/2) + 0.2746 tan14.5° = 0.3852 in


2

PROBLEM 54
The outside diameter of a gear having a pressure angle of 14 Y2 degrees is the be
reduced by 0.2746 in or an amount equal to the increase in diameter of its mating
pinion. The circular pitch is 0.6283 in. Determine the circular tooth thickness at the
standard pitch diameter.
A. 0.434 in C. 0.243 in
B. 0.843 in D. 0.672 in

SOLUTION

Circular Thickness of Tooth when Outside Diameter has been Enlarged


t = tooth thickness
274 Sit WU ioua I Problems 275
Situat ion ul Problems
A.1011.47Ibs C. 1211.47
B 1111.47 D. 1311.47
6. the load at slack side
A.5591bs C 5791bs

P r ~
, R··'
.l-~/~ \J ct . . ..
~
"t, 'T""t i. 2'.<'~
}>i SOLUTION:
B. 5691bs D. 589 Ibs

S "ltY'"'Ur::A"'T'"}'1""0/''N'"A'''
s ., ,
,,'co;' , .'. ;1 J .,t,>, L;
i~.,.
.",,? ,,'.. {{<} t ,,'
Driven
c = 4m

P'ROB,LE~MiS
1. p

SITUATIONAL PROBLEM 1 (ME Board April 1981)


An open belt drive connects a 450 mrn driving pulley to another driven pulley 1000
2. v ==
mm in diameter. The belt is 300 mm wide and 10 mm thick. The coefficient of friction
of the belt drive is 0.30 and the mass of the belt is 2.8 kg per meter of belt length.
Other data are as follows:
Center distance between shafts = 4 meters
Maximum allowable tensile stress of the belt = 1500 kpa
3. e 180 0 - 2 sin-t D22- 1
CD
l
Speed of driving pulley = 900 rpm
Determine: e 180 - 2sin-{1000-450t172°X_~=3 rad
L 2(4000) J 180 0
1. the belt density in lb/in"
A. 0.014 C, 0.034 4. Sw = working stress
B, 0.024 D. 0.044 14.7 psi 21 .
15 00 k pa x = 7.52 PSI
101.325kpa
2. the belt speed in ftlsec
A. 63.34
B. 65.03
C. 67.37
D. 69.57
550 hp
b t - -------'-___=_-
- v(Sw -12pv 2 /g) ef8 -1
[e f8
J
3. the angle of contact, rad. 2
12 p v = 12(0.034)(69.57)2 = 60.836 psi
A. 3 rad C. 5 rad F2
B. 4 rad

4. the power transmitted, kw


A. 37.45
D. 6 rad

C. 43.24
g

300 1 10
[ 25.4 25.4
32.2

'I I
r 550 hp
IL 69.573(217.52 - 60.836)
II
Jl eO
eO 3(3)
3(3) -1
l
B. 40.80 D. 48.34 hp = 54.7 = 40.8 KW

5. the load at tight side 5. F 1 = b t Sw


276 Sit [lot irmal Problems
SittlO{iOfWI Problems 277
300 10
F1 = - - x 217.52 ~ 1011.47 Ibs
25.4
X --
25.4 J
For hollow shaft: mH = (I -71: (Do2 ... Dj2) L w
,4 .
6. (F, - F2) V = 550 hp . . 1
Equating the two equations: mH = - m,
(1011.47 - F2) (69.573) = 550 (54.70) 2
F2 = 5791bs
-71: (D 2 2) L w =
- D) -21[71:-4 d2\)I L w
[ 4 0,
2 2
SITUATIONAL PROBLEM 2 (ME Board April 1981) D0 - D,2 = 6.125 or D,4 = (D 0 - 6.125l Equation 2
A solid transmission shaft is 3.5 inches in diameter. It is desired to replace it with a Equate equation 1 and 2.
4 2
hollow shaft of the same material and same torsional strength but its weight should D 0 - (D 0 - 6.125)2 = 42.87 Do
2
only be half as much as the solid shaft. Find: Simplify: D 0 - 3.5 Do - 3.0625 =0
1. the outside diameter of the shaft
A. 4.23 in C. 7.38 in
B. 5.34 in D. 9.30 in - (·-3.5) ± A-3.5)2 - 4(1)(-3.0625) = 4.225 in = 107 mm
Do =
2(1)
2. the inside diameter of the shaft
A 1.92in C. 3.42 in 2. Solving for the inner diameter:
2
B. 2.56 in D. 4.03 in Do - D,2 = 6.125
2
(4.225)2 - D , = 6.125
3. the volume of the hollow shaft per foot length. D, = 3.424 in = 87 mm
3
A 53.23 in C. 60.30 in 3
3
B. 57.74 in D. 63.48 in 3 3. Solving for hollow shaft volume
4. the shearing stress of hollow shaft if it is used to transmit 150 kw at 600 rpm, Mpa. L = 1 ft = 12 in
A 12.84 C. 16.23 71: 2 2 '\ 3
B. 14.56 D. 17.63 VH = - (Do - D, ) I L .rr. ((4.225)2 - (3.424)2) (12) 57.74 in
[ 4 ) 4

SOLUTION: 4. P=2TTTN
150 = 2 TT (T)(600/60)
T = 2.387 KN-m
1. For solid shaft: S = 16 T
S= 16 T Do = 16 ~387) (0.107) = 1762827 kpa = 17.63 Mpa
71: d3 4 4 -t::- 4 ~ ' .
71: (Do - D j ) 7I:l(0.107) - (0.087) J
16 TD o
For hollow shaft: S = -~4--4-

71: (Do - Dj )

16 T 16 T Do SITUATIONAL PROBLEM 3 (ME Board April 1981)


Equating the stress: --3 = ----4- 4 A double threaded right handed worm gear transmits 15 hp at 1150 rpm. The pitch of
71: d 71: (Do - Dj )
the worm is 0.75 inches and pitch diameter of 3 inches. The pressure angle is 14 Yz 0

Do 4 _ DI4 = d3 D-o and the coefficient of friction is 0 12. Find:


1. the lead angle
Do4 - D,4 = (3.5) 3 D0
A. 1.63° C. 7.23°
D0 4 _
D;4 = 42.87 Do Equation B. 3.83° D. 9.043°

2. the normal pressure angle

~
= ~-
For solid shaft: w
V
or m = Vw

w = density of shaft material


L = length of shaft
~_:~>'-~',~-~ ;~"--
.- - - - ,; -.-
I A 10.327°
B. 12.327°
C. 14.327°
D. 16327°

rn, = l~d2 J L w
I
3 the worm gear efficiency
A. 55.13%
B. 50.23%
C 58.34%
D 60.34%
278 Sit liCIt inned Problems 279
Sit ua t ional Problems
F v
4. the power transmitted to gear (output) Powe r = -""'---"'-
A. 6.39 hp C. 1027 hp 550
B. 8.27 hp D.13.47hp Fw (15.053)
15
550
5. the tangential force on gear Fw = 5481bs
A. 1,798 Ibs C. 1,998 Ibs
B. 1,898 Ibs D. 2,3981bs
Fw sin o, 548(sin14.327°)
7. Fs
6. the tangential load on worm cos <l>n sin x + f cos x cOS14.37°)(sin9.043°) + (0.12)(cos9.043°)
A. 5481bs C. 5681bs Fs = 501 Ibs
B. 5581bs D. 5781bs

7. the separating force SITUATIONAL PROBLEM 4 (ME Board April 1981)


A. 4031bs C. 501 Ibs A band brake is installed on a drum rotating at 250 rpm, and a diameter of 900 mm.
B. 453/bs D. 5671bs The angle of contact is 1.5IT radians and one end of the brake band is fastened to a
fixed pin while the other end to the brake arm 150 mm from the fixed pin. The
SOLUTION: coefficient of friction is 0.25 and the straight brake arm is 1000 mm long and js placed
perpendicular to the diameter bisecting the angle of contact. Determine:
1. For double thread, S 1. the torque transmitted.
L = 2 P = 2 (0.75) = 1.5 in A. 2.73 KN-m C. 4.16 KN-m
Lead 1.5 B. 3.92 KN-m D. 1.91 KN-m
tanx = ~~=--=0.159

IT Ow IT (3) 2. the tension at the slack side of the brake drum.


x = tan' 0 159 = 9.043° A. 1.89 KN C. 3.28 KN
B. 2.63 KN D. 4.93 KN
2. tan <!>n = tan <!> cos x = (tan 14.5°) (cos 9.043°) = 0.255 3. the tension at the tight side of the brake drum.
10.255 A. 5.33 KN C. 7.83 KN
<!>n = tan· = 14.33°
B. 6.14 KN D. 8.38 KN

3. e = tanx l cos<bn-t..t an x
cos o., tanx + f
j =(0.159)l' cos14.327-0.12(0.159)
(cos14.327)(0.159) + 0.12
J
4. the minimum force in Newtons applied at the end of the brake arm necessary to
stop the drum if 50 kw is being absorbed.
A.125N C.175N
e =0.5513 = 55.13% B. 150 N D. 200 N
5. A steel band with a maximum tensile stress of 55 Mpa and 3.0 mm thick will be
used. What should be its width in millimeters?
4. e ~- 0.5513 £!Q. Po = 8.27 hp A. 37 mm C. 41 mm
Pi 15 B. 39 mm D. 43 mm

1150) SOLUTION:
5. v g = ve Iocrty
. a f gear = l(1.51
- -~- = 237 ft / sec
12 60
1. Power = 2 IT T N
Ft vg 50 = 2 IT (250/60)
Power = -- F
550 T = 1.91 KN-m
2. T = (F 1 - Fz) r
8.27 = ~t (2.37)
0.90)
550 1.91 = (F 1 - F)
Z (--
Ft = 1,898 Ibs 2
F1 - Fz = 4.24 KN
6. Vw = IT 0 N = velocity of worm (~r2.150 1=15.053 fUsec
IT l12 . 60 ) 5. = efO = e0 2 5 (1 5 1T ) r-: 3.25
F2 1000 mm
SitUG [lOTI cd Pr ob! ems 281
280 Si[IU~1 i()llGI Problems

F 1 == 3.25 F2
Om == 0, + d == 2.5 + d
Substitute:
l
63,000 == 8 (1.25) (825 (2.5 + dl
3.25F 2 - F2 == 4.24
It d
F2 == 1.89 KN

Try: d == V2 in

3 F1 == 3.25F2 == 3.25 (1.89) == 6.14 KN 63,000 == 8 (1.?5) (825) (2.5 !.Q.501

It (0.50)3

4. Summation of moments about the fixed end equal to zero.


63,000 == 63,000

F2 sin45° (150) - F (1000) == 0


Therefore: d == V2 inch

F == 0.106F2 == 0.106(1.89) == 0.20 KN == 200N

Om == 2.5 + 0.5 == 3 in
5. S == 5_ == -'i C == Om ==~=6vv

A tb d 0.5

55000==-6~ F
, (0.003)(b)
3. k == .­
b == 0.037 m == 37 mm
Y
400 == 825_

SITUATIONAL PROBLEM 5 (ME Board October 1981)


Y == 2.0625 in

A helical compression spring has a scale of 400 Ibs/inch, an inside diameter of 2.5 3
inches, a free length of 8 inches and with squared and ground ends. Material is to be 8FC n

chrome vanadium steel with working stress of 63,000 psi and G == 10,800,00 psi. For Y==GT

a load of 825 lbs, and for average service, Whaal factor, k == 1.25, determine:
2.0625 == ---.!3.~25) (6)3 (n)
_

1. the standard size wire diameter (10,800,000)(0.5)

A. % in c. 3,4 in n == 7.8125 coils

B. V2 inch D. 1 % in
4. For square and ground end,
2. the spring index. Solid length == (n + 2) d == (7.8125 + 2)(0.5) == 491 in
A. 6 C. 8
B. 7 O. 9 5. At solid height,
ys == FL - SL == 8 - 4.91 == 3.09 in
3. number of active coils Fs == force at solid == k Ys == 400 (3.09) == 1237.6 Ibs
A. 8.23 C. 534 Ss == 8 K Fs Om = ~ (1.25) (1237.6)(3~ == 94 546 psi

B. 1123 O. 7.81 It d3 It (0.5)3 '

4. the solid height


A. 2.83 in C. 4.91 in SITUATIONAL PROBLEM 6 (ME Board October 1981)
B. 3.26 in O. 6.34 in A 2 ]/2 inches double square thread with two threads/inch is to be used. The frictional
radius of the collar is 2 inches and the coefficient of friction are 0.10 for the threads
5. the stress at solid height and 0.15 for the collar. The velocity of the nut is 10ft/min. For a lifting load of 4000
A. 94,546 pSI C. 96,735 psi lbs, find:
B. 95,735 psi O. 97,735 psi 1. the lead angle
A. 5.23° C. 7.94°
SOLUTION: B. 6.27° O. 8.05°
2. the torque required to turn the screw
8 K F Om A. 1102.35 in-Ibs C. 1493.65 in-Ibs
1. S -------­
s- It d3 B. 1293.65 in-Ibs O. 1693.65 in-Ibs
282 Sitllnl iO/lClI Problems Sit uatioTwl Problems 283
3. the torque required to overcome collar friction SITUATIONAL PROBLEM 7 (ME Board April 1982)
A. 1000 in-Ibs C 1400 in-Ibs A 16 mm plate is lapped over and secured by fillet weld on the inside and outside to
B. 1200 in-Ibs O. 1600 in-Ibs form a penstock 1.5 m diameter. An allowable stress of 140 Mpa on the plate and an
allowable shearing stress of 90 Mpa on the throat side 01 the 14 mm fillet weld.
4. the efficiency of the screw
Determine
A. 25.58% C. 2765%
B. 26.58% O. 28.58% 1. the internal pressure that can be carried by the plate
A. 3 Mpa C. 7 Mpa
5. the power input of the screw B. 5 Mpa O. 9 Mpa
A. 1.37 hp C. 7.37 hp
B. 4.38 hp O. 9.37 hp 2. the internal pressure that can be carried by the welded joint.
A. 1.38 Mpa C. 3.38 Mpa
SOLUTION:
B. 2.38 Mpa O. 4.38 Mpa
1. Lead '= 2p
3. the thickness of the plate if the internal pressure is 2 Mpa (neglecting welded
Lead '= 2 (1/2) '= 1 in
joint).
For square thread:
A. 16.67 mm C. 14.67 mm
Om = Do - 2p = 2.5 - 2 (1 / 2) = 2.25 in B. 18.67 mm O. 12.67 mm
2 2
Lead 1 4. the thickness of the plate if the internal pressure is 20 Mpa (neglecting welded
tanx = - - = - - - =0.1415
joint).
IT Om IT (2.28)

A. 130.17 mm C. 170.17 mm
X c~ tan- 1 0.1415 = 8.05 0 B. 160.17 mm 0 190.17 mm

5. Factor of safety 01 the plate il ultimate stress is 450 Mpa.


2. T = WO m ~a~x+f 1= (4000)(2.2_5
I 11 J
0.1415+0.10 = 110235 in-Ibs A. 3 C. 5
2 l1 - f tan x 2 J l1- 0.1 O( 0.141 5) B. 4 O. 6

3. Dc = 2 rc = 2 (2) = 4 in SOLUTION: F
I
Tc = fcW(r o Hi) =~ WO,,-= 0.15(4000)(4) =1200 in-Ibs I
2 ­ 1. S =~_Oi I
I
2t I(
tan x(1 - f tan x) I PDL
4. e
f 0
140=~21­ I
tan x + 1 + (__c£ )(1 - f tan x)
2 (0.016) I
I
Om
P '= 2.99 Mpa I
e= (0.1415)[1-0.10(0.1415)J =0.2765 F
27.65% 2. From Faires, Eq. b, p.507, for welded joint:

0.1415 + 0.10 + I ~.15li4)1[1_ 01 0(0.1415)J F = 2 S L b cos45°

l 2.25
But F=~OL =P(1.5)L
5. 2 2
TT = T + Te = 1102.35+1200 = 2302.35in-lbs = 191.86ft-lbs
V,= L N ~ (1.5) L = 2 (90)(L)(0 .014) cos 45 0
10x12=(1)N
2
N = 120 rpm
P = 2.376 Mpa
P, = 2ITT~ =~IT(1~1.86)(120) =4.38 h
S= PO,
33,000 3~ ~~~ P
3.
2t

90 000 = 2.00(")i~_

. 2 t

284 Sit liUt iOllal Problems


Sit lIul iorwl Problems 285
t = 0.01667 m 16.67 mm
SOLUTION: Force
4. S= POi 1.
2t Pr essure = Area 32mm
)
F p
90000 = 20,000 ~~
, 2t 1380 C~ 7t-~32)2
.~.(0.
t = 0.1667 m 4
Checking the ratio of tiD,. F = 1.11 KN

.Q: 1667= 0.111


1.5 C = Om = Do ~d
d d
Since 0.111 is greater than 0.07, then the vessel is thick wall. 2.

t=.l2.
1 rs+~ ~ 1j=~r ~JjOO+20,000 -1j=0.19017 m
C = 115 -13 = 7.85
13
2 lVs-
t=190.17mm
P 2 l ~ 90,000 - 20,000

3n
5. FS = Su _ 450,000 8FC 8(1.11)(7.85)3(7·~=0031 m=31 mm
S - 90,000 =5 3.
y= G d - = 80,000,000(0.013)

4. Compressed Length = Free length - y = 203 - 31 172 mm


SITUATIONAL PROBLEM 8 (ME Board April 1982)
A safety valve spring having 9 - Y2 coils has the ends squared and ground. The 5. For square and ground end:
outside diameter is 115 mm and wire diameter is 13 mm. It has a free length of 203 Actual no. of coil = n + 2 = 9.5
mm. The spring must be initially compressed to hold the boiler pressure of 1.38 Mpa n = 7.5
on the valve seat of 32 mm diameter. Modulus of rigidity is taken as G = 80 Gpa. SL = (n + 2)d = (95)(13) = 1235 mm
Determine: Ys = FL - SL
1. the force acting on the spring. Ys = 203 - 123.5 = 795 mm
A. 0.811 KN C. 1.45 KN
B. 1.11 KN D.1.87KN I

3
2. the spring index 8 Fs C n
Ys =-G(j­ ( Dm = 102 rom )/1
1

A. 6.85 C. 8.85
B. 7.85 D. 9.85 c )
8(Fs)(7.85)3 (7.5)
Do = 115 mm
0.0795 = 80,000,000(0.013)
3. the coil deflection.
A. 31 mm C. 35 mm Fs = 2.85 KN
B. 33 mm D. 45 mm 4C-1 0.615 _ 4(7.85)-1 +2.: 615 =1.188

K= 4C-4"+C-- 4(7.85)-4 7.85

4. the compressed length of spring.


A. 142 mm C. 162 mm 8 K Fs Om _ 8 (1.1_88) (2.85)(0.1 02) = 400,287 kpa = 400.29 Mpa
B.152mm D.172mm
s, =--;T- - 11:(0.013)3

5. the spring stress if it is compressed to its solid length. SITUATIONAL PROBLEM 9 (ME Board April 1982)
A. 360 Mpa C. 400 Mpa The large diameter and face of the disk of a multiple disk clutch are 255 mm and 25
B. 380 Mpa D. 420 Mpa mm respectively. The helical compression spring used to engage the clutch has 9­
1/2 effective coils of 10 mm steel wire. The outer coil diameter is 80 mm. The free
length of the spring is 185 rnrn. When in place with clutch engaged, its length is 130
mm. Assuming that there are 10 pairs of friction surface in contact, that the motor
Sitllal iOl1nl Problems 287
286 Sit uat iOT1Cl1 Problems
runs at 1200 rpm, with coefficient of friction on contact surface f
,determine:
= 0.15, (G = 80 Gpa) ~ l' r()~_~.3j\ =.~ r(127.5)3 _-J1 02'~~J= 115.45 mm

rl' 3 ro 2 __ rI2 3l(127.5)2-(102.5)2

1. the axial force available for the clutch.


A. 1.69 KN C. 3.69 KN 3. T = n f Fa rj = 10 (0.15) (1.688) (0.11545) = 0.292 KN-m
B. 269 KN D. 469 KN
4. . ower = 2 TT TN = 2 TT (0.292) (1200/60) = 3669 kw
2. the mean frictional radius.
A. 125.45 mm C. 115.45 mm
B. 145.45 mm D. 155.45 mm SITUATIONAL PROBLEM 10 (ME Board April 1982)
An electric motor running at 1200 rpm drives a punch press shaft at 200 rpm by
3. the torque transmitted. means of a 130 mm wide and 8 mm thick belt. When the clutch is engaged the belt
A. 0.192 KN-m C. 0.392 KN-m slips. To correct this condition, an idler pulley was installed to increase the angle of
B. 0.292 KN-m D. 0.492 KN-m contact but the same belt and pulley were used. The original contact angle on the
200 mm motor pulley is 160 degrees. The original tension ratio is 2.4 and the net
4. the power the clutch can transmit tension is 12 N/mm of the belt width. If an increase in transmission capacity of 20%
A. 30.69 kw C. 34.69 kw will prevent slippage, determine:
B. 32.69 kw D. 36.69 kw 1. the coefficient of friction.
A. 0.313 C 0.513
SOLUTION: B. 0.413 D. 0.613

251mm 2. the tension at the tight side in the original installation


A. 2.37 KN C. 2.97 KN
B. 2.67 KN D. 3.67 KN

3. the tension at the slack side in the original installation.


255mm
A. 2.22 KN C. 4.44 KN

B.3.33KN D.1.11KN

4. the tension at the slack side when idler is installed.


251mm A. 0.452 KN C. 0.802 KN
B. 0.653 KN D. 1.290 KN

5. the new angle of contact.


1. Dm = Do - d = 80 - 10 = 70 mm

0
A. 210° C. 230
y = 185 - 130 = 55 mm
B. 220
0
D. 240
0

C = Dm = 70 = 7

d 10
SOLUTION:
3
8FC n 1. Under the original installation:
y=----

Gd

3 .

5. = et8 = 2.4

F2

0.055= 8Fa(7) (9.5)

(80,000,000) (0.01) e t8 = e l (160 )(1[/ 180 ) = 2.4


Fa = 1.688 KN f = 0.313
Fl
2. For the clutch: 2. F I - F 2 = F = 12(130) = 1560 N = 1.56 KN

D, = Do - 2d = 255 - 2(25) 205mm T = (F , - F2 ) r = 156 (0.200/2) = 0 156 KN-m

Io = 255/2 = 127.5 mm
r, = 205/2 = 102.5 mm
288 Sill/ut ional Problems Sit uational Problems 289
F F1 5. the axle diameter for an allowable torsional stress at 12,000 psi, the efficiency of
~ = 2.4 or
F2 F2 = 2.4 the bevel gear differential is 80%.
A. 4 V2 in C. 2 V2 in
F1 B. 1 V2 in D. 3 V2 in
F1 - - = 1.56
2.4

F1 = 2.674 KN
SOLUTION: Wheel

F1 _2.674=1.114KN
3. F2 = 2.4 - 2.4

4. Under new Installation, the idler should be placed on slack side so that tight side

tension remains the same, that is, F 1 = 2.674 KN

T' = 1.20T = 1.20(0.156) = 0.1872KN-m

(F 1 - F2) r = T'

(2.674 - F 2)(0.100) = 0.1872

F2 = 0.802 KN
Wheel

5. F1 f 0 F2 T = torque at wheels
-=e 1.
F2 T = F x r = 17,500 (48/2) = 420,000 in-Ibs
P = power at wheels = 500 (0.80) = 400 hp
.2.674
- - - =e 031350
0.802 P=2TTTN

8 = 3.841 rad 180 = 220


0
0 400 (33,000) = 2 TT I 42~,000 IN
12
TI
N = 60 rpm speed of wheels
SITUATIONAL PROBLEM 11 (ME Board October 1982)

2. T s N s = TN
A four-wheel vehicle must developed a draw-bar pull of 17,500 Ibs. The engine,

34 Ns = 51 (60)
which develops 500 hp and drives through a gear transmission a 34-tooth spiral bevel

Ns = 90 rpm speed of shaft


pinion gear which meshes with a spiral bevel gear having 51 teeth. This gear is

keyed to the drive shaft of the 48 in diameter rear wheels of the vehicle. Determine:

1500 =16.67
1. the speed of the wheels. - 3. Transmission gear ratio
90
A. 55 rpm C. 65 rpm Transmission gear ratio 16.67:1
B. 60 rpm D. 70 rpm
I 4. Tw = torque required per wheel
2. the speed of shaft
Tw = 420,000 = 210,000 in -Ibs
A. 80 rpm C. 90 rpm 2
B. 85 rpm D. 95 rpm

3. the transmission gear ratio should be used if the engine develops maximum torque 5. S =~6T
3
at 1500 rpm. TId
A. 12.67:1 C. 14.67:1
B. 13.67:1 D. 16.67:1 12,000 = 16(2~000)
3"
nd
4. the torque required per wheel, in-Ibs. d = 4.47
A. 210,000 C. 230,000 Therefore: use 4 1/2 in
B. 220,000 'J. 240,000
290 Sitwtli(Jllul Problems Sit u a i i(mal Problems 291
SITUATIONAL PROBLEM 12 (ME Board October 1982)
_ Dm _ 250 - 3~ = 6.81
The smaller of two concentric helical springs is made of 22 mm diameter steel spring C 2 ~ d- 32
and has an outside diameter of 100 mm with 10 active coils. The outer spring is
3
made of 32 mm steel spring and has an outer diameter of 250 mm with 8 active coils. 8FC n
Before the load is applied, the outer spring is 25 mm longer than the inner spring. If Y= Gd
the load of 90 KN is applied to the nest of the springs. (G = 80 Gpa)
F2 -~_= 80,000,000(0.03~=126.50 KN/m
1. Find the spring rate of the inner spring.
2
K = Y; - 8 C3 n 8 (6.81)3(8)
A. 493.80 KN/m C. 693.80 KN/m
B. 593.80 KN/m D. 793.80 KN/m 3. F, + F2 = 90 or F2 = 90-F 1 Eqn.1

Y2 - Yi = 0.025 Eqn.2

2. Find the spring rate of outer spring. F F1


A. 126.50 KN/m C. 146.50 KN/m --.l=493.8 or Eqn. 3
Y1 = 493.8
Y1
B. 136.50 KN/m D. 156.50 KN/m

F F2

~ = 126.50 or Eqn. 4
3. Find the load carried by the Inner spring Y2 = 126.50
Y2
A. 49.13 KN C. 69.13 KN Substitute equation 3 and 4 to 2:
B. 59.13 KN D. 79.13 KN

~_+_F_1_ = 0.025 Eqn. 5

4. Find the load carried by the outer spring. 126.5 4938


A. 10.87 KN C. 30.87 KN Substitute 1 in 5:
B. 20.87 KN D. 40.87 KN 90 - F1 + _F_
1 _ = 0025
126.5 493.8
5. Find the percent load carried by the inner spring. F1 = 69.13KN
A. 70.81 % C. 74.81 %
B. 72.81 % D. 76.81 % 4. F2 = 90- F1 = 90 - 69.13 = 20.87KN

::Jl
SOLUTION: . d
5. P ercent carne = -F1 =-
69.13
- = 76.81 %
25mm 90KN F 90
c: ==:;: . Y2
yl
~ SITUATIONAL PROBLEM 13 (ME Board April 1983)
A flange coupling connects two 2" diameter shafts. The flanges are fitted with 6 bolt
of SAE 1040 steel on a 7" bolt circle. The shafts runs at 300 rpm and transmits 4
hp. Assume a factor of safety of 5, ultimate tension of 70,000 psi, and ultimate shes
of 55,000 psi. Determine:

1. the torque transmitted.


A. 8,453.80 in-los C. 10,453.80 in-lbs
1. For inner spring: B. 9,453.80 in-lbs D. 11,453.80 in-los

C = Dm = Do-d = 100-22 =3.545 2. the force applied per bolt.


1 d d 22
A. 451.171bs C. 450.17 Ibs

3
8FC n B. 430.171bs D. 470.17 Ibs

y=---

Gd

3. the diameter of the bolt required.


K1 = £"1 = ~3
= 80,000,000 (0.:-02?l = 493.80 KN / m A. 0.228 in C. 0.428 in

Y1 8 C n 8 (3.545)3 (10) B. 0.328 in D. 0.528 in

2. For outer spring: 4. the thickness of flange to be used.


292 SitUQ t iUTI (II Problems
A. 0141 in C. 0.341 in Sit uat iOTiul Problems 293
B. 0.241 in D. 0.541 in A. 773.80 N C. 777.80 N
B. 775.80 N D. 779.80 N
SOLUTION:
4. the width of a steel band for a tensile stress of 50 Mpa.
1. Power = 2 IT T N A. 40.40 mm C. 46.40 mm
45 (33,000) = 2 IT T (300) B. 44.40 mm D. 48.40 mm
T = 787.82 ft-Ibs = 9.453.80 in-Ibs

SOLUTION:

2. T = F (r) = F (D, 12)


9,453.80 = F (7/2)
F = 2701 Ibs total force
F
Fb = force applied per bolt

Fb = Fin = 2701/6 = 450.171bs

t~ I~
""::::""1~
3. For shearing of key:

S
s, f1,
=--=_._=-­
f1,
FS A n d2

1.5 m
70,000 450.17 Dc = 7"
--------
5 2'. d2
4 1. Torque = F x r

d = 0.228 in Torque = 820 (60/2) = 24,600 kg-cm = 2.413.29 N-m

4 For compresslvs of key:


0.25(2450 X 11:_)

S = f1,
edt
2. £L = e f 8 = e 180° = 2.912
F2
70,000 450.17 F 1 = 2.912 F2

5 0.228 t Torque = (F, - F2) r

2413.26 = (F, - F2 ) (0.76/2)


t = 0.141 in F 1 - F2 = 6350.7 N
2.912F 2 - F2 = 6350.70
F2 = 3321.50 N
SITUATIONAL PROBLEM 14 (ME Board November 1983)
A simple band brake has a 76 cm drum and fitted with a steel band 2/5 cm thick lined 3. F 1 = 2.912 F, = 2.912 (33215) = 9672.20 N
with a brake lining having a coefficient of friction of 0.25. The arc of contact is 245°. Summation of moments about pivot point = 0
The drum is attached to a 60 cm hoisting drum, that sustains a rope load of 820 kg. F(150) = 9672.2 (12)
The operating force has a moment arm of 1.50 m and the band is attached 12 cm F = 773.8CJ N
from the pivot point. Find:

1. the torque just required to support the load (clockwise). 4. S=~=!1


A bt
A. 2,013.26 N-m C. 2,413.26 N-m
B. 2,213.26 N-m D. 2,613.26 N-m t = 2/5 0.4 cm =0.004 m

9672.20

2. the tension at the slack side of the brake.


50,000,000 = ----

b (0.004)
A. 3321.50 N C. 3521.50 N
B. 3421.50 N D. 3621.50 N
b = 0.0484 m = 48.40 mm

3. the operating force at the end of brake arm.


294 SitIW( iOTlo[ Problems Siluu.lioTWl Problems 295
SITUATIONAL PROBLEM 15 (ME Board November 1983)
A mechanical press is used to punch 6 holes per minute on a 25 mm thick plate. The 10.31 == ---'!"- [(10472)2 - (9425)2 ]
2(9.81)
hole is 25 mm in diameter and the plate has an ultimate strength in shear of 420 Mpa. b == 3t
The normal operating speed is 200 rpm and it slows down to 180 rpm during the W == 9.71 KN
process of punching. The flywheel has a mean diameter of one meter and the rim
width is 3 times the thickness. Assume that the hub and arm account for 5% of the 4. W == Wr + Wah
rim weight concentrated at the mean diameter and density of cast iron is 7,200 kg per 9.71 = W r + 0.05W r

cubic meter. Find: W r = 9.248 KN = 942.70 kg

Wr = volume x density

1. the maximum force required to punch a hole. = (IT D b t) w D=1

A. 804.67 KN C. 844.67 KN b = 3t

B. 824.67 KN D. 864.67 KN 942.70 = IT (1) (3t) (t) (7200)

t == 0.118 m == 118 mm

2. the power required to drive the press.


A. 0.833 kw C. 1.56 kw 5. b = 3 t == 3 (0.118) = 0.354 m Rim
B. 1.031 kw D.2.13kw b = 354 mm

3. the flywheel weight. SITUATIONAL PROBLEM 16 (ME Board November 1983)

A. 3.23 KN C. 7.38 KN A flange coupling is to connect two 57 mm shafts. The hubs of the coupling are eacl

B. 5.39 KN D. 9.71 KN 111 mm in diameter and 92 mm thick and the flange web are 19 mm thick. Six 1(

mm bolts in a 165 mm diameter circle connect the flanges. The keyway is 6 mn

4. the thickness of the rim. shorter than the hub's thickness and key is 14 mm x 14 mm Coupling is to transmi

A 118 mm C. 162 mm 45 kw at 160 rpm. For all parts, yield point value in shear is one-half the yield valu:

B. 130 mm D. 184 mm which is 448 Mpa. Find the following:

5. the width of the rim. 1. the force on shaft.


A. 324 mm C. 344 mm A. 9425 KN C. 86.34 KN
B. 334 mm D. 354 mm B. 90.23 KN D. 80.34 KN

SOLUTION: 2. the factor of safety in shear of key.


A. 1.83 C. 348
1. Maximum Force per hole Stress x Shear area == S (IT X d x t p ) B. 2.86 D. 5.34
Maximum Force per hole 420,000 (IT x 0.025 x 0.025) == 824.67 KN
3. the factor of safety in bearing of key.
A. 5.34 C. 3.20
· · 1 (60)
2. T rrne required per hole == - - == 10 sec B. 4.23 'J. 2.86
6
Energy Fave t p (Fmax /2) t 4. the force applied per bolt on coupling.
Power == - - _ . == - ' - == ---~
p
time time time
A. 543 KN C. 7.39 KN
B. 6.94 KN D. 8.39 KN
Power == (824.67/2)(0025) == 1.031 kw

10
5. the factor of safety in shear of bolts.
A. 6.39 C. 8.30
3. v, == IT D N1 == IT (1) (200/60) == 1047 rn/s B. 7.39 D. 9.30
V2 == IT D N2 == IT (1)(180/60) == 9425 rn/s
824.67 ) SOLUTION:
Energy = Fave t p == ( --2-. 0.025) C~ 10.31 KN - m
1. Power = 2 IT T N

W 2 2 45 == 2 IT T (160/60)

KE == -(V1 - V2 )

2g
T = 2.69 KN-m

Sit Ilalional Problems 297


296 Sit UCI t ionul Problems
F 5.43
r = 57/2 = 28.5 mm Ss = ---- = = 26,987 Kpa = 26.99 Mpa
r = 0.0285 m 19 mm---71 ~ d2 ~ (0.016)2
~r
T = F. r 4 4
2.69 = F (0.0285)
FS= 224 =8.3
F = 94.25 KN 26.99

2. Length of key 92 - 6 SITUATIONAL PROBLEM 17 (ME Board April 1984)


Length of key 86mm De = 165 mm A hydraulic control for a straight motion utilizes a spherical pressure tank "A" that is
Length of key 0.086 m connected to a work cylinder "B" as shown in the sketch. "A" maintains pressure in
the tank at 400 psi.
1. The tank is 32 inches in diameter, welded with 100% joint efficiency and made of
steel plates with an allowable tensile strength of 7,500 psi. Calculate the required
thickness of the tank.
For shearing of key:
A. 0.23 In C. 0.54 in
S _ ~~ £.. 94.25 _ ---7 [ I~ 92 mm B. 0.43 in D. 0.65 in
s - A - wL - 0.014(0.086)
= 78,277 Kpa = 78.28 Mpa 2. The pressure drop between tank and the cylinder is 30 psi. Assume allowance for
friction in the cylinder and packing to be 10 percent of F, the operating force.
448
Ssu = - = 224 Mpa Calculate the diameter of the piston if value of F is 5000 Ibs.
2 A. 1.27 in C. 3.29 in
FS = Ssu = 224 = 2.86 14 m.m. B. 2.39 in D. 4.35 in
s,
78.28
14 m.m. 3. Determine the wall thickness of work cylinder B if it is made of cast iron having an
3. allowable tensile stress of 2,000 psi.
For bearing or compressive stress in key:
A. 0.448 in C. 0.653 in
B. 0.560 in D. 0.837 in
F 94.25
Sc = 'h- co 0.014 = 156,555 kpa = 156.55 Mpa
- L - ..- . (0.086) 4. The piston stroke of the work cylinder is 18 inches; the time of a work stroke is 5
2 2 seconds. Calculate the horsepower output of the cylinder.
Siu = 448 Mpa A. 1.29 hp C. 2.73 hp
B. 4.23 hp D. 5.34 hp
FS = ~tu _ 448 5. Assume that the work cycle of the piston rod occurs every 30 seconds, that the
Sc - 156.55 = 2.86 overall efficiency of the hydraulic control is 80 percent, and the pump efficiency is
60 percent, determine the horsepower of the motor continuously operating the
pump.
4. For the coupling:
A. 0.25 hp C. 0.95 hp
T = F (Dc / 2) D. 1.27 hp
B. 5.34 hp
2.69 = F( 0.
\
165
2
1
) SOLUTION:
F = 32.557 KN

Force per bolt =-F 32.557


= '-- = 5.43 KN
n 6

5. For shear in bolts:


448
Ssu = - - = 224 Mpa
2
298 Si ; IIUt ;ollal Problems Sir uat ;OT!GI Problems 299
1. S = PO, ~ITLJATIONAL PROBLEM 18 (ME Board October 1984)
4t Consider a 304.8 mm section of a single-riveted lap joint made up with plates of 6.35
mm thickness and 6 rivets, 15.87 mm in diameter. Assume that rivet holes and rivets,
7500 = 400 (32)
15.87 mm in diameter. Assume that rivet holes are 1.58 mm larger in diameter than
4t
the rivets. In this joint, the entire load is transmitted from one plate to the other by
t = 0.427 in means of the rivets. Each plate and the six rivets carries the entire load. Design
2 2.
stress for shear is 598 kq/crn'', for bearing, 1406 kg/cm and for tension, 703 kg/cm
2 For cylinder B: Assume that the rivets will not tear or shear through the plate to the edge of the joints.
W = Force = Pressure x Area Find:
1. the force that can be carried by unpunched plate.
5000 + 0.10(5000) = (400 - 30) (~0 2 J A. 13,606.40 kg C. 17,606.40 kg
B. 15,606.40 kg O. 19,606.40 kg
o = 4.35 in 2. the force to cause tensile stress on plate between rivets.
A. 6,394.56 kg C. 8,932.6 kg
3. For cylinder B: B. 7,083.09 kg O. 9,099.45 kg
S=fJEL 3. the force to cause shearing stress on rivets.
2t A. 7,580.90 kg C. 9,580.90 kg
B. 8,580.90 kg O. 10,580.90 kg
P = 400 - 30 = 370 psi
4. the force to cause bearing stress on rivets.
2000 = ~70(435)
A. 9,347.70 kg C. 11,347.70 kg
2t B. 10,347.70 kg O. 12,347.70 kg
t = 0.402 in 5. the efficiency
t 0.402 A. 563.4% C. 60.45%
- = - - - = 0.09 (greater than 0.07) B. 63.06% O. 76.34%
o 4.35
Therefore, use Lame's Equation for thick cylinder. I
SOLUTION: ,)
0
t=~( !S+P -1]=~·35l" 12000+370 -1']=0.448 in 0
21 YS - P
\
2 V2000 - 370
~ F .. .... , , 0
0
1304.s m ... ( • F
4. v = Sit = 18/5 = 3.6 in/sec = 0.3 ft/sec 0
HP= Fv =?OOO (~=2.73 h
0
550 550 P
5. .. I ~.'
. •.'~' 65.35
..... ~ mm
Work output F ....' .t: . I •
Efficiency = Work input t F
L
2.73 (5) 1. Fu = the force that can be carried by unpunched plate = S A
0.80(0.60) = (Motor Hp)(30) Fu = 703 (30.48)(0.635) = 13,606.40 kg
Motor Hp = 0.95
2. Ft = the force to cause tensile stress on plate between rivets
F t = (703) [30.48 - 6(1.745)] (0.635) = 8,932.6 kg

3. F s = the force to cause shearing stress on rivets.


F s = 6 (IT/4)(1745)2 (598) = 8,580.90 kg

4. Fb = the force to cause bearing stress on rivets.


Fb = 6 (1745)(0.635)(1406) = 9,347.70 kg

5. F = safe load = smaller force = 8,580.90 kg


300 SU'lo ( i Oil (II Problems
Situa( iorwl Problems 301
8,580.90 ~ 6306%
Efficiency = 13,606.40
1. Torque to be delivered by the clutch in N-m.
A. 224 N-m C. 245 N-m
SITUATIONAL PROBLEM 19 (ME Board April 1985) B. 239 N-m O. 265 N-m
A 76 mm solid shaft is to be replaced with a hollow shaft of equal torsional strength. 2. Axial force on the clutch in KN. Outside and inside diameter of the clutch are 300
The outside diameter of the hollow shaft is 100 mm. mm and 240 mm respectively. There are two pairs of mating surface with a
Find: coefficient of friction of 0.30. (Use uniform wear method)
1. the inside diameter of hollow shaft A. 2.34 KN C. 3.25 KN
A. 86.55 mm C. 90.28 mm B. 2.95 KN O. 4.39 KN
B. 88.34 mm O. 92.34 mm 3. Torque on wheels
2. the percentage of weight saved A. 1.234 KN c. 3.56 KN-m
A. 50.34% C. 56.56% O. 4.476 KN-m
B. 256 KN
B. 53.34% O. 65.34% 4. Draw bar pult developed in KN.
A. 12.57 KN C. 17.30 KN
SOLUTION: B. 15.23 KN O. 19.39 KN
5. Speed of travel of the vehicle in km/hr.
1. For solid shaft: S = _16 T A. 5.34 km/hr C. 10.74 km/hr
3 B. 7.49 krn/hr O. 12.03 km/hr
TI: d

For hollow shaft: S= 16 TOo SOLUTION: Wheel


11:(0 0 4-0,4)
Equating the stress:
16T _ 16TO o
11: d3 - 11:(00 4 -0;4)
16 T _ 16 T (1 00)
11: (76)3 - ~ [(100)4 - 0;4]
(100)4 - 0,4 = (76)3 (100)
0, = 86.55 mm
~ lO.74kph
m
2. For solid shaft: w V or m = Vw Power = 2 rr T N
1.
w = density of shaft material 50 = 2 rr T (2000/60)
L = length of shaft T = 0.239 KN-m = 239 N-m

rn, = l ~ (76)2 J L w = 4536.46 w L 2. ro = 300/2 = 150 mm = 0.15 m


r, = 240/2 = 120mm = 0.12m
For hollow shaft:
T= nfFa(ro H;2
mH = l~(0/-012))L W=l~(100)2_(86.55)2lL w=1970.64 wL 2
239 = 2(0.30)(Fa)(0.15+0.12)
. m -mH 4536.46wL-1970.64wL 2
Percentage of weight saved = s = -- = 56.56%
ms 4536.46 w L Fa = 2,951 N = 2.95 KN

SITUATIONAL PROBLEM 20 (ME Board April 1985) 3. Power on wheels = 50 (0.75) = 37.50 kw
The engine of a motor vehicle with a wheel diameter of 712 mm develops 50 kw at Speed of wheels = 2000/25 = 80 rpm
2,000 rpm. The combined efficiency of the differential and transmission is 75% with P = 2nTwN
an overall speed reduction of 25 is to 1. Determine: 375 = 2nTw(80/60)
T w = 4.476 KN-m torque on wheels
J02 SUllO! ionul Problems Situu t lonul Problems 303
4. Tw,=Fxr
Lead 4
4.476 '= F x (0.712/2) tan x = --- = - - = 0.0707
F = 12.57 KN draw bar pull nOm n(18)

5. Speed = TT 0 N = TT (0 712)(80) = 178.945 m/min


Speed = 178.945 (60/1000) = 10.74 km/hr
T
5
= 5...4.~0) (18) lr(COS 145°)(0.07~7)
2 cos 14S - (0.3)(0.0707) J
l
+ 0.30 =- 1401 kg - mm

3. TT = total torque = Te + T 5
SITUATIONAL PROBLEM 21 (ME Board April 1985) TT = 1920 + 1401 = 3321 kg-mm
A single threaded trapezoidal metric thread has a pitch of 4 mm and a mean diameter
of 18 mm. It is used as a translation screw in conjunction with a collar having an
outside diameter of 37 mm and an inside diameter of 27 mm. The load is 400 kg and SITUATIONAL PROBLEM 22 (ME Board April 1985)
coefficient of friction is 0.3 for both threads and collar. Find: It is found that a shearing machine requires 205 joules of energy to shear a specific
1. the collar torque gauge of sheet metal. The mean diameter of the flywheel is to be 76.20 cm. The
A. 1920 kg-mm C. 1890 kg-mm normal operating speed is 200 rpm, and slows down to 180 rpm during shearin9
B. 1763 kg-mm O. 2003 kg-mm process. The rim width IS 30.48 ern, and the weight of cast iron is 7,196.60 kg/m .
The arms and hub account 10% of the rim weight concentrated at mean diameter.
2. the torque required for thread. Determine:
A. 1293 kg-mm C.1598kg-mm 1. the rirn weight
B. 1192 kg-mm O. 1401 kg-mm A. 33.88 kg C. 37.23 kg
B. 35.23 kg O. 39.45 kg
3. the total torque 2. the thickness of the rim
A. 1083 kg-mm C. 3321 kg-mm A. 0.145 cm C. 0764 cm
B. 2089 kg-mm O. 4873 kg-mm B. 0.345 cm O. 0587 cm

SOLUTION: SOLUTION:

1. v, = TT 0 N, = TT (0762) (200/60) = 7.98 m/s


V2 = TT 0 N2 = TT (0762)(180/60) = 7.182 m/s
KE = 205 J = 205 N-m
W
KE = -_. (V, - V2
2 2
) 30.48 em
2g »»>.

205 = ---'!!...._- [(798)2 - (7.182)2 ]


2(9.81)
W = 332.4 N = 33.88 kg
D =76.20 e
2. W = W r + Wah
33.88 = W r + 0.10W r
W r = 30.804 kg
37 27
0.3(400)(-- + -) W r = volume x density = (TT 0 b t) w
,)_
1. Tc = -fW(ro+r
-2 --- - 22 2 =1920kg-mm 30.804 = TT (0 762)(03048)(t)(7196.6)
t = 0.00587 m = 0587 cm Rim

2. For ACME of trapezoidal thread <tJ = 14.5°


I
T5 = W Om cos <jltan x + f] SITUATIONAL PROBLEM 23 (ME Board April 1985)
2 l
cos <jl - f tan x A double reduction gear assembly is mounted on three parallel shafts located on the
same horizontal plane. Shaft A is driven from a 2 kw source at 3,500 rpm. The
pinion on shaft A has 24 teeth and meshes with a gear on shaft B. Another pinion on
shaft B is in mesh with a gear on shaft C with 160 teeth. The centerline distance
Sit llU/ iOTwl Problems 305
:304 Sit u a t i{JTwl Problems
between shaft A and shaft C is 154 times the module. The overall gear train ratio is C = !JA +OSl+ °B2+ 0C
2 2-
14: 1. Assume module for all gears are the same, determine:
154 M = 24 M + TB1 M + ~B2M + 160 M
1. the speed of shaft B. 2 2---
A. 1000 rpm C. 1200 rpm TB2=124-TB1 ____. __ ._ _.. Eqn. 3
B. 1100 rpm O. 1300 rpm
2. the number of teeth on gears at shaft B.
Substitute Equations simultaneously:
A. 84 & 40 teeth C. 84 & 60 teeth (124 - T B1) NB = 160 (250)
B. 84 & 50 teeth O. 84 & 70 teeth
3. the torque on shaft A. 124 NB - B~~OO Ns == 40,000
A. 3.28 N-m C. 5.46 N-m
B. 4.34 N-m O. 6.94 N-m NB = 1000 rpm speed of shaft B
4. the torque on shaft B
A. 10.23 N-m C. 16.39 N-m
B. 13.45 N-m 0.18.14N-m I 2. TB1 2
24( 3500 == 84 teeth
-1000
5. the torque on shaft C.
A. 56.34 N-m C. 73.23 N-m T B2 160(250) == 40 teeth
B. 68.95 N-m O. 7830 N-m 1000

3. Power = 2 TT T N
SOLUTION: A B c PA = 2 TT TA NA
"-- ..,I ~NA""'.I ~ c::::::: .'1 =Nc 2 = 2 TT TA (3500/60)
T A == 0.00546 KN-m = 5.46 N-m

4. PB = 2 TT TB NB
Ps
e = --
PA

0.95 == PB
2
P B = 1.9 kw
1.9 = 2TTTB(1000/60)
T B == 0.01814 KN-m == 18.14 N-m

5. e = -Pc
PB
Tb2

c = 154 M 0.95 == £'52-


19
1. Nc = speed of shaft C Pc = 1.805
T A NA = T B1 NB Pc =
2 TT TN
24 (3500) = TB1 NB = 84,000 Eqn 1 1.805 = 2 TT r. (250/60)
T, = 0.06895 KN-m == 68.95 N-m
or T = 84,000
S1
Ns
TB2 N B = r, Nc = 160 (250) = 40,000 Eqn.2 SITUATIONAL PROBLEM 24 (ME Board October 1985)
A key is to be desired for a 12 7 cm, shaft which will transmit power .ot 150 kw at 360
o 2
rpm. If the allowable shear stress for the key is 920 kgICm and the allowable
M = - or 0 = MT 2.
T compressive stress is 1200 kg/cm determine the following
C = center distance between shaft A and C

\
306 Sitllr~( iorlU{ Problems Sit u at iorw! Problems 307
1. Cross-sectional dimensions of the flat key to be used. L ~ 4.79 em
A. 1.11 em C. 3.33 em
B. 2.22 em D. 4.44 em 5. The safe length of key to be used should be the longer length.
2. Force acting on a key Therefore choose L = 4.79 em
A. 62.66 KN C. 66.04 KN
B. 64.34 KN D. 68.34 KN 6. Fa = axial force to remove hub
3. Length of key under shearing stress. Fa = 2 f F
A. 7.34 em C 4.23 em Fa = 2 (0.45)(62.66) = 56.40 KN
B.6.31cm D.2.19cm
4. Length of key under compressive stress. SITUATIONAL PROBLEM 25 (ME Board October 1985)
A. 2.03 em C. 4.79 em A pinion rotating at 1800 rpm and supported on each side by a ball bearing transmits
B. 3.49 cm D. 5.34 em 18 kw to a mating spur gear. If the pressure angle is 20 degrees and the pitch
5. Safe length of key to be used diameter is 102 mm, determine the following:
A. 2.03 em C. 6.34 em Note: Add 3 degrees to the pressure angle to compensate for the friction.
B. 4.79 em D. 8.03 cm
6. Axial force to remove the hub from the shaft if the coefficient of friction is 0.45. 1. Pitch line velocity
A. 45.23 KN C. 58.34 KN A. 8.10 rn/sec C. 11.02 m/sec
B. 56.40 KN 0 65.45 KN B. 9.61 m/sec D. 13.03 m/sec
2. Torque transmitted by the pinion.
SOLUTION: A. 65.34 N-m C. 87.23 N-m
B. 7523 N-m D. 95.49 N-m
1. From Doughtie and Vallance, Table 5-1, p. 100 3. Tangential load on gears.
For shaft diameter of 12.7 em or 5 in, A. 1,563 N C. 1,872 N
w = 1 14 in = 3.175 em B. 1,754 N D. 1,908 N
h = 7/8 in = 2.22 em 4. Separation load on gears.
A. 794.6 N C. 982.3 N
2. P = 2 IT TN
B. 872.3 N D. 1,735.8 N
150 = 2 IT T (360/60) 5. Total load on gears.
T = 3.99 KN-m = 397.89 KN-cm A. 2,034 N C. 4,034 N
397.89 = F (12.7/2) B. 3,034 N D. 5,034 N
F = 62.66 KN
Key SOLUTION:
3. For shearing of key:
S=~ • F
1. V TTDN
wL V TT (0.102) (1800/60) 9.61 rn/sec

920 (0.00981) = 62.6~ 2. P = 2TTTN


3.175 L 18 = 2 TT (T) (1800/60)
L = 2.19 cm T = 0.09549 :<:N-m = 95.49 N-m

4. For compressive of key: 3. Ft =


tangential load
T = • r Ft
95.49 = Ft (0.102/2)
F F t = 1.872 KN = 1,872 N
Sc='-
h
- L 4. Fs = load of separation between gears
2
1200(0.00981) = E·66 tan e = ~
2.22 Ft
--- L
2 e = 20 + 3 = 23°
308 :-iiI lIul iOllul Problems
Situational I'rohl ern s: 309
o F
tan 23 = ---s F
1872 W (h + y) = -Y
r, = 794.6 N 2
h = 3 m = 300 cm
5. Fn = total load on gears 100 (300 + 30.48) ~ (30.38)
F 2
cos e = ~
Fe F = 2168.50 kg
o 1872
cos 23 = ---
Fn 3. C = Dm_ = 8 or Om = 8 d
d
F n = 2,034 N

S=8KF~r:rl
SITUATIONAL PROBLEM 26 (ME Board October 1985) TC d3
A dumb waiter designed to travel a height of 3 m when loaded will have a maximum 3868 = 8(1.184)(2168.50)(8d)
gross weight of 100 kg. A coil spring is provided below to absorb shock in the event TC d3
that the dumb waiter should fall freely because of sudden breakage of the wire rope
carrying it. If the coil spring will be deflected to 30.48 em, if the dumb waiter should d = 3.68 cm
accidentally fall and it hit the spring from its maximum height of travel, determine the
following: 4. Om = 8 d = 8 (3.68) 29.42 cm

1. Whaal factor 8FC 3 n


5. y=----
A. 1.045 C. 1.564 Gd
B. 1.863 D. 1.184
3
2. Maximum force acting on spring. 30.48= 8(2168.50)(8) n
A. 1872.34 kg C. 2235.40 kg 808,720 (3.68)
B. 2168.50 kg D. 2634.66 kg
n = 10.2 active coils
3. Wire diameter
A. 1.20 cm C. 3.68 cm
B. 2.34 cm D. 4.73 cm
SITUATIONAL PROBLEM 27 (ME Board April 1986)
4. Mean coil diameter.
The maintenance shop of PAL has a motor operated hoisting winch which consist of
A. 29.42 cm C. 37.45 cm
the following mechanical parts: Motor shaft "A" is fitted with a double threaded left
B. 34.23 cm D.41.23cm
handed worm in mesh with a 56 tooth worm wheel at shaft "B" directly below.
5. Number of active coils.
Compounded on shaft "8" is a 20 tooth 5 module pinion in mesh with a spur gear
A. 10.20 C. 16.34
keyed on shaft "C" with a horizontal center line distance of 150 mm. Mounted on the
B. 14.23 D. 18.34
same shaft "C" is a 200 mm diameter hoisting drum. The cable wrap around the
Note: Use spring index of 8 and assume maximum induced stress and shear
drum sustains a load of 500 kg at motor speed of 1750 rpm. Determine the following:
modulus of elasticity to be 3868 kq/cm" and 808,720 kg/cm 2 , respectively.

--1
SOLUTION: 1. Pitch diameter of pinion
A. 100 mm C.104mm
3m B.102mm D. 108 mm
1. K = 4C =-1 + 0.615 2. Pitch diameter of spur gear.
4C-4 C A. 180 mm C 200 mm
K = 4(8)~+ 0.615 = 1.184 B.190mm D. 210 mm
. 4(8)-4 8 3. Speed of shaft C .
A. 27.34 rpm C. 34.24 rpm
2. F = maximum force on spring B. 31.25 rpm D. 39.45 rpm
For impact load on spring: 4. Power at shaft C.
A. 1.45 kw C. 1.894 kw
310 Situnl iOflul t'ronl ems Sit untiorwl Probl ems 311
B. 1.605 kw D. 2083 kw Power at shaft C 1.605 kw
5. Torque of shaft A.
A. 7.234 N-m C. 11.23 N-m Pc
B. 9.563 N-m D. 13.47 N-m 5. e=-
6. Velocity of the load PA
A. 0.213 m/sec C. 0.327 m/sec 0.65 = 1.605
B. 0.189 m/sec D. 0.485 m/sec PA
PA = 2.47 kw
SOLUTION:
PA = 2 IT TA NA
2.47 = 2 IT TA (1750/60)
Shaft "A': 1750 rpm
TA = 0.01347 KN-m = 13.47 N-m

6. V = IT Dc Nc
V = IT (0.20) (31.25) = 19.63 m/min = 0.327 m/sec

200 mm SITUATIONAL PROBLEM 28 (ME Board April 1986)


A machine shop somewhere in Quezon City fabricated a pair of spur gear 2.5 module
hoisting drum
and to be mounted on shafts with a center line distance of 90 mm. The speed ratio
,....A ! Shaft "en required is 3: 1. Determine the following:
1. Pitch diameter of the gear.
A. 135 mm C. 145 mm
B. 140 mm D. 150 mm
2. Number of teeth of the gear.
A. 50 teeth C. 54 teeth

• B. 52 teeth D. 56 teeth

o 3. Circular pitch
1. Module =~
T A. 6.23 mm C. 8.37 mm
Dp = Module x T = 5 (20) = 100 mm B. 7.85 mm D. 9.02 mm
4. Addendum distance
Dp +D s A. 2.50 mm C. 3.5 mm
2. C = center distance B. 3.0 mm D. 4.0 mm
2 5. Clearance
150 = 1002P~ A.0.125mm C. 0.425 mm
2 B. 0.250 mm D. 0.625 mm
Os = 200 mm 6. Dedendum distance
A. 2.33 mm C. 4.66 mm
3. Number of teeth of spur gear = 200/5 = 40 teeth B.3.125mm D. 5.99 mm
7. Whole depth
S pee d ra ti10 = No. of teeth of worm =-56 = 28 A. 2.38 mm C. 4.36 mm
No. of thread of worm 2
B. 3.02 mm D. 5.625 mm
1750 8. Working depth
NB = speed of shaft B = - -- = 62.5 rpm
C.5mm
28 A. 3 mm
Tc Nc = TBNB B. 4 mm 0.6 mm
(40) Nc = 20 (62.50) 9. Tooth thickness
Nc = 31.25 rpm A. 3.93 mm C. 5.38 mm
B. 4.34 mm D. 6.34 mm
4. Torque at shaft C = F x r = (500 x 9.81) (0.2/2) = 490.50 N-m 10. Space width
Power at shaft C = 2 IT TN = 2 IT (490.50/1000)(3125/60) A. 3 mm C. 5 mm
B. 4 mm 0.6 mm

l __
Situational Problems 313
312 SitllUl iorwl Problems
11. Outside diameter of the gear 6 Dedendum 1.25 _~~ 0.123 in = 3.125 mm
A. 120 mm C 140 mm -DP-10.16
B.130mm D. 150 mm
12. Root diameter of pinion.
7. Whole Depth 2.25 =0.2214in= 5.625 mm
A. 28.75 mm C. 17.45 mm 10.16
B. 32.34 mm D. 38.75 mm
2 2
SOLUTION: 8 Working Depth DP = 10.16 = 0.197 in = 5 mm

1. C=----
Dp+D g Gear j
2 9. Tooth thickness 1.5708 _ 1.5708 == 0.1546 in = 3.93 mm
-Dr - 10.16
o, +D g
90,= .-.-..-
2 10. Space Width = Backlash + Tooth thickness
180 = Dp + Dg 0035 0.035 03 .
Bac klash = - - - . =: - - = 0.0 445 rn = 00875 mm
DP 10.16
o, N p = o, Ng Space width =: 0.0875 + 3.93 =: 4.02 mm
Np Dg
-=-=3
Ng o, 90mm 11. Outside diameter =: Pitch diameter + 2 (addendum)
OD p =: 45 + 2(2.5) =: 50 mm
o, = 3 o, OD g =: 135 + 2(2.5) =: 140 mm
Dp + 3D p = 180
Dp = 45 mm 12. Root diameter =: Pitch diameter - 2 (dedendum)
Dg = 3 (45) '= 135 mm
RD p =: 45 - 2(3.125) =: 38.75 mm
RD g =: 135 - 2(3.125) =: 128.75 mm
D
2. M==-
N
SITUATlONAL PROBLEM 29 (ME Board April 1986)
2.5 = 45
Tp A pulley is keyed to a 2 Y2 inches diameter shaft by a 5/8 x 7/16 in x 3 in flat key. The
shaft rotates at 50 rpm. The allowable shearing stress for the key is 22 ksi. The
Tp =: 18 teeth allowable compressive stress for the key, hub and shaft are 66 ksi, 59 ksi and 72 ksi,
2.5 = 135 respectively. Determine:
Tg
Tg =: 54 teeth 1. the torque that can be carried due to shearing stress of key.
A. 51,562.50 in-Ibs C. 71,562.50 in-Ibs
B. 61,562.50 in-Ibs D. 81,562.50 in-Ibs
TC D TC (45)
3. Pc = - - =--=7.85 mm 2. the torque that can be carried due to compressive stress of key.
T 18 A. 50,110.60 in-Ib C. 54,11060 in-Ib
From Faires, p. 362 B.52,110.60in-lb D.56,110.60rn-lb
3. the torque that can be carried by the shaft.
4. DP = diiarnetraIpitc
' h = -
25.4
-=-25.4
- - = 10.16
M 2.5 A 57,495.10 in-Ibs C. 77,495.10 in-Ibs
B. 67,495.10 in-Ibs D. 87,49510 in-Ibs
Addendum -1 1 = 00
=.-~-
. = 2.5 mm
. 984 In 4. the torque that can be carried by the hub.
DP 10.16
A 42,398.44 in-Ibs C. 46,398.44 in-Ibs
B. 44,398.44 in-Ibs D. 48,398.44 in-Ibs
5. Clearance 0.25 _ 0.25 = 0.0246 in = 0.625 mm 5. the maximum torque the pulley can safely deliver.
DP' 10 16 A. 42,398.44 in-lbs C. 46,398.44 in-Ibs
B. 44,398.44 in-Ibs D. 48,398.44 in-Ibs
314 Situational Problems Situa tiona[ Problems 315
SOLUTION:
36 mm. The coefficient of friction on threads is 0.15. The friction torque on the thrust
bearing of the motor is taken as 20% of the total torque input. Determine:
1. For shearing of key:
F 1. the lead
Ss=-
wL A. 25 mm C. 35 mm
F B 30 mm D. 40 mm
22000 = - - - 2. the lead angle
, (5/8)(3) Hub
F
T
T
= 41,250 Ibs
= F (0/2) = 41,250 (2.5/2)
= 51,562.50 in-Ibs
I A. 10.23°
B. 12.46°
C. 14.34°
D. 16.23°
3. the torque required to turn the screw.
A. 4.23 kg-m C. 8.45 kg-m
B. 6.31 kg-m D. 10.23 kg-m
2. For compressive of key: 4. the total torque input
F A. 0.017327 KN-m C. 0.057327 KN-m
s, =-h-
B. 0.037327 KN-m D. 0.u7428 KN-m
-- L
2 5. the motor power required to operate the screw.
A. 1.239 kw C. 3.110 kw
66,000 = 7 11~ B. 2.239 kw D. 4.239 kw
- (3)
2 SOLUTION:
F = 43312.50 Ibs
T, = F. r = (43312.50)(2.5/2) = 54,110.60 in-Ib 1. v = NL
10 = 400 L
3. Assuming the shaft and key are of the same material. L = 0.025 m = 25 mm
For the shaft:
s, = 16 T
2.
Lead 25
tan x = - - = - . - - = 0.221
3
It d It Om It (36)

22.000 = ~ 3.
x = 12.46°
Torque required to turn the screw, T,
It (2.5)3

T = 67,495.10 in-los I
T = W Om cos <jJ tan x + f]
s
2 l
cos <jJ - f tan x
4. Torque carried by the hub: For ACME thread: <jJ = 14.5°
F
Sc=-
~L
I
T = 900\0.036) cos 14.5° (0.221) + 0.15] = 6.31 kg _ m
2
s
2 l
cos 14,50 - 0.15(0.221)
F
59,000 7 I 16= 4. T = torque input = Ts + Te
- (3) T = Ts + 0.20T
2
T = 6.31 + 0.20 (6.31)
F = 38,718.75 Ibs
T = 7.572 kg-m x (0.00981) = 0.07428 KN-m
T = F . r = (38,718.75)(2.5/2) = 48,398.44 in-Ibs

5. The maximum value of torque that can be safely deliver is the smaller value.
5. Power = 2 rr TN = 2 rr (0.07428)(400/60)
Power = 3.11 kw
Therefore: Safe torque = 48,398.4 in-Ibs

SlTUATIONAL PROBLEM 30 (ME Board October 1986)


SITUATIONAL PROBLEM 31 (ME Board October 1987)
A double thread ACME thread screw driven by a motor at 400 rpm raises the
A gear having 60 teeth is being driven by a 12-tooth gear running at 800 rpm.
attached load of 900 kg at a speed of 10 m/min. The screw has a pitch diameter of
Determine the following:
316 SitzuLl iOTlll1 Problems
SitlLal iOTlul Problems 317
1. the speed of driven gear.
A.150rpm C.170rpm SITUATIONAL PROBLEM 32 (ME Board October 1987)
B.160rpm D.180rpm A disc clutch having an outside diameter of 32 cm and an inside diameter of 12.7 cm
2. The speed of driven gear if a 24-tooth idler IS placed between the driving and is connected to an engine that turns at 750 rpm. The coefficient of friction is 0.60
driven gear. while the pressure between the friction suriaces IS 2 kq/crn",
A.150rpm C.170rpm
B. 160 rpm D. 180 rpm 1. Force on the clutch pedal necessary to disengage the clutch disc from the engine
3. The speed of driven gear if two 24-tooth idler gears were placed between the using uniform pressure.
driving and driven gears. A. 1,155.14 kg C 1,355.14kg
A. 150 rpm C. 170 rpm B. 1,255.14 kg D 1,455.14 kg
B. 160 rpm D. 180 rpm 2. Mean friction radius
4. The direction of rotation of the driven gear when one idlers are used. A. 11.87 cm C. 13.87 cm
A. the same C. ahead B. 1287 cm D.14.87cm
B. opposite D. none of these 3. Clutch torque
5. The direction of rotation of the driven gear when two idlers are used. A 0.6467 KN-m B. 0.7467 KN-m C. 0.8467 KN-m D. 0.9467 KN-m
A. the same C. reverse 4. Power transmitted by the engine
B. opposite D. none of these A. 71.35 kw C. 73.35 kw
B 72.35 kw D. 74.35 kw
SOLUTION:
SOLUTION:

1. For uniform pressure:


1. T1 N 1 = T2N2 R = 32/2 = 16 cm
12 (800) = 60 (N 2) r = 12.7/2 = 6.35 cm
2 2)
N2 = 160 rpm F = P TT (R - r = 2 (TT) [(16)2 - (6.35)2] 1,355.14 kg

3
2. rt 21 R - r3l
3l R 2
_r 2

2. T, N, = T2N2 = T 3N3 rt 3-1 (16)3 - (6.35):l = 11.87 cm


12 (800) = 24 N 2 = 60 (N3) 3l (16)2 - (6.35)
N3 = 160 rpm
Driven
60 teeth 3. T torque
N:J T n f F rt = 1 (0.6) (1355.142) (11.87)
3. T1 N, = T2N2 = T3N3 = T4 N4 T 965132 kq-crn = 0.9467 KN-m
12 (800) = 24 N2 = 24 N 3 = 60 N4
N4 = 160 rpm 4. Power 2TTTN

Dci"~
Power 2 TT (0.9467)(750/60) 7435 kw

SITUATIONAL PROBLEM 33 (ME Board October 1987)


12 teeth
Driven Two short shafts having identical diameters of 38.1 mm and rotating at 400 rpm are
800 rpm
60 teeth connected by a flange coupling havtnq 4 bolts with a 100 mm bolt circle. The design
N:J shearing stress of the bolt is 12 Mpa and design compressive stress of the flange is
4. Refer to question 2 with one idler used:
15 Mpa.
The direction of driver is the same as driven gear.
1. What is the power transmitted by the short shaft?
5. Refer to question 3 with two idler used:
A.20.50kw C 26.50kw
The direction of driver is opposite as driven gear
B 24.50kw D.28.50kw
2. What is the torque transmitted by the shaft?
318 Siluat iOTiu/ Problems Situal ional Problems 319
A 0.032 KN-m C. 0432 KN-m
SITUATIONAL PROBLEM 34 (ME Board April 1988)
B 0.232 KN-m D. 0.632 KN-m
Two shafts are connected by spur gears. The pitch radii of the gears A and Bare 207
3. What diameter of bolt should be used?
mm and 842 mm respectively, If shaft A makes 350 rpm and is subjected to twisting
A. \4 in C. 3,4 in
moment of 236 N-m. Pressure angle is 14.5° What is:
B. Y2 in D. 1 in
4. How thick should the flange be in mrn?
1. Rpm of gear B.
A 7.07 mm C. 9.07 mm
A. 86 rpm C. 90 rpm
B. 5.07 mm D. 11.07 mm
B. 88 rpm D. 92 rpm
SOLUTION: 2. Torque in shaft B.
A 920 N-m C. 960 N-m
B. 940 N-m D. 980 N-m
1. For short shaft, from PSME Code
3. Separation load of two gears.
D3 N A. 294.82 N C. 298.82 N
P=-~-
38 B. 296.82 N D. 300.82 N
4. Total load on gears.
P = (38.1/254)3 (400) = 35.53 hp = 26.50 kw A 1166 N C.1188N
38 B.1177N D.1199N

2. p = 2rrTN SOLUTION:
2650 = 2 IT T (400/60)
T = 0632 KN-m 1. DA N A = DBNB
(842 x 2) (350) = (207 x 2) NB
3. T = F. r = F (DJ2) Dc = 100 mm NB = 86 rpm
0.632 = F (0.10/2)
F = 12.654 KN 2. T A = F1 . rA
F = 12,654 N 0.236 = F1 (0.207)
F1 = 1.140 KN = 1140 N
Fb = ~ = 12.654 = 3.163 KN
TB = F1 . rB
n 4 T B = 1140 (0.842) = 960 N-m
For shearing of bolt:
F F
Ss =-~- 3. tan 8 = .2.. Fs
Ft
It d 2
4
tan 14.5° = ~~

12000 = 3.163 1140


, It d 2 Fs = 294.82 N
4
d = 0.01832 m = 18.32 mm = 0.72 in 4. cos 8 = .5..
Use standard d = 3,4 in (19.05 mm) Fn
IE,f\ )1< ~ )1
4. For compressive of bolt: cos 14.5° = ~~O
207 mm .:» ' - 842 mm
F Fn
Sc =~
F n = 1177 N
dt
15000=~63
, (0.01905) t
t = 0.01107 m = 11.07 mm SITUATIONAL PROBLEM 35 (ME Board April 1988)
A rectangular key was used in a pulley connected to a line shaft with a power of 746
KW at a speed of 1200 rpm. If the shearing stress of the shaft and key are 30 Mpa
and 240 Mpa, respectively.
320 Si t u at i o n a! Problems Situational Problems 321
1. What is the diameter of the shaft? SOLUTION:
A. 1860 mm C. 26.60 mm
B 21.60 mm D 2960 mm 1. P = 2rrTN
2. What is the length of the rectangular key if the width is one fourth that of the shaft 40 = 2 rr T (500/60)
diameter? T = 0.763 KN-m
A. 324 mm C. 524 mm
B. 4.24 mm D 624 mm T = F . r
0.763 = F (0.610/2)
SOLUTION: F = 2.505 KN = 2.505 N
Fl
1. p = 2rrTN 2. F1 - F2 = F = 2.505 KN = 2,505 N
746 = 2 rr T (1200/60)
T = 00593 KN-m
F 0.35(144 a x-"- )
16 T _J..=e f 8=e 180' =2410
S = ---- 3.
nd 3 F2

30,000 = ~6 (0. 05931 r F2 = - -1


F

T
nd 3 241
F
d = 0.0216 m = 21.60 mm F1 - - 1- - = 2.505
241
2. T = F. r F1 = 4.282 KN
0.059365 = F (0.0216/2)
F = 5497 KN S = F1
w = -~ = .2:
02 16 A
= 0.0054 m
4 4 2100 =_ 428~

S =_£... , b(0.006)
S w L b = 0.3398 m = 340 mm
5.497
240,000 = - - - -.. SITUATIONAL PROBLEM 37 (ME Board April 1989)
(0.0054) L A steel shaft transmits 40 hp at 1400 rpm. Considering allowable shearing stress
L = 0.00424 m = 4.24 mm based on pure torsion to be 5000 psi, find:

1. the shaft diameter of nearest commercial size.


SITUATIONAL PROBLEM 36 (ME Board April 1(88) A.11/4in C.27/16in
A pulley 610 mm in diameter transmits 40 kw at 500 rpm. The arc of contact between B. 1 7/16 in D. 1 3/4 in
the belt and pulley is 144 degrees, the coefficient of friction between belt and pulley is 2. the torsional deflection of the shaft in degrees per foot
0.35 and the safe working stress of the bel' is 2.1 Mpa. It is required to find: A. 0246 deg C. 0266 deg
B. 0.256 deg D. 0276 deg
1. The tangential force at the rim of the pulley in Newtons.
A. 2,405 N C. 2,605 N SOLUTION:
B. 2,505 N D. 2,705 N
2. The effective belt pull in Newton. 1. P = 2rrTN
A. 2,305 N C. 2,505 N 40 (33,000) = 2 rr T (1400)
B. 2,405 N D. 2,605 N T = 150 ft-Ibs = 1800 in-Ibs
3. The width of the belt used if its thickness is 6 mm. S _ ~6T_
A. 340 mm C. 360 mm s - TC d3
B. 250 mm D 370 mm
5000 = ] 6 (1800)
7( D"
;:):2:2 Sit u at iorlUl Problems Sit u a; ional Problems 323
D = 1.224 in D + d == OAO
Therefore use D = 1 7/16 in = 1A375 in (standard) D = OAO - d _ _ _ _ _ _ _ _ _ _ Eqn.2

2. e=~ Equate 1 and 2:


5.305 = 314[d(OA - d) - d
2]
J G 2
628 d - 125.6 d + 5.305 = 0
L = 1 ft = 12 in
4 4 125.6 ± J(=.125.6)2 - 4(628)(5:305)
J = ~~ = 1t (1A375) . = 0.4192 in4 d = -- = 0.139 m = 139 mm
32 32 2(628)
6
G = 12 x 10 for steel D = OA - 0.139 == 0.261 m = 261 mm
. D- d 261 - 139
Face Width = - - - = = 61 mm
8 = .. __1800 (12) 2 2
OA192 (12,000,000) = 0.00429 rad = 0246 deg
SITUATIONAL PROBLEM 39 (ME Board October 1989)
With the present water interruptions prevailing in Metro Manila, you have been asked
SITUATIONAL PROBLEM 38 (ME Board October 1989)
to design an upright cylindrical water tank 6 meter in diameter and 6 m high, vented,
A Pajero jeep wagon's engine develops 40 kw at 1200 rpm, with a single plate clutch
and to be filled completely with water.
with two pairs of friction surfaces transmitting the power. Consider the coefficient of
1. Determine the minimum thickness of the tank plate if the stress is limited to
friction to be 0.30 and mean diameter of disc to be 200 mm, determine
40 Mpa.
A. 2.20 mm C. 4AO mm
1. Axial force required to engage the clutch and transmit the power.
B. 3.30 mm D. 5.50 mm
A. 5.305 KN C. 5.505 KN
2. If the water tank had to be elevated 10 meters from its bottom and has to be filled
B. 5A05 KN D. 5.605 KN
up in 2 hours, determine the pump capacity needed to fill the tank and motor
2. Inside and outside diameter of the discs and the face width if the maximum
pressure is limited to 200 Kpa. power to drive the pump, if velocity and head loss is 5 meters. Consider pump
A. 61 mm efficiency of 70% and motor efficiency of 85%,
C. 65 mm
B. 63 mm A. 5. 16 kw C. 7. 16 kw
D. 67 mm
B. 6.16 kw D. 8.16 kw
SOLUTION:

1~
SOLUTION:
1. P = 2ITTN 1. P == w h
40 = 2 IT T (1200/60) P = 9,81 (6) == 58.86 kpa
T = 0.31831 KN-m S = PD j
2 t
rj = 200/2 = 100 mm = 0.10 m 200mm ID
T = n f Fa rf
0.31831 = 2 (0.30) (F a)(0.1 0)
Fa = 5.305 KN
40000 = 58.86 (6)
, 2t
t = 0.0044 m = 4AO mm
i
1 0 m

2. Fa 2 IT Pmax r, (ro - r,) 2. r


Volume = IT h = TT (6/2)2 (6) == 169.646 m
3/s
3

Q == 169.646/2 == 84.646 m%r = 0.0235 m


5305 2 IT (200) ( -d I -
D---
d ,'\ P = w Q h == 9.81 (0.0235) (10 + 6 + 5) == 4.854 kw
\2 2) 4.854
5.305 2) Input power == - - - - - = 8.16 kw
314 (Dd - d - . Eqn. 1 (0.70)(0.85)

Dj = !?+d SITUATIONAL PROBLEM 40 (ME Board June 1990)


2 A 48 in diameter diamond saw blade is mounted on a pulley driven steel shaft,
0.20 == ~-+:d requiring a blade perimeter linear speed of 150 ft/s. Motor drive is 125 hp at 1200
2 rpm, with 6 inches diameter pulley, Determine:
1. The shaft rpm to attain blade peripheral speed required .

...........".._-,,.
324 Sit uat u nu i] ['rob/ems
A. 71220 rpm Situa! iOrl a I I'robl ems 325
C. 716.20 rpm
B. 71420 rpm D 71820 rpm SITlJATIONAL PROBLEM 42 (ME Board June 1990)
2. The shaft diameter. A flange coupling has an outside diameter of 200 mm and connects two 40 mm
A. 2.10 in C. 230 in shafts. There are four 16 mm bolts on a 140 mm bolt circle. The radial flange
B. 2.20 in D. 2.40 in thickness is 20 mm. If the torsional stress in the shaft is not to exceed 26 Mpa,
3. The shaft pulley diameter. determine'
A. 5.053 in C 15.053 in 1. The power that can be transmitted at 600 rpm.
B. 10.053 in D 20.053 in A. 17.12 kw C. 24.23 kw
B. 20.53 kw D. 32.23 kw
SOLUTION:
1. V=nDN 2. The shearing stress in the bolts if uniformly distributed.
150 (60) = rr (48/12) Ns A. 1.29 Mpa C. 3.04 Mpa
Ns = 716.20 rpm 1200 rpm B. 2.34 Mpa D. 5.804 Mpa

2. For line shaft: 125bp~


3. The maximum shearing stress induced in the bolts.
3 A. 111.65 Mpa C. 115.65 Mpa
D N

Is
p=-- B. 113.65 Mpa D. 117.65 Mpa
53.5

125 = Q~716.20~ 4. The bearing pressure in the bolts.


53.5 11 I' t10 in A. 2.34 Mpa
B. 1.03 Mpa
C. 3.65 Mpa
D. 5.34 Mpa
D = 2.10 in
3. o, Nm = o, Ns , 1~20mm
6 (1200) = D s (716.20) SOLUTION:
......-::::="'~

D, = 10.053 in

SITUATIONAL PROOLEM 41 (ME Board June 1990)


In the LRT II project, steel railroad rails of 10m long are to be installed. If lowest
temperature considered is 16°C, and a maximum temperature of 36°C is designed
for, assuming coefficient of thermal expansion of steel to be 11.6 x 10.6 rn/rn- °c and 200m De = 140 mm
modulus of elasticity of steel to be 207,000 Mpa,
1. Determine the clearance between rails such that the adjoining rail will just touched
at maximum design temperature
A. 1.23 mm C. 2.32 mm 1.
B. 1.56 mm D. 3.23 mm S = 16 T
2. Should the excessive high summer temperature of 40°C occur, determine the n D3
induced stress in the rails at that temperature.
A. 8.34 Mpa C. 10.56 Mpa 26.000 = 16 T
B. 9.605 Mpa D. 12.34 Mpa n (0040)3
T = 0.327 KN-m
SOLUTION: p = 2 rr TN = 2 rr (0.327) (600/60) = 2053 KW
y/2 y/2

.·Tl~p
2. Torque = F. r = F (D, /2)

1. Clearance
Clearance
y/2 +
}
y'~ lei mm 10 mm
1rr 10 mm
I 0.327 = F (0.140/2)
F = 4.67 KN
k L (t2 - t1) I c I F 4.67
6 Fb = force per bolt = - = ---- = 1.167 KN
Clearance 11.6 X 10. (10) (36 - 16)
n 4
Clearance 0.00232 m = 2.32 mm
Fb
Ss = . - - =
1.167
---- - = 5804 kpa = 5.804 Mpa
2 Stress = k E (b - t1)
6 n d2 rt (0.016)2
Stress = 11.6 x 10. (207,000) (40 - 36) = 9605 Mpa 4 4
Sit uat ional Probl ems 327
326 Situational Problems
A. 12.39% C. 18.23%
3. The maximum induced shearing stress in the bolt is caused by the direct B. 14.39% D. 16.29%
shear and initial tension due to tightening. From Vallance p. 134.
SOLUTION:
F, = initial tension due to tightening = 16,000 d
Sft
Where d = diameter of bolt, in
1. S=~=_F_
F, = 16,000 (16/25.4) = 10,078.74Ibs = 44.84 KN A ~D 2
SI = tensile stress due to initial tension 4 r
44.84
SI = = 233,000 Kpa = 233 Mpa 6000 = ~2000l

n (0.016)2 2' 0 2
4 4 r
1 ~2--2 1 I 2 2 Dr = 2.53 in
Ssmax =-~St +4S s =-.y(223) +4(5.804) =111.65 Mpa
2 2 Therefore use Dr = 2.75 in (standard)

4. Bearing pressure or compressive stress in bolts:


S =~= 1167 =3.65 M a
edt (0.016) (0.020) P
___ ._
1
= - = .50,In
2. Pitch 0
no. of thread per inch 2
SITUATIONAL PROBLEM 42 (ME Board June 1990)
A square single thread jackscrew has 2 threads per inch. It is to lift 15 tons. The Lead P = 050 in (for single thread)
friction radius of the collar is 1 inch. The coefficient of friction between the treads of Om = Dr + £'Itch = 2.75,. 0.50 = 30 in
the screw and base is 0.15, that between the screw and collar is 0.13. Determine: 2 2
1. The diameter of the root of the screw if the allowable stress in compression is
6,000 psi. tan x = Lead. = 0 )0 = 0.053
A. 0.45 in C. 2.75 in
n Om n (3)
B. 1.50 in D. 3.85 in
Ts W Om tan_~ lJ =
I ~O.OOOill r_~?53 ~~ l
2 l1-ftanx 2 l1- 0.15(0.053)J
2. The torque required to turn the screw.
A. 6826.45 in-lbs C. 8734.57 in-Ibs T s = 9,210.60 in-Ibs
B. 7834.40 in-Ibs D. 9,210.60 in-Ibs

3. The pull required at the end of a 5 ft bar raises the load. 3. Te = fe W rc = 0.13 (30,000)(1) = 3,900 in-Ibs
A. 218.51 Ibs C. 265.301bs T = total torque = Ts + T, = 9,210.60 + 3,900
B. 245.231bs D. 287.451bs T = 13,110.60 in-lbs
T = F x Length of arm
4. The twisting moment exerted at the root of the threads. 13,110.60 = P (5x12)
A. 9,210.60 m-lbs C. 9304.56 in-lbs P = 218.51 Ibs
B. 9563.40 in-Ibs D. 9873.45 in-Ibs
4. Twisting moment or torque at the root of the threads
5. The power output of the screw if it is turning at 60 rpm. T, = 9,210.60 in-lbs
A. 1.87 hp C. 3.04 hp
B. 4.20 hp D. 2.27 hp 5. V = L N = (0.5/12) (60) = 2.5 ft/min
Po = _W V = ~?.900)(2.5) = 2.2727 h
6. The power input of the screw. 33,000 33,000 P
A. 10.28 hp C. 14.93 hp
B. 12.46 hp D. 16.93 hp
~.~TN = 2n(218.15x5)(60) _ 12.46 h
6. P,
33,000 33,000 P
7. The efficiency of the screw and collar.
328 SituatioTlul Problems Situutiona[ Problems 329
F
663,595.37 = -n n2~~ (0.175)
7.
e= PPo -__~~27~
I
12.46
=c 18.23%
2
F= 1633.36 KN
SITUATIONAL PROBLEM 44 (ME Board October 1991)
A 1200 mm cast iron pulley is fastened to a 112.50 mm shaft by means of a 28.13 3. Therefore, the safe force to operate the key is 997.59 KN (minimum force)
mm square key 175 mm long. The key and shaft are SAE 1030 steel annealed with T = F. r = 997.59 (0.1125/2) = 56.1144 KN-m
allowable shearing of key of 29.400 psi and compressive of 6765.3 kg/cm 2 . P = 2rrTN = 2rr(56.114)(600/60) = 3525.77KW
1. Force required to shear the key
A. 997.59 KN C. 756.03 KN 4. T=T'
B. 873.45 KN D. 652.34 KN F.r = F.R
997.59 (112.5/2) = F (1200/2)
2. Force needed to compress the key. F = 93.52 KN
A. 1431.93 KN C. 1633.36 KN
B. 1596.24 KN D. 1763.48 KN 5. T=T'
F.r = F.R
3. Safe power of the shaft at 600 rpm. 1633.36(112.5/2) = F(1200/2)
A. 2083.54 'r<:VV C. 4823.49 'r<:VV F = 153.13 KN
B. 3525.77 'r<:VV D. 5623.74 KW

4. What force acting at the pulley rim will shear this key? SITUATIONAL PROBLEM 45 (ME Board October 1992)
A. 93.52 KN C. 75.23 KN A 76 mm bearing using oil with an absolute Viscosity of 0.70 poise running at 500 rpm
2
B. 85.27 KN D. 64.23 KN gives satisfactory operation with a bearing pressure of 14 kg/cm The bearing
clearance is 0.127 mm.
5. What force acting at the pulley rim will crush the key? 1. Determine the unit pressure at which the bearing should operate if the speed is
A. 143.37 KN C. 165.39 KN changed to 600 rpm.
2 2
B.153.13KN D.173.29KN A. 168 kg/cm C 20.30 kg/cm
2 2
B. 18.30 kg/cm D. 22.04 kg/cm
SOLUTION: -- ---.. 2. If the bearing is given a total clearance of 0076 mm, what change should be made
in the oil?
1. For shearing of key: A. 0.176 poise C 0.364 poise
F B. 0.251 poise D. 0.472 poise
Ss=' - r = 112.5 mm
w L SOLUTION:

1. The bearing characteristic number (Sommerfield number) will be equated for


both conditions.
2
_ u n, ( 01
S
S, = 29,400 (101.325/14.7) = 202,650 kpa - p'lcdj
F When speed is changed to 600 rpm:
202,650=- -
(0.02813)(0.175) S1 = S2

~0.70)(50.91(~ 1 (0.70)(60~(
F = 997.59 KN 2 2
_ 76. 1

2. For compressive stress of key:


14 0.127 ) - P2 l. j
0.127
2
F P2 = 16.8 kg/cm
S C =--
h
L 2 When the total clearance is changed to 0.076:
2
Sc = 67653 (101.325/1.033) = 663,595.37 Kpa
330 Sit u a t i onri! Problems Sit uci! ional Problems 331
S1 = S2
1. How much energy does the wheel loss?
iQ2~l(~2~ll(' - 76_ J2 =
.
~2)~~2°l r _?6 - j'
'2 A. 120,763 N-m C. 130,458 N-m
14 0,127 B.125,159N-m D.145,609N-m
16,8 0,076
1J2 = 0251 poise
2. What moment opposes the motion of the flywheel?
A. 65.34 N-m C. 75.45 N-m
B. 7034 N-m D. 81.63 N-m
SITUATIONAL PROBLEM 46 (ME Board October 1992)
A Mitsubishi car transmission has two spur gears in mesh having a velocity ratio of SOLUTION:
1.4. The number of teeth in the driven gear is 36 and the diametral pitch of 3.
1. Determine the number of teeth in the driver.
A. 22 teeth C. 30 teeth
B. 26 teeth D. 34 teeth

2. Determine the distance between centers.


A. 6.23 in C. 10.34 in
B. 8.30 in D. 12.09 In

3. What is the circular pitch?


A. 0.456 in C. 128 in Journal
B. 0.847 in D. 1.047 in Bearing

SOLUTION:
1. From Faires, p. 535
1. T 1 N 1 = T2 N2 T2 = 36 teeth m k2 2 2
,'I,KE = - - (W1 w2)

=T{~~ j
2
T2 where: m = 910 kg
k radius of gyration = 1524 mm = 1.524 m
36 = T 1 (1.4) Driven 120 (2rr)
T 1 = 25.7 say 26 teeth Driver W1 = ------.-- = 4 rr rad
60
T T 60 (2rr)
2. DP =-- or 0 = ~
w1 =---=2rr rad
o 60
DP c
O1
26 .
= - = 8.67 In 6.KE = 91 0 (1~524 )2 [( 4rr)2 _ (2rr)2]= 125,159 N _ m
3
36 .
02 = - = 12 In 2. Opposing moment = Tangential force x Radius = (f W) r
3
910 (0.06) (0.3048/2)
C = center distance
= 8.321 kg-m = 81.63 N-m
C = ~ + O2 = 8.67 + 12 = 1034 in
2 2
SITUATIONAL PROBLEM 48 (ME Board April 1993)
rr 0 _ rr (1 2) = 1.047 in A 2 Y2 in (8.9 cm) diameter shaft is driven at 3600 rpm by a 400 Hp (298.3 KW)
3. Pc = T - 36 motor. The shaft drives a 48 in. (121.9 ern) diameter chain sprocket having an output
efficiency of 85%. Determine:
SITUATIONAL PROBLEM 47 (ME Board October 1992) 1. Torque in the shaft.
A. 7,003 in-Ibs C. 7,563 in-Ibs
A flywheel weighing 910 kg has a radius of gyration of 1524 mm. The shaft journals
B 7,197 in-Ibs D. 8,408In-lbs
are 304.8 mm in diameter and have a coefficient of friction of 006 After the wheel
2. The output force on the sprocket.
reaches 120 rpm, the driving force is withdrawn and the wheel slows to 60 rpm.
A, 291.79 Ibs C. 327.451bs
332 SUuol iOT!ul Problems
B. 302.301bs D. 367.451bs
3. The power delivered by the sprocket. ALCORCO~

A. 320 hp C. 340 hp ENGINEERING REVIEW CENTER


B. 330 hp D. 350 hp
CONGRA TULA TIONS!!
SOLUTION:
TO OUR BOARD TOPNOTCHERS AND
1. P=2nTN
400 (33,000) = 2 rr T (3600) PASSERS
T = 583.57 ft-Ibs = 7,003 in-Ibs
Alcorcon Engineering Review Center maintains high percentage
2. Torque = F. r passing performance for the past 5 years.
7003 = F. (48/2)
F = 291.79 Ibs

3. Power delivered = 400 x 0.85 = 340 hp


October,1999 ME Board Examination
National Passing: 51% Alcorcon Passing: 84%

8{Jril, 2000 ME Board Examination


FIRST PLACE - Engr. Jefferson S. Talledo (MSU-IIT)
National Passing: 48% Alcorcon Passing: 80%

October, 2000 ME Board Examination


National Passing: 47% Alcorcon Passing: 74%

8{Jril, 2001 ME Board Examination


11 TH PLACE Engr. Florante Go (UC)
14TH PLACE Engr. Gilbert Zamayla (XU)
IS THpLACE Engr. Jefferson Amahan (UC)
ALCORCON PASSING: 84% NATIONAL PASSING: 45%

October, 2001 ME Board Examination


5th - Place Engr. Joe Rey N. Dumandan (CIT)
10th - Place - Engr. Clyde S. Igot (XU)
NATIONAL PASSING = 43% ALCORCON PASSING = 83%

Anda mungkin juga menyukai